You are on page 1of 122

Number System

Answer and Explanations 1 1


STEPWISE MARKING SOLUTION p
4. Numbers which can be written in the form of where p, q are
q
Objective Type Question integers and q ≠ 0 are called rational numbers. Otherwise they
( ) ( 7 ) + ( 3) are irrational numbers.
2 2 2
1. (b) 7− 3 = −2 7 × 3
∴ 28 is an irrational number. (1 Mark)
= 7 + 3 − 2 21 = 10 − 2 21 p
5 5. Numbers which can be written in the form of where p, q are
5 3 16 16 16 q
32 3
( )

2. (b) = 3
32
= 3 5
3215 = 25 15 = 23 integers and q ≠ 0 are called rational numbers. Otherwise they
32 5 are irrational numbers.
48 48 4589
3. (d) = = 16 4 is a rational number.
= ∴ 0.4589 = is a rational number (1 Mark)
3 10000
3
p
245 6. 10.1010010001… cannot be expressed in the form of
= 49 7 is a rational number.
= q
5
∴10.1010010001….. is an irrational number. (1 Mark)
343 × 7= 73 × 7= 7 4 = 72 = 49
p
is a rational number. 7. Numbers which can be written in the form of where p, q are
q
216 × 4= 63 × 2= 12 6 is not a rational number. integers and q ≠ 0 are called rational numbers. Otherwise they
4. (d) 3
625= 3 3
5 ⋅ 5= 5 5 is not a rational number
3 are irrational numbers.
21
p is not a rational number. 441= 21 = is a rational number. (1 Mark)
1
48 = 4 3 is not a rational number. 8. (6 + 7 )(3 + 5 ) =18 + 3 7 + 6 5 + 35  (1 Mark)
( )
1
4
4
256 = 4 = 4 is a rational number.
4
3 3
9. 2401= 4
( 7 )=
4 4
7=3 343  (1 Mark)
5. (b) 4.9 = 4.9999…….. = 5 8
8 2 6
21 3 −
6. False 10. =2
213 = 3
21
=3
=2 441 
21 (1 Mark)
7. True
213

Exercise 2
8. am – n
 1 6 8
9.   =
1. 0.54
= and 0.615384  (1 Mark)
 36  11 13
3 ∴ Three irrational numbers between them are as follows.
10. (a) 
3 is a non – zero rational number and is an irrational 0.55055505555….
5
number 0.567568569 ….
Hence, both (A) and (R) are true and (R) is the correct 0.60660666…. (1 Mark)
explanation of (A) 2 2 13 2 13
2. (a) = =  (1 Mark)
( )
2
13 13 13
Subjective Type Question
1 17 + 16
(b) =
Exercise 1
4
17 − 16 ( 17 − 16 )( 17 + 16 )
1. “Every rational number is an integer” is false. For example, 17 + 16
5 =  (1 Mark)
( ) − ( 16 )
2 2
is a rational number but not an integer.  (1 Mark) 17

2. “Irrational numbers are not real numbers” is false. Since the 17 + 16


= = 17 + 16  (1 Mark)
union of set of rational and irrational numbers is the set of real 17 −16
numbers. (1 Mark)
3. Let x = 0.231 …(1)
3. The union of set of rational numbers and set of irrational
numbers is the set of real numbers. Multiply x by 1000, we get
∴ Given, statement is true. (1 Mark) 1000x = 231.231231231 …(2) (1½ Marks)
On subtracting equation (1) from (2), we get 999x = 231.
231 77
= x =  (1½ Marks)
999 333
2 Fundamental of Mathematics

4. 0.003 3. (a) Let x = 2.9999 …(1)

Let x = 0.003333 …(1) Let us multiply x by 10 we get


Let us multiply x by 1000, we get 10x = 29.9999 …(2) (1 Mark)
1000x = 3.3333 ...(2) (1½ Marks) By subtracting (1) from (2), we get
On subtracting (1) from (2), we get 9x = 27 ⇒ x = 3 (1 Mark)
999x = 3.33 (b)
3.33 333 111 1 13 1 0.07692307...
= x = = =  (1½ Marks)
999 99900 33300 300 0
2 1 100
5. =2× = 0.1176470588235294  (1 Mark)
17 17 91
5 1 90
=5× = 0.2941176470588235  (1 Mark) 78
17 17
7 1 120
=7× = 0.4117647058823529  (1 Mark) 117
17 17
30
Exercise 3 26
1 40
1. (a) is an irrational number. (1 Mark)
3− 5 39
100
2− 3
(b) is an irrational number (1 Mark) 91
2+ 3 9
16 − 64 + 60
(c) 1
4 + 15 ∴ = 0.076923
13
16 − 8 + 4 ×15 8 + 2 15 There are six digits in the repeating block of digits.
= =
4 + 15 4 + 15  (2Marks)
5 5
2(4 + 15)
=
4 + 15
= 2 is a rational number. (1 Mark) 4. (a) ( )
729 6 = 36 =
6 5
3= 243  (1 Mark)
−2
−2
1 1
13 − 8 − 121 (b) ( 2197) = (13 ) 3
3 3 −2
13= =  (1 Mark)
(d) 132 169
3 − 18 2
1 10 1 + 10 − 2
 1 3
13 − 4 × 2 −11 2−2 2 (c) 5 .5 .   = 5
3 3 3
= =  5
3− 9× 2 3−3 2 11 − 2 9

( ) = 2 is a rational number. (1 Mark)


3
2 1− 2 = 5 3
= 5=
3
5= 125 
= (1 Mark)
3 (1 − 2 ) 3
9
9 7 2
11 2 −
(d) 7
= 112 2
= 112 = 11  (1 Mark)
2+ 9 − 4 + 5 11 2
(e)
9+ 5 3 1 3 1 3 3 1 3 1 4
( 2 × 7) 4 .7 4 = 2 4.7 4.7=
+
14 4.7 4 4
74 4 74
2+3−2+ 5 (e)  3 = 3 3
=
= 25 25
3+ 5 52.2 4 52.2 4 25.2 4
1
7 7
3+ 5 = =
= = 1 is a rational number. (1 Mark) 25 25
3+ 5
5. Let the number line be as follows:
2. Given,
225
(a) is terminating decimal
256
(since 256 = 28) (1Mark) D
367
(b) is terminating decimal
640
(∵ 640 = 27× 5) (1 Mark)
891
(c) is non-terminating decimal
960 –5.6 –5 –4 –3 O –2 –1 0 1 E
(∵ 960 = 26 × 3 × 5) (1 Mark) A B C
783
(d) is non-terminating decimal Consider the number line. Mark the point A, B and C at –5.6, 0,
560 1 on the number line. Mark the point O on the number line such
(∵ 560 = 24 × 5 × 7) (1 Mark) AC
that OA = OC =  (1 Mark)
2
Number System Answer and Explanations 3

Draw a semicircle with O as centre and OC as radius. Draw


( )
3 −2
2
a perpendicular to AC passing through B and intersecting the 53.25−2 5 . 5 53.5−4
17. (c) = =
semicircle at D. 5 7
5 7
57
6.6 1
OD
= = 3.3 (∵ OD = OA = OC) = 53 .5–4 .5–7
2 = 53 + (–4) + (–7) = 53–11 = 5–8
6.6 1
OB = OB= − 1= 2.3 = 8
2 5
Hence, (A) is true but (R) is false
BD = OD 2 − OB2 = ( 3.3) 2 − ( 2.3) 2 = 5.6  (2 Marks)
3 4
Now draw an arc with B as centre BD as radius, which intersect 18. (d) Between and their exist infinite rational number
the number line in E. 5 5
∴ BE = BD ⇒ BE = 5.6 31 32 33 34
i.e. , , , −−−−−−−
50 50 50 35
E represents 5.6 on the number line. (1Mark)
Hence, (A) is false but (R) is true

PROBLEMS FOR ADDITIONAL PRACTICE Subjective Type Question


1. Natural numbers: 6, 45, 4, 8
Objective Type Question
12 2. Whole numbers: 6, 45, 4, 8
4 3 12 1
1. (c) x 5 = x 5 × 3× 4 × 2 = x 10 3. Integers: –25, 6, 45, 4, 8
2. (d) 1.25 = 1.2525….. is a rational number. 2 11 −13
4. Rational numbers: –25, 6, 45, , , , 4, 8.
0 is a rational number 9 6 7
1.342323…… = 1.34 23 is a rational number. 1 7
5. Irrational numbers: 2, ,
3 3
1.010010001 is neither repeating nor terminating.
33 66
∴ 1.010010001 is an irrational number. 6. = = 0.66 . It is terminating decimal.
50 100
3. (d) Clearly 11.14211421…. is the biggest number. 13
7. = 1.857142 . It is non-terminating recurring decimal.
1 1 5− 6 7
4. (b) = ×
5+ 6 5+ 6 5 − 6 3
8. = 0.27 . It is non-terminating and recurring decimal.
5− 6 5− 6 6− 5 11
== =
25 − 36 ( −9 ) 9
9.
13
= 1.625. It is terminating decimal.
5 5 5
8
 1 2  1 2  1 2 1 1
(2 =
)
5

5. (d)   =  4  =  4 =
−4 2 −10
2= = 2 44
 16  2   2  210
1024 10. 6= = 6.285714 . It is non terminating recurring decimal.
7 7
1 1
( )
−2
6. (a) 3− 2 = = 28
( ) 11. = 3.111111………
2
3− 2 3+ 2− 2 3 × 2 9
1 1 5+2 6 5+2 6 It is non-terminating and recurring decimal.
= × = = 5+ 2 6
5−2 6 5−2 6 5+2 6 25 − 24 56
12. = 1.142857 …….
−2 49
9
( )
3 1
−2
+ 34 1 1
7. (d) = 93 = 912 = 32 = 36
12
−3 It is non-terminating and recurring decimal
9 4

24
8. True 13. = 3.428571 ……
7
9. True It is non-terminating and recurring decimal.
10. False 25
14. = 8.33333.........
11. True 3
It is non-terminating and recurring decimal.
a −b
12. 1
a −b 15. = 0.166666........
6
13. a – b It is non-terminating and recurring decimal.
14. 2 −1 3
16. = 0.6
15. 22 5
It is terminating decimal.
16. (a) 
The decimal expansion of the number 3 is non –
6
terminating non-recurring 17. = 1.5
4
Hence, both (A) and (R) are true (R) is the correct
explanation of (A) It is terminating decimal
4 Fundamental of Mathematics

9 = 12 × 2 3 + 18 2 + 6 6 + 9
18. = 1.8
5
= 9 + 18 2 + 24 3 + 6 6
It is terminating decimal
13 1 20 − 17
9. =
19.
40
= 0.325 20 + 17 ( 20 + 17 )( 20 − 17 )
It is terminating decimal.
33 =
20 − 17
=
( 20 − 17 ) = 20 − 17
20. = 0.66 2 2
50 ( ) ( )
20 − 17
20 −17 3
It is terminating decimal.
1 8+2
10. =
Exercise 2 8 −2 ( 8 −2 )( 8+2 )
25 30
1. 5 =
= ,6 8+2 8+2 2 2+2 2 +1
4 +1 4 +1 = = = =
( 8)
2
− ( 2)
2 8−4 4 2
26 27 28 29
Four rational numbers between 5 and 6 are , , , .
5 5 5 5 Exercise 3
2 6 × 2 12
2.= = 1 1

1
7 6 × 7 42 1. (a) (81)
= 2 2 2
(9=) 9=2
9
3 6 × 3 18 3 3
= =
7 6 × 7 42 (b) 256=
8
( 2 )=
8 8
2=3 8
6 8 6 8 14
2 3 +
∴ The five rational numbers between and are 2. (a) 37 .3=
7
37 =
7
3 7= 3=2 9
7 7 5
 1  1 1
13 14 15 16 17 (b)   = 10 =
, , , and  22  2 1024
42 42 42 42 42
3. (a) The RF of 5 is 5 3. (a) 4 − 63 + 63 =
4 , which is a rational number.
1 5 5 4 8 4
∴ = = (b) = which is a rational number.
2 5 2 5× 5 10 13 8 13
(b) The RF of 5 – 2 is 5 + 2 1 1× 3 3 1
(c)  = = = × 3 , which is an irrational


3
=
3 5+ 2
=
(
3 5+ 2
=
3 5+ 2 ) ( ) ( ) 3
number.
3× 3 3 3

5− 2 (
5− 2 5 + 2 25 − )(
2 23 ) (d) 2 5 + 6 . This is an irrational number.
4. Let x = 0.8 2
(e) π , π is an irrational number.
3
x = 0.88888 … (1)
Now, multiply x by 10 2π
∴ is an irrational number.
10x = 8.8888 … (2) 3
On subtracting equation (1) from (2), we get 4. Here, we have to locate 4.876 on the number line, which lies
8 between 4 and 5. Let us divide the portion between 4 and 5 into
9x = 8 ⇒ x =
9 10 equal parts.
8 4 4.1 4.2 4.3 4.4 4.5 4.6 4.7 4.8 4.9 5
Hence, 0.8 =
9
5. Let x = 0.43 = 0.43333…… … (1) Since, it lies between 4.8 and 4.9. Let us divide the portion
Multiply x by 10. between 4.8 and 4.9 into 10 equal parts.
10x = 4.333…..
4.8 4.9
⇒ 10x = 4.3333…… … (2)
On subtracting equation (2) from (1), we get 4.81 4.82 4.83 4.84 4.85 4.86 4.87 4.88 4.89
⇒ 9x = 3.9
3.9 39 13 Again, 4.876 lies between 4.87 and 4.88. Let us divide the
⇒ = x = =
9 90 30 portion between 4.87 and 4.88 into 10 equal parts.
4.88
( 4 + 5 ) ( 4 − 5 ) =4 − ( 5 ) 
2
6.
2 4.87
4.871 4.872 4.873 4.874 4.875 4.876 4.877 4.878 4.879
= 16 – 5 = 11

( ) ( 2 ) + ( 3 ) − 2 ( 2 )( 3 )
2 2 2
7. 2− 3 =
This is the required point
= 2+3–2 6 = 5−2 6

8. (2 )(
6 +3 6 2 +3 ) 5. Representation of 6 on the number line:
Let ‘ℓ’ be the number line.
Number System Answer and Explanations 5

Mark 0, 1, 2, 3 ……. on the number line. 635


Let A represents ‘0’ and B represents 2 on the number line. (b) is terminating decimal. (∵ 3200 = 27 × 52)
 3200
Draw AP perpendicular to ‘ℓ’ such that AP = 1 unit. 1
458
Join B and P (c) is non-terminating decimal
3584
A, B and P form is a right triangle ⇒ BP2 = AB2 + AP2
BP2 = 22 + 12 (∵3584 = 512 × 7 = 29 × 7)
BP2 = 5 32
(d) is non-terminating decimal
BP = 5 1050
Draw an arc to cut the number line at ‘C’ with A as centre and (∵1050 = 2 × 3 × 52 × 7)
BP as radius. 9345
(e) is non-terminating decimal.
∴ AC = BP = 5 453600
Join C and P. (∵453600 = 25 × 34 × 52 × 7)
Now CPA is a right triangle.
( )( )=( )
2
CP2 = AP2 + AC2 3 −1 3 −1 3 −1 3 −1
11. =
CP2 = 12 + ( 5)
2
3 +1 ( 3 + 1) ( 3 − 1) 3 −1

CP2 = 1 + 5 3 − 2 3 +1 4 − 2 3
= = = 2− 3
CP = 6 2 2
Draw an arc to cut the number line at D with A as centre Compare 2 − 3 with a + b 3
and CP as radius.
∴ a = 2, b = – 1
∴ AD = CP
12. x= 3 + 2 2
⇒ AD = 6
1 1 3− 2 2 3− 2 2
∴ D represents 6 on the number line. = = =
x 3+ 2 2 ( 3+ 2 2 )( 3− 2 2 9−8 )
2 = 3−2 2
1
x + = 3+ 2 2 +3− 2 2 = 6
x
2 2
 1  2 1  1  1
P  x +  = x +2 x +  = x+2+ x
22 +12 = 5 x x  x
1 Unit 1  1 
= x+ + 2 = 6 + 2  x + =6 = 8
22 +12 = 5 x  x 
ℓ 1
0 1 2 C D 3 ∴ x+ = 8 =2 2
x
In the figure above the point D represents the position
of 6. 13.
330 + 329 + 328
=
328 32 + 3 + 1
=
(
9+4
=
13 )
1 1 331 + 330 − 329 328 33 + 32 − 3 (
27 + 9 − 3 33 )
6. (a) 15625
= 6
(=
5 ) 6 6
5
1 2− 3 2− 3
14. = = = 2− 3
(2 + 3) (2 − 3) 4 − 3
3
3 3

( )

( 625 ) 2+ 3
4
4 4
(b) = 5
= 5 4
= 53 = 125

2( 5 + 3) 2( 5 + 3)
3 3
7. (a) 1024= 5
( 4 )=
5 5
4=3 64 2
= = = 5+ 3
(b)
−4
243 = 3
5
( )
−4
5 5 1 1
= 3−=4 =
5− 3
( )
5 − 3 ( 5 + 3 ) 5−3
34 81
1 2+ 5 2+ 5 2+ 5
1 1 1 1 1 = = = = − 5−2
( )( )
1 1 1
8. (a) 6 .9 =( 9 × 6) =( 54) = (3 × 2) =(3
3 3 3 3
3 3
)
3 3
× 2 =3 × 2
3 3
2− 5 2− 5 2+ 5 4−5 −1

5 5
 5  1 1
5
1
5
1 2 1
(b) = =    = 5 = ∴  + +
10 5
10 2 2 32 2+ 3 5 − 3 2− 5
= 2− 3+ 5+ 3− 5 −2=
0
9. We have to locate 4.73. First, we see that 4.73 lies between 4
and 5. In the next step, we observe that 4.73 lies between 4.7 15. x= 2 + 5
and 4.8. We divide this portion of the number line into 10 equal x2 = 4 + 4 5 + 5 = 9 + 4 5
parts and we see that 4.73 lies between 4.73 and 4.74. Again by 1 1 9−4 5 9−4 5
= = = = 9−4 5
dividing it into 10 equal parts, we notice that 4.73 lies between
4.733 and 4.734. Again by dividing it into 10 equal parts, we
x2 9 + 4 5 ( 9+4 5 9−4 5 )(
81 − 80 )
notice that it is located closer to 4.7333 than to 4.7334.
125
x2 −
1
x2
(
= 9+4 5 − 9−4 5 = 8 5 ) ( )
10. (a) is non terminating decimal. (∵ 512 = 29)
512
6 Fundamental of Mathematics

HOTS 4. 1280 − 245 + 1125 − 80

1. Given, x = 1.34 and y = 0.47 = 256 × 5 − 49 × 5 + 225 × 5 − 16 × 5


134 − 13 121 = 16 5 − 7 5 + 15 5 − 4 5
1.34
⇒= =
102 − 10 90 = 31 5 − 11 5 =
20 5 .
47 − 4 43 1
0.47 =
⇒= 5. = a 3 −b 2
2
10 − 10 90 2 3− 8
121 43 164 1 1 2 3+ 8
x + y= + = = ×
90 90 90 2 3− 8 2 3− 8 2 3+ 8
121 43 78
x − y= − = 2 3+ 8 2 3+ 8
90 90 90 = =
(2 3) − ( 8)
2 2
12 − 8
164 78 12792
∴ x 2 − y 2 = ( x + y )( x − y ) = × = = 1.57925 .
90 90 8100 2 3+ 8 2 8
= = 3+
22 4 4 4
2. π =
7 3 2 2 3 2
= + = +
7 22 3.1428571 2 4 2 2
21 3 2
10 Compare + with a 3 − b 2
7 2 2
30 1 1
28 ∴ a= ,b = −
2 2
20
14 1  1 1 1
Hence, a − b = − −  = + = 1.
60 2  2 2 2
56 4 3 3
40 6. 4096 + 729 − 3
15625
35 4096 2=
= , 729 3 ,1
= 17
5625 56 6

50 4 3
49 ∴ 4096 = 12 4096 (∵ m n a = mn a )
10 3
729 = 6 729
7
1 3
15625 = 6 15625
4 3 3
∴ π = 3.142857 …. ∴ 4096 + 729 − 3
15625
2π = 2 × 3.142857 …..= 6.285714…. = 12
4096 + 729 − 15625
6 6

To find two irrational numbers between π and 2π, we must


write two nonterminating and non-terminating numbers = 212 + 6 36 − 6 56
12

between 3.142857…… and6.285714 ….. =2+3–5=0


∴ 4.101210015……., 5.230210045000…… are two 1 1 1
irrational numbers between π and 2π. 7. a −b a −c
+ b−c b−a
+
1+ x + x 1+ x + x 1 + x + xc −b
c−a

2 × 22 × 23 × …… × 2n 1 1 1
3. = 32678 = + +
2n xa xa xb xb xc xc
1+ b + c 1+ c + a 1+ a + b
2(1+ 2 + 3+….+ n) x x x x x x
⇒ = 215
2n 1 1
n ( n +1)
+ +
x b .x c + x a .x c + x a .x b x a .x c + x a .x b + x b .x c
2 2
⇒ = 215 x b .x c x c .x a
2 n =
1
n2 + n
⇒ 2 2
−n
= 215 x a .x b + x b .x c + x c .x a
n2 + n − 2 n x a .x b
⇒ 2 2
= 215 xb + c xa + c
n2 − n b+c a+c a+b
+ a+c +
⇒ 2 = 215 x +x +x x + x a + b + xb + c
2
=
xa +b
Since, the bases are same, then the powers are equal, we
x + xb + c + xc + a
a+b
get
n2 − n xb + c + x a + c + x a + b
⇒ = 15 ⇒ n 2 − n =30 ⇒ n ( n − 1) =
30 = b+c =1
2 x + xa + c + xa +b
n ( n − 1) = 6 × 5 ⇒ n ( n − 1) = 6 × ( 6 − 1)
x +1
⇒ 8. 9 × 8 = 1 + 3 + 5 + 7 + …..… + 23
n=6 24
∴ ⇒ (12)
9 × 8 x +1 =
2
(∵2n – 1 = 23 ⇒ =
n = 12 )
2
Number System Answer and Explanations 7

Sum of the first ‘n’ odd natural number is n 2 48 > 45 > 40 > 30
4 3 2 1
⇒ 9 × 8 x +1 = 144 ⇒ > > >
x +1 144 5 4 3 2 1
⇒ 8= = 16
9 ∴ 5
16 > 8 > 4 > 2 .
4 3

⇒ (2 ) 3 x +1
=2 4
 xa 
2
 xb 
a
 xc 
2 a 2 c

13.  bc  ×  ac  ×  ab 
⇒ 23 x + 3 = 2 4 x  x  x 
⇒ 3x + 3 = 4
3 3 3 3 3 3 3 3 3

⇒ 3x = 1 xa xb xc xa +b +c xa +b +c
= × abc × abc = abc + abc + abc =
1 x abc
x x x x 3abc
∴ x= .
3 ∵ a+b+c=0
144 ⇒ a3 + b3 + c3 = 3abc
=
9. x 2 0.0144
=
10000 3 3 3
xa +b +c x 3abc
144 12 12 ∴ = = 1.
⇒ =x = = = 0.12 x 3abc x 3abc
10000 102 100
144 12 1728 14. x3 + 1 = 2198, y3 + 1 = 1332
∴ x3 = x 2 × x = × = = 0.001728. x3 = 2198 – 1; y3 = 1332 – 1
10000 100 1000000
= 2197 = 1331
16 4 4 2
10. = = , ⇒ x3 = (13)3; y3 = (11)3
81 9 9 3
⇒ x = 13 y = 11
a c a+c
If and are two rational numbers, then is a rational x + y 13 + 11 24
b d b+d ∴ = = = 12.
x − y 13 − 11 2
a c
number between and
b d x
15. = 5−2
4 6 2 4 1 2 4 5 1 2 1 1 5+2
, , ⇒ , , ⇒ , , , = ×
9 12 3 9 2 3 9 11 2 3 x 5−2 5+2
16 4 5 5+2 5+2
∴ The two rational numbers between and are = = = 5+2
81 9 11
( 5)
2
− ( 2)
2 5−4
1
and . 1
2 ∴ x+ = 5 −2+ 5 +2 = 2 5
x
11 3 11 3 11 3
=
11. = Squaring on both sides, we get
847 − 363 121 × 7 − 121 × 3 11 7 − 11 3
2
1

( 2 5 )=
2
11 3 3  x + = 20
= = x
11( 7 − 3) 7− 3
1
=
3
=
3 7+ 3 ( ) 3 ( 7+ 3 ) ⇒ x2 +
x2
+2=20

7− 3 ( 7− 3 7+ 3 )( ) 7−3
⇒ x2 +
1
= 20 − 2
x2
=
3 ( 7+ 3 )= 21 + 3
. 1
4 4 ⇒ x2 + 18
=
x2
12. 4
8, 2, 5 16, 3 4 1  1  1
∴ x3 + =  x +   x2 − 1 + 2 
⇒ 4 3
2 , 2, 2 , 2 5 4 3 2 x3  x  x 


3
2 ,22 ,25 ,23
4
1 4 2
( )
= 2 5 (18 − 1) = 34 5 .

Since, the bases are same, comparing the powers. A


16. = 5 + 11 , =
B 3 + 13

( ) =( 5 ) + ( 11)
2 2 2
3 1 4 2 A 2 = 5 + 11 + 2 55
, , ,
4 2 5 3
= 5 + 11 + 2 55
= 16 + 2 55
L.C.M of 2, 3, 4, 5 is 60
( ) =( 3 ) + ( 13 )
2 2 2
3 45 1 30 4 48 2 40 B2 = 3 + 13 + 2 39
= = , =, =,
4 60 2 60 5 60 3 60
= 3 + 13 + 2 39 = 16 + 2 39
Since, the denominators are same, comparing the powers.
∵ 55 > 39 ⇒ 55 > 39
8 Fundamental of Mathematics

We have, A 2 > B2 1 1 1
abc = ( abc ) x
+ +
⇒ y z
⇒ A>B yz + zx + xy

∴ 5 + 11 > 3 + 13 . ⇒ abc = ( abc ) xyz

yz + zx + xy
5+ 2 6 ⇒ =1
17. xyz
8 + 2 15 ∴ yz + zx + xy = xyz.

( 2 ) + ( 3) 1
2 2
19. ( 2187 ) =
x
5 + 2 6= +2 2× 3
( 729) y
( )
2
= 2+ 3 = 2+ 3 2187 = 37, 729 = 36
1
( )
x
⇒ 37 =
( 5) + ( 3) ( )
2 2 y
8 + 2 15= +2 5× 3 36
1
⇒ 37 x = 6 y
( )
2
= 5+ 3 3
⇒ 37x = 3–6y
= 5+ 3 Since, bases are same, powers are equal.
5+ 2 6 2+ 3 ⇒ 7x = –6y
∴ = . ∴ 7x + 6y = 0
8 + 2 15 5+ 3
20.=x 4 3 +1
18. ax–1 = bc, by–1 = ac, cz–1 = ab
ax by cz
⇒ ( x − 1) =4 3
⇒ = bc, = bc, = bc
a b c Squaring on both sides

(4 3)
2
⇒ ax = abc, by = abc, cz = abc ⇒ ( x − 1) 2 =
⇒ ax = by = cz = abc
⇒ x2 – 2x + 1 = 48
1 1 1
⇒ a = ( abc ) , b = ( abc ) , c = ( abc )
x y z Subtracting 4 from both sides
⇒ x2 – 2x + 1 – 4= 48 – 4
1 1 1
⇒ abc = ( abc ) x × ( abc ) y × ( abc ) z ∴ x2 – 2x – 3 = 44
Polynomials
Answer and Explanations 2
STEPWISE MARKING SOLUTION 4. The highest degree of a term in a polynomial is the degree of 2
the polynomial.
∴ The degree of the given polynomial is 10. (1 Mark)
Objective Type Question
5. The highest degree of a term in a polynomial is the degree of
1. (b)
573 + 433
=
(
(57 + 43) 57 2 � 57 � 43 + 432 ) the polynomial.
57 2 � 57 � 43 + 432 57 2 � 57 � 43 + 432 ∴ The degree of the given polynomial is 5. (1 Mark)
(∵ a + b = (a + b)(a – ab + b )]
3 3 2 2
6. (a) The given polynomial is 2x – 5x + 6.
2

= 57 + 43 = 100 The value of the polynomial at x = –2 is


2(–2)2– 5(–2) + 6 = 8 + 10 + 6 = 24. (1 Mark)
2. (b) p(y) = 3y3 – 2y2 – y – 10
3 2 1
� 1� � 1� � 1� � 1� (b)  At x = , the value of the polynomial is
⇒ p �� ÷ = 3 �� – 2 �� ÷ – �� ÷ – 10 2
� 3� � 3÷
� � 3� � 3�
2
1 2 1 �1 � �1 �
= – � + �10 = – 10 2 � ÷ � 5 � ÷ + 6.
9 9 3 �2 � �2 �
1 1 �1 � 2 5 2 �10 + 24 16
3. (d) Given, x + = 10 ⇒ x2 + 2 + 2 ( x ) � ÷ = 100 = � +6= = = 4.  (1 Mark)
x x �x � 4 2 4 4
1 1
⇒ x2 + 2 = 98  ... (1) 7. f(x) = 3x2– 4, at x = ,
x 3
2 2
� 1� 1 �1 � 1 �1 � � 1 �
�x� = x2 + 2 – 2(x) � ÷ = x2 + 2 � 2 f � ÷= 3 � � ÷ � 4 = 1 � 4 = �3
� x÷� x � x � x � 3� � 3�
= 98 – 2 [From (1)] = 96 1
1 ∴ is not a zero of f(x). (1 Mark)
⇒ x – = ± 96 = ± 4 6 3
x
2
4. (c) 64x3 + 27y3 = (4x)3 + (3y)3 At x = ,
3
= (4x + 3y) [(4x)2 – (4x)(3y) + (3y)2] 2
= (4x + 3y)(16x2 – 12xy + 9y2) �2 � �2 �
f � ÷ = 3� ÷ � 4 = 4 � 4 = 0
� 3� � 3�
5. (a) 
6252 – 5752 = (625 + 575)(625 – 575)
[∵ a2 – b2 = (a + b)(a – b)] 2
∴ is the zero of f(x). (1 Mark)
= 1200 × 50 = 60000 3
6. False 8. f(x) = x2 – 5x + 6
7. True
Let x2– 5x + 6 = 0
x2– 3x – 2x + 6 = 0
8. y2 – 7y + 12 x(x – 3) – 2(x – 3) = 0 (1 Mark)
9. (x – 12) (x – 2) (x – 3) = 0
x – 2 = 0 or x – 3 = 0
1
1
10. (d) 
x= 3 x = ( 3 x ) 2 +1; exponent of x is which is not a ∴ x = 2 or 3 are the zeroes of the given polynomial.
2  (1 Mark)
whole number
∴ 3 x + 1 is not a polynomial 9. (a) 
When f(x) = 2x3 + 3x2 + 4x + 7 is divided by x – 1, the
remainder is f(1).
Hence, (A) false but (R) is true
∴ f(1) = 2(1)3 + 3(1)2 + 4(1) + 7
= 2 + 3 + 4 + 7 = 16  (1 Mark)
Subjective Type Question 1
(b) 
When 2x3 + 3x2 + 4x + 7 is divided by x � , then the
2
Exercise 1
�1 �
remainder is f � ÷.
1. The highest degree of a term in a polynomial is the degree of �2 �
the polynomial. 3 2
∴ The degree of the given polynomial is 100.  (1 Mark) �1 � �1 � �1 � �1 �
f � ÷ = 2 � ÷ + 3� ÷ + 4 � ÷+ 7
�2 � �2 � �2 � �2 �
2. The highest degree of a term in a polynomial is the degree of
2 3 4
the polynomial. = + + +7
8 4 2
∴ The degree of the given polynomial is 3. (1 Mark)
1 3
3. The highest degree of a term in a polynomial is the degree of = + + 2+7
4 4
the polynomial. = 1 + 2 + 7 = 10 (1 Mark)
∴ The degree of the polynomial is 5. (1 Mark)
2 Fundamental of Mathematics


10. �3 +
2 �� 2 � � 2 �
x÷�3 + x÷ = �3 + x÷ 
2

(1 Mark) ( )
= 2x x + 2 3 + 2 3 x + 2 3  ( ) (1 Mark)
� 3 �� 3 � � 3 �
�2 x �
2
= ( x + 2 3 )( 2 x + 2 3)  (1 Mark)
2 2
= 3 + 2 �3 � x + � ÷ 2. (a) 2 x 2 + 8 y 2 + 3 z 2 + 8 xy + 4 6 yz + 2 6 zx
3 �3 �
2 2 2
(∵ (x + y)2 = x2 + 2xy + y2) =( 2 x ) + ( 2 2 y ) + ( 3z )
4x2 4x2
= 9 + 4x + = + 4x + 9  (1 Mark) + 2 ( 2 x )( 2 2 y ) + 2 ( 2 2 y )( 3 z )  (1 Mark)
9 9
Exercise 2 + 2 ( 3 z )( 2 x )
2

1. Given, f(x) = �x � ÷�x � ÷


� 1 �� 2� = ( 2 x + 2 2 y + 3 z ) = [∴ (a + b + c) 2

� �� � 2 3
= a2 + b2 + c2 + 2ab + 2bc + 2ca] (1 Mark)
� 1 �� 2 � 1 1 1 2
f(1) = �1 � ÷�1 � ÷ = × =  (1 Mark) �1 2 4 �
� 2 �� 3 � 2 3 6 (b) Given, � a + b + c÷
�3 3 3 �
� 1� � 2 � ��1� ��2 � 1
f(0) = �0 � ÷ �0� = × =  (1 Mark) We have, (x + y + z)2
� 2� � 3÷� ��2÷
� � �3÷
� 3 = x2 + y2 + z2 + 2xy + 2yz + 2zx (½ Mark)
� 1� � 2� 2 2
�a 2b 4c � �a � �2b � �4c �
2 2
f(–1) = ��1 � ÷ ��1 � ÷ + + ÷ = � ÷ +� ÷ +� ÷
� 2� � 3� �
�3 3 3 � �3 � � 3 � � 3 �
��3 ���5 � 5 a 2b 2b 4c 4c a
= � ÷� ÷ =  (1 Mark) +2 � � +2� � + 2� �
� 2 �� 3 � 2 3 3 3 3 3 3
2. (a) 
Let f(x) = x6 – 2x3 + 3. a 2 4b 2 16c 2 4ab 16bc 8ac
= + + + + +  (1½Marks)
The remainder, when x6 – 2x3 + 3 is divided by 9 9 9 9 9 9
x + 1, is f(–1)
3. If a + b + c = 0, then a3 + b3 + c3 = 3abc (1 Mark)
∴ f(–1) = (–1)6– 2(–1)3 + 3
(a) Since –13 + 5 + 8 = 0,
= 1 + 2 + 3 = 6 ≠ 0 (1 Mark)
(–13)3 + (5)3 + (8)3 = 3 × (–13) × 5 × 8
∴ (x + 1) is not a factor of x6 – 2x3 + 3 (½ Mark)
= –39 × 40 = –1560 (1½ Marks)
(b) Let f(x) = x2– (3 + 2 )x + 6 (b) Since 100 – 36 – 64 = 0,
The remainder, when (100)3 + (–36)3 + (–64)3
x2 – (3 + 2 )x + 6 is divided by x + 1, is f(–1). = 3(100)(–36)(–64)
= 300 × 2304 = 691200 (1½ Marks)
f(–1) = (–1)2– (3 + 2 )(–1) + 6  (1 Mark)
4. (a) 54x – 99x + 27
2
= 54x – 81x – 18x + 27
2
=1+3+ 2+ 6≠0 = 9x(6x – 9) – 3(6x – 9) (½Mark)
∴ (x + 1) is not a factor of = (9x – 3) (6x – 9)
x2–(3 + 2 )x + 6 (½ Mark) Area of the rectangle  (1 Mark)
= Length × Breadth
3. (a) The remainder when f(x) is divided by g(x) is f(2).
The dimensions of the rectangle
∴ f(2) = (2)2– (2) – 6 = 4 – 2 – 6
are 9x –3 and 6x – 9 (½ Mark)
= 4 – 8 = –4 ≠ 0 (1 Mark)
(b) 7x2– 17x – 12
∴ g(x) is not a factor of f(x). (½ Mark)
= 7x2– 21x + 4x – 12
(b) The remainder when f(x) is divided by g(x) is f(1).
= 7x(x – 3) + 4(x – 3)
f(1) = 13 – 2(1)2 – 1 + 2
= (7x + 4) (x – 3) (½ Marks)
= 1 – 2 – 1 + 2 = 0 (1 Mark)
∴ Area of the rectangle
∴ g(x) is a factor of f(x). (½ Mark)
= Length × Breadth (1 Mark)
3 1 3 27 2 9 2 The dimensions of the rectangle are (7x + 4) and (x – 3).
4. 27 a + b + a b + ab
8 2 4  (½ Mark)
3 2
3 �1 � �1 � �1 � 5. 8x3 + 27y3 + 64z3– 72xyz
�2 � �2 �
( )
= ( 3a ) + � b÷ + 3 � 9a 2 � b÷ + 3 � ( 3a ) � b÷
�2 � = (2x)3 + (3y)3 + (4z)3– 3 × 2x × 3y × 4z (1 Mark)
3 = (2x + 3y + 4z) (4x2 + 9y2 + 16z2– 6xy – 12yz – 8zx)
� 1 �
= �3a + b÷  (1 Mark) [∵ x3 + y3 + z3– 3xyz = (x + y + z)
� 2 � (x2 + y2 + z2 – xy – yz – zx)] (1 Mark)
[∴ (a + b)3 = a3 + 3a2b + 3ab2 + b3] (1 Mark)

Exercise 3 PROBLEMS FOR ADDITIONAL PRACTICE


2
(
1. 2 x + 2 x 2 3 + 3 + 12 ) Objective Type Question
= 2 x + 4 3 x + 2 3 x + 12
2
1. (b) (4p + 1)2 + (1 – 16p2) = (4p + 1)2 + (1 + 4p)(1 – 4p)
= (4p + 1) [(4p + 1) + (1 – 4p)] = 2 (4p + 1)
Answer and Explanations 3

2. (b) Let p(x) = 3x2+ kx – 11 17. Binomial


x + 5 is a factor of 3x2+ kx – 11
18. (a + b + c)2
⇒ p(–5) = 0 ⇒ 3(–5)2+ k (–5) – 11 = 0 1
64 19. 2x(x2 + 3y2)
⇒ 75 – 5k – 11 = 0 ⇒ k =
5 20. (b) Given, p(x) = x2 –3 2 x + 2
3. (a) 100 – 95 – 5 = 100 + (–95) + (–5)
3 3 3 3 3 3 For zeroes of P(x); P(x) = 0
2
Here, 100 + (–95) + (–5) = 0
If a + b + c = 0 then a3 + b3 + c3 = 3abc
( ) � (3 2 )(3 2 ) + 2 = 2
� 3 2

∴ 1003 + (–95)3 + (–5)3 = 3(100) (–95)(–5) = 142500 Both (A) and (R) are true and (R) is not the correct
explanation of (A).
4. (c) f(x) = 5x + 10x – 11
2

⇒ f(–11) = 5(–11)2 + 10(–11) – 11 21. (a) (18)3 + (–6)3+ (–12)3 = 3 (18) (–6) (–12) = 3888
= 605 – 110 – 11 = 484 x3 + y3 + z3 = 3xyz
∵ x + y + z = 0 here 18 – 6 – 12 = 0
5. (d) 216 x2 – 343y3 = (6x)3 – (7y)3 Both (A) and (R) are true and (R) is the correct explanation
= (6x – 7y) [(6x)2 + (6x)(7y) + (7y)2] of (A).
= (6x – 7y) (36x2 + 42xy + 49y2)
22. (d) Given, p(x) = x3 – x + 1; g(x) = 2 – 3x
6. (d) g(x) = 5x3 – 10x2 – 15x + 20 For g(x) = 2– 3x = 0
3 2
� 2� � 2� � 2� � 2� 2
g �� ÷ = 5 �� ÷ �10 �� ÷ �15 �� ÷ + 20 ⇒ x=
� 5� � 5� � 5� � 5� 3
8 8 48 3
�2 � �2 � � 2 �
=– � + 6 + 20 = – + 26 p( x) = P � ÷ = � ÷ � � ÷+ 1
25 5 25 �3 � �3 � � 3 �

7. (c)
613 � 453
=
(
( 61 � 45) 612 + 61� 45 + 452 ) =
8 2
� +1 =
8 � 18 + 27 17
=
612 + 61� 45 + 452 (
612 + 61� 45 + 452 ) 27 3 27 27
Since remainder not equal to zero
∴ [a3 – b3 = (a – b)(a2 + ab + b2)] So p(x) is not multiple of g(x)
= 61 – 45 = 16 Hence,(A) is false but (R) is true
8. (b) 27 x3 – 54x2 + 36x – 8 = (3x)3 –3(3x)2 2 + 3(3x)(22) – 23
= (3x – 2)3 Subjective Type Question:
2
1 � 1�
9. Given, y – = 11 ⇒ �y � ÷ = 11 2 Exercise 1
y � y�
1. An algebraic expression in which the variables involved have
1 �1 �
only non-negative integer exponents is called a polynomial.
⇒ y2+ – 2 (y) � ÷ = 121
y2 �y� Therefore, the given expression is polynomial of degree 3.
1 2. An algebraic expression in which the variables involved have
⇒ y2 + = 123  ... (1)
y2 only non-negative integer exponents is called a polynomial.
2 Therefore, the given expression is a polynomial of degree 2.
� 1� 1 �1 �
∴ � y + ÷ = y2 + 2 + 2( y) � ÷ 3 An algebraic expression in which the variables involved have
� y� y �y� only non-negative integer exponents is called a polynomial.
1 Therefore, the given expression is not a polynomial.
= y2 + + 2 = 123 + 2 [from (1)] = 125
y2 4. An algebraic expression in which the variables involved have
1 only non-negative integer exponents is called a polynomial.
⇒ y+ = ± 125 = ± 5 5
y Therefore, the given expression is not a polynomial.

10. (a) 
5152 – 4852 = (515 + 485) (515 – 485) [∵ a2 – b2 = (a + b) 5. An algebraic expression in which the variables involved have
(a – b)] only non-negative integer exponents is called a polynomial.
= 1000 × 30 = 30000 Therefore, the given expression is not a polynomial.

11. (d) 
(3a – 2b)2+ (4b2 – 9a2) = (3a – 2b)2 + [(2b)2 – (3a)2] = (3a 6. The given polynomial is 2x2 – 5x + 6
– 2b) + (2b + 3a) (2b – 3a)
2 (a) When x = 1, the value of the polynomial is 2(1)2– 5(1) + 6
= (3a – 2b) [(3a – 2b) – (2b + 3a)] =2–5+6=3
= –4b(3a – 2b) (b) at x = 3, the value of the polynomial is 2(3)2 – 5(3) + 6
3a – 2b, b and 4 are factors. = 18 – 15 + 6 = 9

12. True 7. (a) f(x) = 3x2– 5 at x = 2


f(2) = 3(2)2– 5 = 12 – 5 = 7
13. False ∴ x = 2 is not a zero of f(x)
14. True 1
(b) f(x) = 14x2 + 7x –7 at x =
15 False
2
�1 � 1 1 7 7
16 Linear f � ÷ = 14 � + 7 � � 7 = + � 7 = 0
�2 � 4 2 2 2
4 Fundamental of Mathematics

1 3. (a) When 2x3 + 3x2 + 4x + 7 is divided by x + 2


∴ x= is the zero of f(x). The remainder is f(–2).
2
∴ f(–2) = 2(–2)3 + 3(–2)2 + 4(–2) + 7
8. (a) Given, f(x) = 2x – 5
= 2(–8) + 3(4) + 4(–2) + 7
Let 2x – 5 = 0 for some values of x.
= –16 + 12 – 8 + 7 = 19 – 24 = –5
5
⇒ 2x – 5 = 0 ⇒ 2x = 5 ⇒ x = (b) When 2x3 + 3x2 + 4x + 7is divided by 2x + 1
2
��1�
5 The remainder is f � ÷
∴ is the zero of f(x) �2�
2 3 2
(b) Given, f(x) = ax ��1� ��1� ��1� ��1�
f � ÷= 2� ÷ + 3 � ÷ + 4� ÷ + 7
Let f(x) = 0 �2� �2� �2� �2�
ax = 0 ⇒ x = 0 �2 3 4
∴ x = 0 is the zero of the given polynomial. = + � +7
8 4 2
9. (x + a) (x + b) = x2 + (a + b) x + ab �1 3 2 22 11
= + � 2 + 7 = +5 = =
(x + 5) (x + 10) = x2 + (10 + 5)x + 10 × 5 = x2 + 15x + 50 4 4 4 4 2
10. (108) (107) = (100 + 8) (100 + 7) 4. (a) Let f(x) = x3 + x2 + x + 1
= 100 + (8 + 7) × 100 + 8 × 7 = (100) + 15 × 100 + 56
2 2 The remainder when f(x) is divided by x + 1 is f(–1)
= 10000 + 1500 + 56 = 11556 f(–1) = (–1)3 + (–1)2 + (–1) + 1
11. (a) Coefficient of x is 24. = –1 + 1 – 1 + 1 = 0
(b) Coefficient of x is – 1. ∴ (x + 1) is a factor of x3 + x2 + x + 1
(b) Let f(x) = 10x4– 5x3 + 3x2– 4x + 7.
12. (a) Coefficient of x is 0.
The remainder when f(x) is divided by x + 1 is f(–1)

(b) Coefficient of x is f(–1) = 10(–1)4– 5(–1)3 + 3(–1)2– 4(–1) + 7
2
= 10 + 5 + 3 + 4 + 7 ≠ 0
13. Cubic Polynomial ∴ (x + 1) is not a factor of 10x4 – 5x3 + 3x2 – 4x + 7
14. Linear polynomial 5. (a) Given, f(x) = 2x3– 3x2 + 5x + 6
15. Quadratic polynomial and g(x) = (x – 2)
3 When f(x) divided by g(x) the remainder is f(2).
16. f(x) = 5 x �
2
x �5
5 f(2) = 2(2)3– 3(2)2 + 5(2) + 6
2 3 3 3 = 16 – 12 + 10 + 6 = 20 ≠ 0
f(1) = 5 (1) � (1) � 5 = 5 � � 5 = � ∴ g(x) is not a factor of f(x).
5 5 5
2 3 (b) Given, f(x) = x3 – ax2 – x – a + 1 and g(x) = x – a
f(0) = 5 ( 0) � (0) � 5 = � 5 When f(x) divided by g(x) the remainder is f(a)
5
3 3 3 f(a) = a3 – a.a2 + a – a + 1
2
f(–1) = 5 ( �1) � ( �1) � 5 = 5 + � 5 = a3– a3 + a – a + 1 = 1
5 5 5
∴ (x – a) is not a factor of f(x).
17. f(y) = y3– 27; f(1) = 1 – 27 = –26;
6. (a) (104) (96) = (100 + 4) (100 – 4) = (100)2– 42
f(0) = 0 – 27 = –27; f(–1) = –1 – 27 = –28
∵ {(x + y) (x – y) = x2– y2} = 10000 – 16 = 9984
18. f(x) = (x + 1) (x – 2) (b) (205) (195) = (200 + 5) (200 – 5)
When x = 2, f(2) = (2 + 1) (2 – 2) = 3 × 0 = 0
= (200)2– (5)2 [∵ (x + y) (x – y) = x2 – y2]
∴ x = 2 is a zero of f(x)
= 40000 – 25 = 39975
When x = 3, f(3) = (3 + 1) (3 – 2) = 4
∴ x = 3 is not a zero of f(x) �1 1 �3
7. (a) � a � b÷
19. Identity: (x + a) (x + b) = x2 + (a + b) x + ab �2 3 �
(x – 2) (x + 11) = x2 + (11 – 2)x + 11 × (–2) = x2 + 9x – 22 3 3 2 2
�1 � �1 � �1 � �1 � �1 � �1 �
= � a ÷ – � b÷ – 3 � a ÷ � b÷ + 3 � a ÷ � b÷
20. Identity: x – y = (x + y) (x – y)
2 2
�2 � �3 � �2 � �3 � �2 � �3 �
2
5 5 �5 � 25 1 1 1 1
(x2 + ) (x2– ) = (x2)2– � ÷ = x4– = = a 3 � b3 � a 2b + ab 2
2 2 � 2 � 4 8 27 4 6
3 2 3
Exercise 2 � 1 � �q � �q � �q �
(b) � p � q = p3– 3.p2 � ÷ + 3. p � ÷ � � ÷
� 4 ÷ � �4 � �4 � �4 �
1. f(x) = 2x2 – 5x + 17
f(1) = 2(1)2– 5(1) + 17 = 2 – 5 + 17 = 2 – 5 + 17 = 14 1 3 3
= p 3 � q 3 � p 2 q + pq 2
f(0) = 2(0)2 – 5(0) + 17 = 17 64 4 16
f(–1) = 2(–1)2– 5(–1) + 17 = 2 + 5 + 17 = 24 8. (a) 3x2 – 7x + 2 = 3x2 – 6x – 1x + 2
2. x2 – 11x + 18 = x2– 9x – 2x + 18 = 3x(x – 2) – 1(x – 2) = (x – 2) (3x – 1)
⇒ x(x – 9) – 2(x – 9) = (x – 9) (x – 2) (b) 4x2– x – 5 = 4x2– 5x + 4x – 5
= x(4x – 5) + 1(4x – 5) = (4x – 5) (x + 1)
Answer and Explanations 5

9. (a) Since x + 1 is a factor of x2 – ax + 5, ( x + 30 y ) ( x 2 � 30 xy + 900 y 2 )


(–1)2 – a(–1) + 5 = 0 =
6 36
1+a+5=0 1
1
a=–6 =
216 �
(
�( x + 30 y ) x 2 � 30 xy + 900 y 2 �
� )
(b) Since x + 1 is a factor of x4 + 9x3 – 2x – 3a,
(–1)4 + 9(–1)3– 2(–1) – 3a = 0 ⇒ 1 – 9 + 2 – 3a = 0 5. Let f(x) = x3– 9x2 + 26x – 24
–6 – 3a = 0 We know that ± 1, ± 2, ± 3, ± 4, ± 6, ± 8, ± 12 and ± 24 are the
�6 factors of 24.
a= = a = –2
3 f(1) = 1 – 9 + 26 – 24 = 27 – 33 ≠ 0
(c) Since x + 1 is a factor of x2– ax – 5 f(–1) = (–1)3– 9(–1)2 + 26(–1) – 24
(–1)2– a (–1) – 5 = 0 ⇒ 1 + a – 5 = 0 ⇒ a=4 = –1 – 9 –26 – 24 ≠ 0
2 f(2) = 23– 9(2)2 + 26(2) – 24
(d) Since x + 1 is a factor of ax + 5x + ,
2
= 8 – 36 + 52 – 24 = 60 – 60 = 0
3
∴ (x – 2) is a factor of f(x).
2 13
a(–1)2 + 5(–1) + = 0 ⇒ a = Since f(x) is a third degree polynomial, there may be
3 3 another two linear factors for f(x).
10. (a) The remainder when f(x) = x3 + ax2 + 7x – a is divided
by x + a is f(–a). x–2 x3 – 9x2 + 26x – 24 x2 – 7x + 12
f(–a) = (–a)3 + a(–a)2 + 7(–a) –a = –a3 + a3– 8a = –8a x3 – 2x2
– +
(b) Let f(x) = 4x2 + 5x. –7x2 + 26x
If f(x) is divided by 5 + 4x, then the remainder is –7x2 + 14x
+ –
��5 �
f � ÷. 12x – 24
�4�
12x – 24
2 – +
��5 � ��5 � ��5 � 25 25 0
∴ f � ÷ = 4 � ÷ + 5� ÷ = 4 × � =0
�4� �4� �4� 16 4
∴ f(x) = (x – 2) (x2– 7x + 12)
Exercise 3 = (x – 2) (x2– 4x – 3x + 12)
= (x – 2) [ x( x � 4) � 3( x � 4)]
1. (a) 16x2 + 8xy + y2
[∵ (a + b)2 = a2 + 2ab + b2] f(x) = (x – 2) (x – 4) (x – 3)
= (4x)2 + 2.4x.y + y2 ∴ x3– 9x2 + 26x – 24 = (x – 2) (x – 4) (x – 3)
= (4x + y)2 6. (x3 + 5) (4 – x5) = 4x3 – x8 + 20 – 5x5
(b) 9x2 + 16y2 + 25z2 – 24xy + 40yz – 30zx Degree of polynomial is 8
= (3x)2 + (–4y)2 + (–5z)2 + 2(3x)(–4y) + 2(–4y) (–5z) + 3
2(–5z) (3x)
= (3x – 4y – 5z)2

(
7. � 1 + 2 �
1 � �
)
3

= �
(
1 + 2 2 � 1�

)
� � 1+ 2 �
1+ 2 �
[∵(a + b + c)2 = a2 + b2 + c2 + 2ab + 2bc + 2ca] �
� �
3
2. (a) (x – 2y + 3z)2 �1 + 2 + 2 2 � 1�
We have =� �
(x + y + z)2 = x2 + y2 + z2 + 2xy + 2yz + 2zx � 1+ 2 �
3
∴ (x – 2y + 3z)2
= x2 + (–2y)2 + (3z)2 + 2x (–2y) + 2(–2y) (3z)
3
�2 + 2 2 � � 1 + 2 �
= � � = �2
(

)
+2.(3z) (x) � 1+ 2 � � �
1+ 2 �

= x2 + 4y2 + 9z2– 4xy – 12yz + 6xz =2 =8
3
(b) (2a + b)3
= (2a)3 + (b)3 + 3(2a)2b + 3(2a)b2 8. Let p(x) = kx3 + 3x2 – 3
= 8a3 + b3 + 12a2b + 6ab2 q(x) = 2x3 – 5x + k
3. 8a3– b3– 12a2b + 6ab2 Given, p(4) = q(4)
= (2a)3– (b)3 + 3(2a)2(b) + 3(2a)(–b)2 k × 43 + 3 × 42 – 3 = 2 × 43 – 5 × 4 + k
= (2a – b)3 64k + 48 – 3 = 128 – 20 + k
4. (a) (98)3 = (100 – 2)3 64k + 45 = 108 + k
= 1003– 23– 3 × 100 × 2(100 – 2) 63
63k = 108 – 45 = 63 � k = =1
= 1000000 – 8 – 600 × 98 63
= 1000000 – 58800 – 8 9. p3 – q3 = (p – q) (p2 + pq + q2)
= 1000000 – 58808 = 941192 2
3 �10 � 5
1 3 �1 � p2 + q2 = (p – q)2 + 2pq = � ÷ + 2 �
(b) x + 125 y3 = � x÷ + ( 5 y )3 �9 � 3
216 �6 �
100 10 100 + 270
�1 �� 1 5 � = + =
= � x + 5 y÷� x 2 � xy + 25 y 2 ÷ 81 3 81
�6 ��36 6 �
6 Fundamental of Mathematics

370 ∴ f(x) = (x + 1) (x2 – x – 2) = (x + 1) [x2 – 2x + 1x – 2]


= = (x + 1) [x(x – 2) + 1(x – 2)] = (x + 1) (x – 2) (x + 1)
81
∴ x3 – 3x – 2 = (x + 1) (x + 1) (x – 2)
10 �370 5 � 10 505 5050
p3 – q3 = � + ÷= � = 13. (a) We have, (x + y + z)2 = x2 + y2 + z2 + 2xy + 2yz + 2zx
9 � 81 3 � 9 81 729
(p + q – 3r)2= p2 + q2 + (–3r)2 + 2.pq + 2q(–3r) + 2(–3r)p
Alternate method: = p2 + q2 + 9r2 + 2pq – 6qr – 6pr
p3 – q3 = (p – q)3 + 3pq (p – q) (b) (–4a + 5b – c)2
3 Identity: (x + y + z)2 = x2 + y2 + z2 + 2xy + 2yz + 2zx
�10 � �5 ��10 � 10 �100 � 5050
= � ÷ + 3 � ÷� ÷ = � + 5� = (–4a + 5b – c)2
�9 � �3 �� 9 � 9 � 81 � 729
= (–4a)2 + (5b)2 + (–c)2 + 2(–4a)(5b) + 2(5b)(–c)
10. x3 – x2 – 9x + 9 + 2(–c)(–4a)
Let f(x) = x3 – x2 – 9x + 9 = 16a2 + 25b2 + c2 – 40ab – 10bc + 8ac
f(1) = 13 – 12 – 9 (1) + 9 = 1 – 1 – 9 + 9 = 0 (c) 128m3– 250n3
∴ (x –1) is a factor of f(x). 2(64m3– 125n3)
= 2[(4m)3– (5n3)]
x–1 x3 – x2 + 9x – 9 x2 – 9
= 2(4m – 5n) (16m2 + 20mn + 25n2)
x3 – x2
– +
14. (a) 4x2– 16x + 12
9x + 9
–9x + 9 = 4(x2 – 4x + 3) = 4[x2– 3x – 1x + 3]
+ – = (4) [x(x – 3) –1(x – 3)]
0 = 4(x – 3) (x – 1)
The possible expressions of the dimensions of a cuboid
∴ f(x) = (x – 1) (x2– 9) = (x – 1) (x + 3) (x – 3)
are 4, (x – 3) and (x – 1).
∴ x3 – x2 – 9x + 9 = (x – 1) (x – 3) (x + 3)
(b) 4x3 – 16x
11. x3– 12x2 + 41x – 42 = 4x(x2– 4) = 4x(x – 2) (x + 2)
Let f(x) = x3– 12x2 + 41x – 42 The possible expressions of the dimensions of a cuboid
f(1) = 1 – 12 + 41 – 42 ≠ 0 are 4x, (x – 2) and (x + 2).
∴ f(–1) = (–1)3 – 12(–1)2 + 41(–1) –42 = – 1 – 12 – 41 – 42
≠0
HOTS
 f(2) = (2)3– 12(2)2 + 41(2) – 42 = 8 – 48 + 82 – 42 = 90 –
90 = 0
∴ (x – 2) is a factor of f(x). 1
1. x+ = 2 10
x–2 x3 – 12x2 + 41x – 42 x2 – 10x + 21
x
x3 – 2x2 2
– + � 1� 2
–10x2 + 41x – 42 ⇒
� x�
(
�x + ÷ = 2 10 = 4 �10 = 40)
–10x2 + 20x
+ –
21x – 42 1 1
21x – 42
⇒ x 2 + 2 + 2 � x � = 40
– + x x
0
1
∴ f(x) = (x – 2) (x2 – 10x + 21) = (x – 2) (x2 – 7x – 3x + 21)
⇒ x 2 + 2 + 2 = 40
x
= (x – 2) [x(x – 7) – 3(x – 7)] = (x – 2) (x – 3) (x – 7)
∴ x3 – 12x2 + 41x – 42 = (x – 2) (x – 3) (x – 7) 1
⇒ x 2 + 2 = 40 � 2 = 38
12. x3 – 3x – 2 x
Let f(x) = x3 – 3x – 2
2
f(1) = 1 – 3 – 2 ≠ 0 � 1� 2 1 1
f(–1) = (–1)3 – 3(–1) – 2 = – 1 + 3 – 2 = 3 – 3 = 0 Consider �x � ÷ = x + 2 � 2 � x �
∴ x + 1 is a factor. � x� x x
2 1
x+1 x3 – 3x – 3x – 2 x2 – x – 2 = x + 2 �2
x3 + x2 x
– –
= 38 – 2 = 36
–x2 – 3x
–x2 – x 1
+ + ∴ x � = 36 = ± 6
–2x – 2 x
–2x – 2
+ + 1 � 1 �� 2 1 1 �
0 ∴ x3 � 3
= �x � ÷�x + x � + 2 ÷
x � x �� x x �
Answer and Explanations 7

5. a = – 13, b = 37, c = –24


� 1 �
= 6 �x 2 + 2 + 1÷ Here, a + b + c = –13 + 37 – 24
� x � = –37 + 37 = 0 1
We know that of a + b + c = 0
= 6 (38 + 1) = 6 × 39 = 234
a3 + b3 + c3 = 3abc
1 ∴ a3 + b3 + c3 = 3 × (–13) × (37) × (–24)
2. x+ =4
x = 34632.

1 1 6. Given, x + y = 6, y + z = 11, x + z = 9 and xy + yz + zx = 50


⇒ x2 + 2
+ 2 � x � = 42 = 16 x + y + y + z + x + z = 6 + 11 +9 = 26
x x ⇒ 2x + 2y + 2z = 26
1 ⇒ 2(x + y + z)= 26
⇒ x 2 + 2 = 16 � 2 = 14 ⇒ x+ y+z =
26
= 13
x 2
3 ∴ (x + y + z)2 = x2 + y2 + z2 + 2(xy + yz + zx)
� 1� 3 ⇒ (13)2 = x2 + y2 + z2 + 2(50)
⇒ �x + ÷ = 4 = 64 ⇒ x2 + y2 + z2 = 132 – 100 = 169 – 100
� x �
∴ x2 + y2 + z2 = 69.
1 1� 1� 7. 27x3 – 64y3 + 125z3 + 180xyz
⇒ x3 + 3
+ 3 � x � �x + ÷ = 64
x x� x� (3x)3 + (–4y)3 + (5z)3 – 3 × 3x × –4y × 5z
This is an the form a3 + b3 + c3 – 3abc
1 ∴ 27x3 – 64y3 + 125z3 + 180xyz
⇒ x3 + 3 + 3(4) = 64
x = (3x – 4y + 5z) [(3x)2 + (–4y)2 + (5z)2 – (3x)(–4y) – (–4y)
(5z) – (3x)(5z)]
1 = (3x – 4y + 5z) (9x2 + 16y2 + 25z2 + 12xy + 20yz – 15xz)
⇒ x3 + 3 = 64 �12 = 52
x 8. (2x – y)3 + (y – z)3 + (z – 2x)3
Let A = 2x – y,
� 3 1 �� 2 1 � � 5 1 1 � B = y – z and C = z – 2x
�x + 3 ÷�x + 2 ÷ = �x + x + + 5 ÷ A + B + C = 2x – y + y – z + z – 2x = 0
� x �� x � � x x �
We know that, when A + B + C = 0, then A3 + B3 + C3 = 3abc
� 1 � ∴ (2x – y)3 + (y – z)3 + (z – 2x)3 = 3(2x – y) (y – z) (z – 2x)
⇒ (52)(14) = �x 5 + 5 + 4 ÷
� x � 9. x6 + x3 – y6 – y3
= x6 – y6 + x3 – y3
5 1 = (x3)2 – (y3)2 + x3 – y3
⇒ 728 = x + 5 + 14
x = (x3 – y3) (x3 + y3) + (x3 – y3)
= (x3 – y3) [x3 + y3 + 1]
1 = (x – y) (x2 + xy + y2) [(x + y) (x2 – xy + y2) + 1]
⇒ x5 + 5 = 728 �14 .
x 10. f(x) = x2 + px + q
1 When f(x) is divided by x + 1 the remainder is –8
∴ x5 + 5 = 714 . ∴ f(–1) = (–1)2 + p (–1) + q = –8
x ⇒ 1 – p + q = –8
3. x3 + x2 – 2 ⇒ q – p = – 8 – 1 = –9 ... (1)
= x3 – 1 + x2 – 1 When f(x) is divided by x – 4 the remainder is 32
= (x – 1) (x2 + x + 1) + (x – 1) (x + 1) ∴ f(4) = (4)2 + p × 4 + q = 32
= (x – 1) [x2 + x + 1 + x + 1) ⇒ 16 + 4p + q = 32
= (x – 1) (x2 + 2x + 2) ⇒ 4p + q = 32 – 16 = 16 ... (2)
4. a + b + c = 10, ab + bc + ca = 31, abc = 30 � p + q = �9
a + b + c = 10
+ 4 p + q = + 16
Squaring on both sides � � �
(a + b + c)2 = 102 = 100 �5 p = �25
a2 + b2 + c2 + 2(ab + bc + ca) = 100
⇒ a2 + b2 + c2 + 2 (31) = 100 25
⇒ a2 + b2 + c2 = 100 – 62 = 38 ⇒ p= =5
Now (a + b + c) [a2 + b2 + c2 – (ab + bc + ca)] = a3 + b3 + c3
5
– 3abc The value of ‘p’ in eq (1), we get
⇒ (10) (38 – 31) = a3 + b3 + c3 – 3 × 30 ⇒ – 5 + q = –9
⇒ (10) (7) = a3 + b3 + c3 – 90 ∴ q = – 9 + 5 = –4
∴ a3 + b3 + c3 = 90 + 70 = 160. Hence, the required value of p is 5 and q is –4.
8 Fundamental of Mathematics

∴ x3 – 2x2 – 5x + 6 = (x + 1) (x2 – 3x – 2) + 8
11. Since 3 and � 3 are zeros of the polynomial x4 – 3x3 – ∴ 8 should be subtracted from f(x) = x3 – 2x2 – 5x + 6 such
that (x + 1) divides f(x) exactly.
( )(
7x2 + 9x + 12, we can say that x + 3 , x � 3 are factors ) 14. f(x) = x4 – 7x3 + 6x2 + 28x – 36
of the given polynomial.
2
(
∴  x + 3 )( x � 3 ) = x2 � ( 3) = x 2 � 3 is a factor of
the given polynomial.

∴ x4 – 3x3 – 7x2 + 9x + 12 = (x2 – 3) (x2 – 3x – 4) ∴ f (x) = x4 – 7x3 + 6x2 + 28x – 36 = (x – 2) (x3 – 5x2 – 4x +
= (x2 – 3) (x2 – 4x + x – 4) 20) + 4
= (x2 – 3) [x (x – 4) + 1 (x – 4)] Hence, –4 should be added to f(x) = x4 – 7x3 + 6x2 + 28x –
= (x2 – 3) (x – 4) (x + 1) 36 such that f(x) is exactly divisible by x – 2.
15. f(x – 1) = x3 + x – 6
( )( )
= x + 3 x � 3 ( x � 4)( x + 1)
The remainder when f(x) is divided by x – 2 is f(2) (by
∴ The other zeros are 4, –1 remainder theorem)
Now putting x = 3 in f(x – 1) = x3 + x – 6
12. f(x) = x3 + ax2 + bx – 2 We get f(3 – 1) = 33 + 3 – 6 = 9 + 3 – 6 = 6
x – 1 is a factor ∴ f(1) = 0 (by factor theorem) f(2) = 6
⇒ (1) + a (1) + b (1) – 2 = 0
3 2 ∴ The remainder when f(x) is divided by x – 2 is 6.

⇒ 1+a+b–2=0 16. Given, (x) = 2x3 + 7x2 + 3x – 2 and g(x) = 2x2 + 5x – 3

⇒ a + b = 1 ... (1)
x + 2 is a factor ∴ f(–2) = 0 (by factor theorem)
⇒ (–2)3 + a (–2)2 + b (–2) – 2 = 0
⇒ –8 + 4a – 2b – 2 = 0
⇒ 4a – 2b = 10
⇒ 2(2a – b) = 10
⇒ 2a – b = 5 ... (2)
q(x) = (x + 1)
a +b =1
f(x) = q(x) g(x) + r(x)
2a � b = 5 = (2x2 + 5x – 3) (x + 1) + (x + 1)
3a = 6 = 2x3 + 2x2 + 5x2 +5x – 3x – 3 + x + 1
= 2x3 + 7x2 + 3x – 2 = f(x)
a = 2 ⇒ b = –1
17. x2 + x2y + y2x – y2
∴ The required values of a is 2 and b is –1. = x2 – y2 + x2y + y2x
13. f(x) = x – 2x – 5x + 6
3 2
= x2 – y2 + xy (x + y)
= (x + y) (x – y) + xy (x + y)
= (x + y)(x – y + xy)
= (x + y) (x – y + xy)
18. Let the zeros be α, 2α.
∴ zeros of x2 + px + 8 are α, 2α
�p
Sum of zeros = � + 2� = = �p
1
⇒ 3α = –p ⇒ p = –3α
Product of zeros = α × 2α = 8
⇒ 2α2 = 8
Answer and Explanations 9

= (a – 1) (a + 1 + b)
8
⇒ �2 = =4 = (a – 1) (a + b + 1).
2
⇒ α = ±2
20. a3 – 3a2b + 3ab2 – b3 1
= a3 – b3 – 3ab (a – b)
∴ p = 3 × 2 or p = 3 × (–2)
= (a – b) (a2 + ab + b2) – 3ab (a – b)
Hence, the required values of ‘p’ are 6 or –6.
= (a – b) (a2 + ab + b2 – 3ab)
19. a2 + ab – 1 – b = (a – b) (a2 – 2ab + b2)
= a2 – 1 + b (a – 1) = (a – b) (a – b)2 = (a – b) (a – b) (a – b).
= (a + 1) (a – 1) + b(a – 1)
Coordinate Geometry
Answer and Explanations 3
6. (a)  he perpendicular distance of the point A(3, –4) from the
T
STEPWISE MARKING SOLUTION
y–axis is 3 units.
7. (c) a < 0 and b < 0 ⇒ (a, b) lies in the III quadrant.
Objective Type Question
1. (b) 8. True
y B (3, 4) 9. False
C (0, 4)
10. y-axis

11. Zero
3
xc x 12. (a)
 ere, (–1, 3) x coordinate (–1) is negative and y –
H
O (0, 0) A (3, 0) coordinate is positive
\ (– +) lies in the second quadrant.
\ Both (–1, 3) and (–2, 5) lies in the same quadrant
Hence, both (A) and (R) are true and (R) is the correct
yc
explanation of (A)

2. (b)
y SUBJECTIVE TYPE QUESTION
T(–4, 4) Exercise 1
4
3 1. A = (1, –1) (1 Mark)
2 2. C = (4, 0) (1 Mark)
P(–1, 1)
1
3. B (1 Mark)
xc x 4. –3 [∵ B = (–3, 2)] (1 Mark)
–4 –3 –2 –1 1 2 3 4
–1 R(1, –1) 5. –5 [∵ D = (0, –5)] (1 Mark)
–2 6. R = (1, 3) (1 Mark)
–3 7. Q = (0, 4) (1 Mark)
S(–2, –3)
–4
Q(3, –4) 8. 0 [∵C = (4, 0)] (1 Mark)
yc 9. R = (1, 3)
\ The distance between the point R and y-axis is 2 unit.
Q and R lies in fourth quadrant.
 (1 Mark)
3. (b) Abscissa of P = x coordinate of P = -2 10. P = (–3, –3)
Abscissa of Q = x coordinate of Q = -3 ∵ The distance between the point P and x-axis is 3 units.
Required difference = (–2) - (–3) = 1  (1 Mark)
4. (d) Exercise 2
y
1. The point (2, 3) lies in I quadrant. (1 Mark)
5 S(0, 5)
2. The point (4, 0) lies on the positive x-axis. (1 Mark)
4R(0, 4)
3 3. The point (–5, –7) lies in III quadrant. (1 Mark)
2
4. The point (0, 1) lies on the positive y-axis. (1 Mark)
1 P(5, 1)
Q(8, 0) 5. The point (3, –2) lies in IV quadrant. (1 Mark)
xc x
(0, 0) 1 2 3 4 5 6 7 8 9 Exercise 3

1. If (x, y) lies on the positive x-axis, then x > 0 and y = 0.


 (2 Marks)
yc For example, the point (3, 0) lies on the positive x-axis.
 (1 Mark)
O and Q lies on the x-axis
2. If (x, y) lies on the negative x-axis, then x < 0, and y = 0.
5. (a) The required distance is 5 units.  (2 Marks)
2 Fundamental of Mathematics

For example, the point (–2, 0) lies on the negative x-axis. Y


 (1 Mark)
5
3. If (x, y) lies on the positive y-axis, then x = 0 and y > 0. 4
 (2 Marks) 3 P (0, 3)
For example, the point (0, 4) lies on the positive y-axis. 2 T (1, 2)
 (1 Mark)
S (–5, 0) 1 Q (1, 0)
4. If (x, y) lies on the negative y-axis, then x = 0 and y < 0. Xc X
 (2 Marks) –5 –4 –3 –2 –1–1 1 2 3 4 5
R (0, –1)
For example; (0, –7) lies on negative y-axis. (1 Mark) –2
5. Positions of the points are shown by dots in the figure below. –3
–4
y –5

7 Yc
6 13. (c) 
Point lies on the y-axis means x-coordinate is 0. Also its
(4, 5)
5 y-coordinate is negative.
4 Hence, the required point is (0, –5)
Option (c) is correct.
3
(–6, 2) (3, 2) 14. (a) 
Perpendicular distance of the point p(3,4) from the
2
y – axis is 3.
1
Option (a) is correct.
xc x
–6 –5 –4 –3 –2 –1 1 2 3 4 5 6 7 15. (c) 
By joining the given points we get a rectangle of length 5
–1
units and breadth 3 units.
–2
Y
–3
(7, –4) 5
–4 (0, 3) (5, 3)
–5
3 3
–6
X
–7 0 (0, 0) 5(5, 0) X

yc

PROBLEMS FOR ADDITIONAL PRACTICE 16. (c) The distance between the points is 9 units.
Objective Type Question: 17. (d) The required distance is 6 units
1. (b) Abscissa of a point is positive in I and IV quadrant. 18. True
2. (d) In second and fourth quadrant, the points whose abscissa
and ordinate have different signs. 19. False

3. (b) Co-ordinate of P are (–2, 4) 20. False


4. (c)Co-ordinate (–5, 3) identified by the point L. 21. False
Because x-coordinate is negative and y coordinate is
positive. 22. Positive
5. (c) Points lie in I and III quadrants 23. Origin
6. (b) The mirror image of (–5, 7) on the y–axis is (5, 7).
24. Negative
7. (d) (–6, 8) lies in the II quadrant.
25. y-axis
8. (b) The required point is (–7, 0).
26. (b) In point (–8, 0) : x coordinate (–8) is negative and
9. (a) Ordinate of all the points on the x–axis is 0.
y – coordinate is zero.
10. (b) (–5, 10) lies in the II quadrant. \ (–8, 0) lies in the negative direction of the x–axis
11. (b) Ordinate of the point = 4 Hence, both (A) and (R) are true and (R) is not the correct
Point lies on y-axis means abscissa is zero. explanation of (A)
Hence, required point is (0, 4)
27. (d) In point ( 0, 3) x-coordinate is zero and y-coordinate is 3.
12. (c) P, R and T do not lie on the x-axis.
\ (0, 3) lies on the y-axis
Hence, (A) is false but (R) is true
Answer and Explanations 3

16. M(–6,–7) = Q3, x < 0 and y < 0.


SUBJECTIVE TYPE QUESTION
17. N(–2, 2) = Q2, x < 0 and y > 0.
Exercise 1
18. P(4, 4) = Q1, x > 0 and y > 0.
Solutions for questions 1 to 5: 19. Q(2, –6) = Q4, x > 0 and y < 0.
y
20. R(8,–20) = Q4, x > 0 and y < 0.

Exercise 2
7
E(–5, 5) 6 Solutions for questions 1 to 5:
5
1. (a) A = (2, 3)
4
3
A(2, 3) \ The ordinate is 3. 3
B(–2, 1) 2 (b) B = (4, –3)
1 \ The distance between the point B and x-axis is 3 units
xc x 2. (a) B = (4, –3)\ The abscissa is 4.
–7 –6 –5 –4 –3 –2 –1 1 2 3 4 5 6 7 8
–1 C(4, 0) (b) C = (–2, –2)
–2 \ The distance between the point C and y-axis = 2 units.
–3
D(–3, –3) 3. (a) C = (–2, –2)
–4
–5 \ The ordinate is –2.
–6 (b) D = (–3, 5)
–7 \ The distance between the point D and x-axis is 5 units
–8
4. (a) D= (–3, 5)
(b) E = (–2, 4)
yc
5. (a) G = (4, 5)
\ The abscissa = 4
Solutions for questions 6 to 10 (b) F = (2, –1)
y \ The distance between the point F and y-axis is 2 units.
8 Solutions for questions 6 to 10:
7
6 6. two perpendicular
G(0, 5)
P(–3, 4) 5 7. four, quadrants
4
3 8. x, abscissa
2 9. y, ordinate
1
xc x 10. (0, 0)
–8 –7 –6 –5 –4 –3 –2 –1 1 2 3 4 5 6 7 8
–1 Exercise 3
–2
–3 1. The name of the horizontal line is x-axis and the vertical line is
H(–4, –3) –4 Q(2, –3) y-axis.
–5 2.
–6 y
F(6, –7)
–7
–8
4
yc 3
2
Solutions for questions 11 to 20: 1
xc x
11. A(3,0) lies on the positive x-axis, since ordinate is zero and –6 –5 –4 –3 –2 –1 1 2 3 4 5 6
abscissa is positive. –1
–2
12. B(–2, 3) = Q2, x < 0 and y > 0. N (–2, –2) –3 M (6, –2)
13. C(–5, 0) lies on the negative x-axis, since ordinate is zero and
–4
abscissa is negative.
14. D(0, –3) lies on the negative y-axis, since abscissa is zero and yc
ordinate is negative.
15. E(–3, –3) ∈ Q3 x < 0 and y < 0. length of MN = 8
4 Fundamental of Mathematics

3.
HOTS
y
1. (x, –y) Є Q1
⇒ x > 0, –y < 0 –x < 0 and –y < 0
4
(–3, 3) (3, 3) ∴ (–x, –y) Є Q3,
3
2 2.
Y
1
xc x
–7 –6 –5 –4 –3 –2 –1 1 2 3 4 5 6
–1
–2
–3 Xc X
(–3, –3) (3, –3) 4
–4
P
yc 3 (4, –3)
The figure formed by joining the points is a square of side 6
uints. Yc
\ Area of the square = 62 = 36 sq.units.  he point is located in the 4th quadrant and its co-ordinates
T
are (4, –3).
4.
y 3.

4 C (0, x)
3
2 A B
1 (– 2, 0) 0 (– 2, 0)
B(3, 0)
xc x
–7 –6 –5 –4 –3 –2 –1 1 2 3 4 5 6
–1
–2
–3
–4 Given, ABC form an equilteral triangle.
–5 ∴ AB = AC = BC = 8
C(0, –5) A(3, –5)
In ∆AOC, ∠AOC = 90°
yc
=OA 2unitsAC
= 8units , OC = x units
5. (i) (4, 3) (ii) (–4,3) (iii) (–4, –3) (iv) (4, –3) By Pythagoras theorem,
6. (a) G = (4, 5) AC2 = OC2 + OA2
(=
8) ( 2)
2 2
(b) D = (–3, 5) ⇒ + x2
\ The distance between the point D and y-axis is 3 units.
⇒ x2 = 8 – 2 = 6
7. (a) F = (2, –1)
(b) E = (–2, 4) ⇒ x= ± 6
\ The distance between the point E and y-axis is 2 units. ∴ x = 6 or − 6 .
8. (a) B = (4, –3) 4.
\ The ordinate is –3.
(b) C = (–2, –2)
\ The ordinate is –2. B (0, b)

9. (a) A = (2, 3)
\ The distance between the point A and x-axis is 3 units.
(b) G = (4, 5) 0 A (4, 0)
\ The distance between the point G and y-axis is 4 units.
10. (a) A = (2, 3)
\ The abscissa is 2.
(b) B = (4, –3)
\ The distance between B and y-axis is 4 units
1
Given, area of ∆OAB = × OA × OB =
6 sq units
2
Answer and Explanations 5

⇒ OA × OB = 12
⇒ y co-ordinate of ‘G’ is 3
⇒ 4 × b = 12

b
⇒ =
12
= 3
∴ (
Centroid ‘G’ = 3, 3 . )
4
7.
∴ b = 3.
D (2, 3) C (x, y)
5.
Y

C B (x, y) A (2, 1) B (4, 1)


(0, 3)

Xc X
3
0 A (5, 0)

From the figure, AB = CD = AD = BC = 2 units


BC Parallel y-axis
Yc
∴ Abscissa of C is ‘4’
OA = BC = 5 units CD Parallel x-axis
Perpendicular distance to the point B from the ‘y’ axis ∴ Ordinate of C is 3
5 units ∴ x = 4, y = 3.
∴ Abscissa of B is 5 8.
OC = AB = 3 units Y
Perpendicular distance to the point ‘B’ from the ‘x’ axis is
3 units
B
∴ Ordinate of B is 3
6 6
∴ x = 5, y = 3.
6.
Xc X
0 P A (4, 0)
(0, 0)
Y

B (3, 3 3 )
Yc

Xc X In the figure, BP is the altitude onto the x-axis.


0 P A (6, 0)
∵ OAB is an isosceles triangle, the altitude BP bisects the
base OA.
We know OA = 4 units
⇒ OP = PA = 2 units
Yc ∴ Co-ordinates of P(2, 0)
In ∆OBP, ∠OPB = 90°, OB = 6 units and OP = 2 units.
In the figure, BP is the altitude drawn from B to the ‘x’ ∴ By Pythagoras theorem,
axis.
∴ Co-ordinates of P are (3, 0). BP = OB2 − OP 2 = 62 − 22
∵ ∆OAB is an equilateral triangle, the altitude BP is also the = 36 − 4 =32
median
= 4 2 units .
∴ Length of BP = 3 3 units
∴ Perpendicular distance to B from x-axis = 4 2 units
Let ‘G’ be the centroid of the ∆OAB
∴ x co-ordinate of ‘G’ is 3 (∵ G is on BP.) ∴ Ordinate of B is 4 2
The centroid divides the median in the ratio 2:1 from the ∵ BP Parallel to y-axis, the ‘x’ co-ordinate of all the points
vertex on BP is the same
∴ Perpendicular distance of G from the x-axis ⇒ Abscissa of B = 2

=
1
3
×3 3 =3 units ∴ (
Co-ordinates of 2, 4 2 . )
6 Fundamental of Mathematics

9. Given, A (1, 4), B(1, 1), C(5, 1) OA = AB = BC = OB, ∠OAB = 90°


Since the y co-ordinates of ‘B’ and ‘C’ are same, we can say ∵ BA Parallel y-axis, the ‘x’ co-ordinate of all the points on
BC Parallel x-axis. BA will be the same.
⇒ x co-ordinate of A = 3.
Y ∴ Co-ordinates of A are (3, 0)
A (1, 4) ⇒ OA = 3 units
OC = 3 units (∵OA = AB = OC = BC)
∴ Co-ordinates of ‘C’ are (0, 3).
Hence, the required area of OABC = 32 = 9 sq units.
B (1, 1) C (5, 1) 12.
Xc X
Y

D (0 , 5)

Yc
Since the ‘x’ co-ordinates of ‘A’ and ‘B’ are the same, we can Xc X
say that AB Parallel y-axis A 0 (0, 0) C
Extend CB to D, CD// x-axis and transversl to y-axis. (–4, 0) (4, 0)
∴ ∠YDB = ∠DOX (∵Corresponding angles)
But ∠DOX = 90°
⇒ ∠YDB B (0, –5)
Now AB // YD. CD is the transversal.
∠YDB = ∠ABC (Corresponding angles) Yc
∴ ∠ABC = 90° (∵ ∠YDB = 90°)
∴ ∆ABC is a right angled triangle. Area of rhombus = Area of ∆ADC + Area of ∆ABC
1
10. Area of ∆ADC = × AC × DO
Y 2
D (2, 6) C (6, 6) AC = 4 + 4 = 8 units
DO = 5 – 0 = 5 units
1 40
= × 8 × 5= = 20sq units
2 2
A (2, 3) B (6, 3) 1
Xc X Area of ∆ABC = × AC × BO
2
AC = 4 + 4 = 8 units
OB = 5 units
1
= ×8×5 =20sq units
2
Yc ∴ Area of ABCD = 20 + 20 = 40 sq units.
13. Given, AD = 3 units
From the figure,
AB = 6 – 2 = 4 units DA Parallel y-axis
BC = 6 – 3 = 3 units ∴ x co-ordinates of all the points on DA is the same
∴ ar(ABCD) = 4 × 3 = 12 sq units. ⇒ x co-ordinate of A = 3.
AD is the altitude and it is 3 units of length.
11. The Perpendicular distance of A from x-axis is 3 units
Y
Y

C B (3, 3) B (3, 3 3 )

Xc X
0 A
Xc X
B D C
(2, 0) (3, 0)  


Yc

In the figure, OABC is a square Yc


Answer and Explanations 7

⇒ y co-ordinate of A = 3 Area of pentagon ABCD = Area of rectangle OABD + Area


∴ Co-ordinates of A are (3, 3) of ∆BCD
1 From the figure, OA = 4 units and AB = 2 units
∴ Area of ∆ABC = × BC × AD
2 ∴ Area of rectangle OABD = 4 × 2 = 8 sq units
1 1
= × 5.5 × 3 Area of ∆BCD = × BD × CE
2 2
= 2.75 × 3 BD = OA = 4 units.
= 8.25 sq units. CE = 2 units (from figure)
14. 1
∴ Area of ∆BCD = ×4×2 =4sq units
C (2, 4)
2
Hence, the required area of pentagon
2 B (4, 2) 3
D 4 OABCD = 8 + 4 = 12 sq units.\
(0, 2) E
2
0 (0, 0) A (4, 0)
Linear Equations in Two Variables
Answer and Explanations 4
Comparing it with ax + by + c = 0, we get a = 0, b = 3, c =
STEPWISE MARKING SOLUTION –4 (1 Mark)
9 7
Objective Type Question 5. y+6 =x
5 2
1. (c) The given linear equation 2x + 3y = 6 meet the x-axis
7 9
\ y co-ordinate is zero. ⇒ x− y−6 =0
∴ 2x + 3(0) = 6 2 5
⇒ 2x = 6 ⇒ x = 3 7 −9
Hence, the co-ordinate on x-axis is (3, 0) Comparing it with ax + by + c = 0, we get a = ,b=
2 5 4
2. (c) The given linear equation y = x has same value , c = –6 (1 Mark)
\ Co-ordinate of x = 1
6. Given, 3x = y
Co-ordinate of y = 1
\ Required point is (1, 1) which must lie on the line y = x.
x 0 1 2 3
3. (b) When we multiply or divide both sides of a linear equation y = 3x 0 3 6 9
with a non-zero number then solution of the linear equation
remains same. \ Four solutions of 3x = y are (0, 0), (1, 3), (2, 6), (3, 9).
 (2 Marks)
4. (a) (8, 9) lies on ax – 5y + 13 = 0
⇒ 8a – 9(5) + 13 = 0 x − 18
7. Given, x – 2y = 18 ⇒ y=
8a = 45 – 13 2
8a = 32
a=4 x 0 2 4 6
y –9 –8 –7 –6
5. (c) Going by options,
Given point (3, –8) \ Four solutions of x – 2y = 18 are (0, –9), (2, –8), (4, –7),
(a) ⇒ 3x – 6y = 48 given (6, –6) (2 Marks)
3(3) – 6 (–8) ≠ 48 20 − 4 x
(b) ⇒ 9x – 2y = 10 given 8. Given, 4x + 5y = 20, y=
5
9(3) – 2 (–8) ≠ 10
(c) ⇒ 7x – 3y = 45 given x 0 5 10 15
7(3) – 3 (–8) = 45
y 4 0 –4 –8
6. (d) A point can be common for infinite number of lines.
\  our solutions of 4x + 5y = 20 are (0, 4), (5, 0), (10, –4),
F
7. (c) The equation of a line parallel to the x–axis is y = k. (15, –8). (2 Marks)
8. True 9. Given, 2x – y = 10 ⇒ y = 2x – 10

9. Origin
x 0 1 2 3
10. Straight line y –10 –8 –6 –4
\ Four solutions of
Subjective Type Question 2x – y = 10 are (0, –10), (1, –8),
(2, –6),(3, –4). (2 Marks)
Exercise 1
10. Given, 5x – 5y =20
1. 5x – 9y = 8⇒ 5x – 9y – 8 = 0
⇒ x–y=4⇒y=x–4
Comparing it with ax + by + c = 0, we get a = 5, b = –9, c = –8
 (1 Mark)
x 0 1 2 3
4x 9 4x 9
2. + =3y ⇒ − 3y + =0 y –4 –3 –2 –1
3 2 3 2
4 \ Four solutions of 5x – 5y = 20 are (0, –4), (1, –3), (2, –2),
Comparing it with ax + by + c = 0, we get a = ,b = –3, (3, –1). (2 Marks)
3
9 Exercise 2
c=  (1 Mark)
2 1. Let the age of Ram be x.
3. 4y + 8 = 9x ⇒ 9x – 4y – 8 = 0 And the age of Rahim be y.
Comparing it with ax + by + c = 0, we get a = 9, b = –4, c = –8 The age of Ram is twice the age of Rahim. (1 Mark)
 (1 Mark) \ x = 2y (1 Mark)
4. 3y = 4
⇒ x - 2y = 0 (1 Mark)
⇒ (0)x + 3y – 4 = 0
2 Fundamental of Mathematics

2. Given, 3x – 5y = k is passing through (2, –5). 3. Let the marbles with the two friends be x and y respectively.
\ x = 2, y = -5 (1 Mark) ⇒ x + y = 50
Substitute x = 2 and y = -5 in
3x - 5y = k (1 Mark) x 0 20 30 35 25
\ 3(2) – 5(–5) = k y 50 30 20 15 25
⇒ k = 31 (1 Mark)
3. Given point is (–3, 4). Y'
Infinitely many lines pass through a point. 70
4x + 5y = 4(–3) + 5(4)
60
i.e. 4x + 5y = 8 (2 Marks) (0,50)
4y – 3x = 4(4) –3(–3) 50 x + y = 50
i.e. 4y – 3x = 25 A
\ 4x + 5y = 8 and 4y – 3x = 25 are two lines pass through 40 (20, 30)
(–3, 4) (2 Marks) B
30 (25, 25)
4. Given, 3x + 4y = 12 C
(a) (0, 4) 20 (30, 20) D
3(0) + 4(4) ≠ 12 (1 Mark) (35, 15)
10
(b) (0, 3)
3(0) + 4(3) = 12 (1 Mark) X' X
0 10 20 30 40 50 60
(c) (3, 0)
3(3) + 4(0)≠ 12 (1 Mark)
(d) (4, 0)
3(4) + 4(0) = 12 Y
\ (0, 3), (4, 0) lie on 3x + 4y = 12 (1 Mark)
Since (20, 30) and (30, 20) are on the line x + y = 50, the other
Exercise 3 have 30 marbles. (2 Marks)
4. Let the marbles with the two friends be x and y respectively.
1. (3, 2) and (–3, –2) are the points on the line.
⇒ x + y = 50
(a) 3x + 2y = 7 but 3(3) + 2(2) ≠ 7 (1 Mark)
(b) 4x – 7y = 0 but 4(3) – 7(2) ≠ 0 (1 Mark)
x 20 35 25
(c) y = 4x + 3 but 3 ≠ 4(2) + 3  (1 Mark)
(d) 2x = 3y, 2(3) = 3(2) and 2(–3) y 30 15 25
= 3(– 2) (1 Mark)
\ The given graph of the line represented by 2x = 3y Y'
 (1 Mark) 70

2. (a) In one variable, y = –5: (1½ Marks) 60


(0,50)
y = –5 50 x + y = 50
–5 –4 –3 –2 –1 0 1 2 3 4 5 6 40 (20, 30) A
(c) In two variables, (0)x + y = -5: (2½ Marks) B
30 (25, 25) C
y (30, 20)
20 D
(35, 15)
5 10
4 X' X
3 0 10 20 30 40 50 60
2
1
xc x Y
–5 –4 –3 –2 –1 1 2 3 4 5
–1 
The point (25, 25) represents that, both of them have equal
number of marbles. (2 Marks)
–2
–3 5. Let the marbles with the two friends be x and y respectively.
–4 ⇒ x + y = 50
y = –5
–5
x 0 10 20 30 35 25
y 50 40 30 20 15 25
yc
Answer and Explanations 3

Y' y
70 2. x+ 7
=
2
60 1
(0,50) ⇒ x+  y−7 =0
50 x + y = 50 2
Comparing it with ax + by + c = 0, we get
40 (20, 30) A 1
a = 1, b = , c = -7
B 2
30 (25, 25) C
(30, 20) 3. 3x – 7 = 4y ⇒ 3x – 4y -7 = 0
20 D Comparing it with ax + by + c = 0, we get
(35, 15)
10 a = 3, b = –4, c = –7
X' X 4. 5x = 6 ⇒ 5x + 0y – 6 = 0 4
0 10 20 30 40 50 60 Comparing it with ax + by + c = 0, we get
a = 5, b = 0, c = – 6
8
Y 5. 7y − 4 =x
3
A(20, 30), C(30, 20) shown in the graph, which represent the 8
difference in Marbles with them to be 10. (2 Marks) ⇒ x −7y + 4 =0
3
Comparing it with ax + by + c = 0, we get
PROBLEMS FOR ADDITIONAL PRACTICE 8
a= b = –7, c = 4
3
Objective Type Question
6. 3x + 4y – 3 = 0
1. (c) Let linear equation be ax + by + c =0 Comparing with ax + by + c = 0, we get
Put x = 1 and y = 2 a = 3, b = 4 and c = – 3
∴ a (1) + b ( 2 ) + c =0 ⇒ a + 2b + c =0 2 2
7. x + 4y =
7 ⇒ x + 4y − 7 =0
Where a, b and c are real numbers. 3 3
Hence, infinitely many linear equation in x and y can be Comparing with ax + by + c = 0, we get
satisfied. 2
a= , b = 4 and c = – 7
2. (c) The given point (a, a) has same values i.e. co-ordinate of 3
x = co-ordinate of y
y y
\ Point (a, a) must lies on the line y = x 8. 2x − = 1.3 ⇒ 2 x − − 1.3 =
0
5 5
3. (d) By going through options point (a, –a) always lies on the
line x + y = 0 i.e. a + (–a) = 0 Comparing with ax + by + c = 0, we get
−1
4. (a) The line parallel to x = 6 i.e. x – 6 = 0 and c = −1.3
a = 2, b =
5
5. (a) Equation of a line parallel to y = 3 is y = k i.e. y = 0 2 2
9. x− =0 ⇒ 1x + 0 y − = 0
6. (b) 3x – y = 4x ⇒ x + y = 0 3 3
7. (d) 3x – 2y = 9 given Comparing with ax + by + c = 0, we get
Going by options, (5, 3) → 3 × 5 – 2 × 3 = 9 −2
a = 1, b = 0 and c =
8. False 3
9. True 10. 5x = 3y ⇒ 5x – 3y = 0 ⇒ 5x – 3y + 0 = 0
Comparing with ax + by + c = 0, we get
10. Origin a = 5, b = –3 and c = 0
11. y-axis x
11. Given, x = 2y ⇒ y=
12. x-axis. 2
x 0 4 6 8
Subjective Type Question
x
Exercise 1 y= 0 2 3 4
2
1. Given, 2x + 3y = 5
⇒ 2x + 3y – 5 = 0 \ Four solutions of x = 2y are (0, 0), (4, 2), (6, 3), (8, 4)
Comparing it with ax + by + c = 0, we get
a = 2, b = 3, c = –5
12. x + 3y = 24
24 − x
⇒ y=
3
4 Fundamental of Mathematics

x 0 3 6 9 Four solutions of 2x + 2y = 10 are (1, 4), (2, 3),


\ 
(3, 2), (4, 1)
24 − x 16. It is given 2x + y = 5 ⇒ y = 5 – 2x
y= 8 7 6 5
3 x 1 2 0 –1 –2
\ Four solutions of x + 3y = 24 are (0, 8),(3, 7), (6, 6), (9, 5) y 3 1 5 7 9
36 − 3 x (x , y) (1 , 3) (2, 1) (0, 5) (–1, 7) (–2, 9)
13. 3x + 4y = 36 ⇒ y=
4 The coordinate points are (1, 3), (2, 1), (0, 5) and (–1, 7),
x 0 4 8 12 (–2, 9).

y 9 6 3 0 17. 2x + py = 10; py = 10 – 2x
10 2
\  our solutions of 3x + 4y = 36 are (0, 9), (4, 6), (8, 3),
F y
= − x
(12, 0) π π
x−6 10 2 10
14. x – 2y = 6 ⇒ y= x=0⇒ y= − ×0 = x=1
2 π π π
10 2 8
x 0 2 4 6 ⇒ y= − =
π π π
y –3 –2 –1 0
10 2 10
x=p⇒ = y − ×π
= −2
\ Four solutions of x – 2y = 6 are (0, – 3), (2, – 2), (4, – 1), π π π
(6, 0).
10 2 14
x = –2 ⇒ y = − (−2) =
15. Given, 2x + 2y = 10 π π π
⇒ x+y=5⇒y=5–x
10 2 4
x 1 2 3 4 x = 3 ⇒ y = − (3) =
π π π
y 4 3 2 1

x 0 1 p –2 3
y 10 8 10 14 4
−2
π π π π π
(x , y)
 10   8 10  14   4
 0,  1,  (p, −2)  −2,   3, 
 π  π π  π  π

\ Five infinite solutions of 2x + py = 10 19. x = 5


 10   8  10 Let us express this in the form of ax + by + c = 0
are  0,  ,  1,  , (p, − 2 ), x + 0.y – 5 = 0
 π   π  π
i.e. for any value of y, x value will be 5 only.
 14   4
 −2,  and  3,  . x 5 5 5 5 5
 π  π
18. y = 4x y 0 1 2 3 4
Let x = 0 ⇒ y = 4 × 0 = 0 (x, y) (5, 0) (5, 1) (5, 2) (5, 3) (5, 4)
x=1⇒y=4×1=4
\ Five infinite solutions of x = 5 are (5, 0), (5, 1), (5, 2),
x=2⇒y=4×2=8 (5, 3) and (5, 4).
x = –1 ⇒ y = 4(–1) = –4
x = –2 ⇒ y = 4(–2) = –8 20. y = 6 ⇒ y = (0) x + 6

x 0 1 2 –1 –2 1 2 3 4 5
x
y 0 4 8 –4 –8 6 6 6 6 6
y
(x , y) (0, 0) (1, 4) (2, 8) (–1,–4) (–2,–8)
(x, y) (1, 6) (2, 6) (3, 6) (4, 6) (5, 6)
\  ive infinite solutions of y = 4x are (0, 0), (1, 4), (2, 8),
F \ Five infinite solutions of y = 6 are (1, 6), (2, 6),(3, 6),
(–1, –4), (–2, –8). (4, 6) and (5, 6).
Answer and Explanations 5

Exercise 2 ⇒ k = 10
4. Given, 4x + 7y = 26
1.
(a) (2, 3)
Y 4(2) + 7(3) ≠ 26
5
(b) (3. 4)
4 4(3) + 7(4) ≠ 26
3 (c) (3, 2)
2 x – 2y – 5 = 0 4(3) + 7(2) = 26
1 (7, 1) (d) (4, 5)
(5, 0)
X´ –4 4(4) + 7(5) ≠ 26.
–3 –2 –1 0 1 2 3 4 5 6 7 8 X
\ (3, 2) lies on 4x + 7y = 26
–1 (3, –1)
5. Given point (8, – 5)
4
–2
Infinitely many lines pass through a point.
–3 2x + 3y = 2(8) + 3 (–5)
–4 i.e., 2x + 3y = 1
Y´ 3x – 2y = 3(8) – 2(–5) = 34
i.e. 3x – 2y = 34
Given, x – 2y – 5 = 0 \ 2x + 3y = 1 and 3x – 2y = 34
⇒ x = 5 + 2y
are two lines passes through (8, – 5)
If y = 0 ⇒ x = 5
If y = 1 ⇒ x = 7 6.
If y = –1 ⇒ x = 3
Y
4
x 5 7 3
3
y 0 1 –1
2
(x, y) (5, 0) (7, 1) (3, –1) (0, 0)
(–1, 1) 1
2.
Xc X
7 –3 –2 –1 0 1 2 3 4 5
6 –1 (1, –1)
5 (2, –2)
–2
4 (0, 3)
3 –3
P
2
–4
1 (3, 1)
X Yc
–3 –2 –1 0 1 2 3 4 5 6 7 8
–1 (6, –1)
–2 x 0 1 –1 2
–3 y 0 –1 1 –2
–4 (x, y) (0, 0) (1, –1) (–1, 1) (2, – 2)
Y

Given, 2x + 3y = 9 7.
−2 9 Y
⇒ =
3y = –2x + 9 ⇒ y x+ 4
3 3 (2, 2)
3 y = 4x – 6
−2
\=y x+3 2
3 1
Let x = 0 ⇒ y = 3 X' X
–3 –2 –1 0 1 2 3 4 5
Let x = 3 ⇒ y = –2 + 3 = 1
–1 (1, –2)
Let x = 6 ⇒ y = – 4 + 3 = – 1 –2
x 0 3 6 –3
–4
y 3 1 –1
–5 (0,–6)
(x, y) (0, 3) (3, 1) (6, – 1)
–6
–7
3. Since (2, 4) is a point on 7x – y = k
14 – 4 = k Y'
6 Fundamental of Mathematics

Given, y = 4x – 6 7. When C = 37
If x = 0 ⇒ y = – 6 9
x=1⇒y=4–6=–2 F = × 37 + 32 =98.6
5
x=2⇒y=8–6=2
8. Let the ‘t’ is the temperature at which both the scales will have
x 0 1 2 same numeral values.
y –6 –2 2 9
t
= t + 32
(x, y) (0, – 6) (1, – 2) (2, 2) 5
4
\ –32 = t
8. The given equation is 2x + 2 = 3y ⇒ 2x – 3y = –2 5
(i) When x = 0 and y = 0
5
L.H.S. = 2(0) – 3(0) = 0 ≠ R.H.S. –32 × = t
\ (0, 0) is not a solution of 2x – 3y = –2 4
(ii) (2, 3) \ t = –40
L.H.S. 2(2) – 3(3) = 4 – 9 = –5 ≠ R.H.S. \ at –40 both centigrade and Fahrenheit scales shows the
\ (2, 3) is not a solution of 2x – 3y = –2 same number value.
(iii) (2, 2)
L.H.S. = 2(2) –3(2) = 4 – 6 = –2 = R.H.S. HOTS
\ (2, 2) is a solution 2x – 3y = –2
1. 2x + 3y = 24
(iv) (5, 4)
L.H.S = 2(5) – 3(4) = 10 – 12 = –2 = R.H.S. 3y = 24 – 2x
\ (5, 4) is a solution of 2x – 3y = –2 24 − 2 x 24 2 x 2x
y= = − = 8−
3 3 3 3
9. Since (2, 3) is a solution of 2x – y + k = 0
2(2) – 3 + k = 0 2x
For ‘y’ to be positive integer must be an integer and less
4 – 3 + k = 0 ⇒ k = –1 3
10. x + y = 7 and 2x + y = 9 are two different lines which passes than 8
through (2, 5). 2x
For to be an integer ‘x’ should be a multiple of 3 i e 3, 6,
Infinite number of lines can pass through a single point. 3
\ There exist infinitely many lines which passes through
9, 12 ……..
(2, 5).
2x
⇒ <8
Exercise 3 3
1. When r = 2 cm, p = 2p(2) cm 24
⇒ x<
When r = 3 cm, p = 2p(3) cm 2
When r = 4 cm, p = 2p(4) cm ⇒ x < 12
\ p = 2pr ∴ x can take values 3, 6, 9.
2. Given, r = 16 cm ∴ The positive integer solutions are
P = 2pr for x = 3, y = 6 ⇒ (3, 6)
= 2p (16 cm) = 32 p cm x = 6, y = 4 ⇒ (6, 4)
3. Given, p = 100p cm x = 9, y = 2 ⇒ (9, 2)
⇒ 2pr = 100p cm
⇒ r = 50 cm 2.
4. Number of apples = 5 56 x + 64 y = 232 — (1)
The cost of each apple = ` x +64 x + 56 y = 248 — (2)
Number of oranges = 4
The cost of each orange = ` y
120 x + 120 y = 480
Total cost = ` 100 120( x + y ) =480
\ 5x + 4y = 100
480
5. 5x + 4y = 100 ⇒ x + y= = 4 ...(3)
120
x = 16
\ 5(16) + 4y = 100 ⇒ 4y = 100 – 80 ⇒ 4y = 20 ⇒ y = 5 (1) − (2) 56 x + 64 y =232
+ 64 x + 56 y =+ 248
6. (– 3, 1) and (1, – 3) are the points on the line. − − −
(a) 2x + y = 5 but 2 (–3) + 1 ≠ 5 − 8x + 8 y =
−16
(b) 3x – 2y = 0 but 3 (–3) – 2(1) ≠ 0
(c) x + y = – 2; – 3 + 1 = – 2, 1 – 3 = – 2
8( y − x) =−16
\ The given graph of the line is represented by −16
x+y=–2 y−x= =−2 ...(4)
8
(d) 4x – y = 7 but 4 (– 3) – 1 ≠ 7
Answer and Explanations 7

x+ y = 4 20 2
⇒ A
= =
− x + y =−2 30 3
2y = 2 1 2 3
∴ A= = ⇒ x=
y=1 x 3 2
⇒ x=3 1 1
∴ B= = 2 ⇒ y =
40 21 y 2
3. + 8
=
x y 5 10
5. + 7
=
24 7 x+ y x− y
− 2
=
x y 4 15
− 1
=
1 1 x+ y x− y 4
Let ,
= A= B
x y 1 1
= A,
Let = B
40A + 21B = 8 ... (1) x+ y x− y
24A – 7B = 2 ... (2) 5A + 10B = 7 ...(1)
Adding eq(1) and 3 × eq(2), we get 4A – 15B = 1 ...(2)
Adding 3 × eq(1) and 2 × eq(2), we get
40A + 21B = 8 15A + 30B = 21
72A − 21B = 6 8A − 30B =2
112A = 14 23A = 23
14 1
A
= = 23
112 8 A
= = 1
23
Substituting the value of ‘A’ in eq(1), we get
Substituting the value of ‘A’ in 15A + 30B = 21, we get
1 15 × 1 + 30B = 21
⇒ 40 × + 21B =8
8 ⇒ 30B = 21 – 15 = 6
⇒ 21B = 8 – 5 = 3 6 1
⇒ B
= =
3 1 30 5
B =
⇒ =
21 7 1
=A = 1
1 1 x+ y
A =
∴ = ⇒ x = 8 ⇒ x = 64
x 8 1 −1
1 1 =B =
B =
∴ = ⇒ y = 7 ⇒ y = 49. x− y 5
y 7 1
x+ y =
15 7 5
x− y =
4. + =24
x y 2x = 6
10 2 8 ∴ x = 3, y = –2
− =
x y 3 6. 2x + 5y = 3xy ... (1)
1 1 10x + 19y = 12xy...(2)
= A,
Let = B Substract eq(2) from 5 × eq(1), we get
x y
10 x + 25 y =15 xy
15A + 7B = 24 ... (1)
8 + 10 x + 19 y =
+ 12 xy
− − −
10A − 2B =  ... (2)
3 6 y = 3 xy
Substract 3 × eq(2) from 2 × eq(1), we get 6 = 3x
30A + 14B =48 ⇒ x=2
+ 30A − 6B =
+8 Substituting the value of ‘x’ in eq(1), we get
− + −
2 × 2 + 5y = 3 × 2 × y
20B = 40 ⇒ 4 + 5y = 6y
40 ⇒ y=4
B= ∴ x = 2, y = 4
20
∴ B=2 28 3
7. + 5
=
Substituting the value of ‘B’ in 30A + 14B = 48, we get x+ y x− y
30A + 14 × 2 = 48
12 1
⇒ 30A = 48 – 28 = 20 − 1
=
x+ y x− y
8 Fundamental of Mathematics

1 1 ⇒ 24 + x + y + z = 33
Let= P,
= Q ∴ x + y + z = 33 – 24 = 9
x+ y x− y
10. Let ‘A’ have ‘x’ number of apples and ‘B’ have ‘y’ number of
28P + 3Q = 5 ...(1) apples.
12P – Q = 1 ...(2) According to the problem,
x + 9 = 8 (y – 9)
Adding eq(1) and 3 × eq(2), we get
⇒ x + 9 = 8y – 72
28P + 3Q =5 ⇒ x – 8y = –81 … (1)
36P − 3Q =3 y + 6 = 2(x – 6)
⇒ y + 6 = 2x – 12
64P =8 ⇒ 2x – y = 18 ... (2)
x – 8y = –81
8 1 2x – y = 18 × 8
P
= =
64 8 Substract 8 × eq(2) from eq(1), we get
Substituting the value of ‘P’ in eq(1), we get x – 8 y = –81
1 + 16 x − 8 y =+ 144
12 × − Q = 1 − + −

8 −15 x −225
=
12 4 1
⇒ Q= −1 = = 225
8 8 2 =x = 15
15
1 1 Substitute the value of ‘x’ in eq(2), we get
=P =
x+ y 8 2 × 15 – y = 18
⇒ 30 – y = 18
⇒ 8
x+ y = ⇒ y = 30 – 18 = 12
∴ x = 15, y = 12
1 1
=Q = 11. Let the breadth of the rectangle be ‘b’
x− y 2 then, the ℓ = 2b + 2
2
x− y = Given, Perimeter = 2(ℓ + b) = 34
⇒ 34
2 x = 10 ⇒  + b= = 17 cm
2
10
x = 5
= ⇒ 2b + 2 + b = 17
2 ⇒ 3b + 2 = 17
⇒ x = 25 ⇒ 3b = 17 – 2 = 15
Substituting the vlue of ‘x’ in x + y , we get 15
∴ b
= = 5cm
25 + y =
8 3
∴ ℓ = 2b + 2 = 2 × 5 + 2 = 12 cm
⇒ 5+ y =
8 Hence, the required length of the rectangle is 12 cm.
⇒ y =8−5 = 3 12. Let the present age of father and son be ‘x’ years and ‘y’ years
⇒ y=9 respectively
∴ x = 25, y = 9 x = 3y – 3
x – 3y = –3 ... (1)
8. Let the cost of 1 pen, 1 pencil and 1 eraser be ₹x, ₹y and ₹z
Father’s age six years ago = x – 6 years
respectively.
Son’s age six years ago = y – 6
5x + 4 y + 6 z = 44
According to the problem,
4 x + 5 y + 3z =37 x – 6 = 4(y – 6)
∴ x – 6 = 4y – 24
9x + 9 y + 9z = 81
x – 4y = –18 ... (2)
9( x + y + z ) =81
x − 3y = −3
81 + x − 4 y =−18
⇒ x+ y+z= = 9 − + +
9
y = 15
Hence, the cost incurred on purchasing 1 pen, 1 pencil and
1 eraser = ₹9. Put the value of ‘x’ in eq(1), we get
9. Given, 5x + 7y + 11z = 57 and 2x + 3y + 5z = 24 x – 45 = –3
5x + 7y + 11z = 57 ⇒ x = –3 + 45 = 42
⇒ (2x + 3y + 5z) + 3x + 4y + 6z = 57 ∴ The present age of the father is 42 years and the son is 15 years.
⇒ 24 + (3x + 4y + 6z) = 57 (∵ 2x + 3y + 5z = 24) 13. Let the father’s age be ‘x’ years and the daughters age be ‘y’
∴ 3x + 4y + 6z = 57 – 24 = 33 years
⇒ (2x + 3y + 5z) + x + y + z = 33 ⇒ x – y = 29
Answer and Explanations 9

Given, the son is two years elder then the daughter 240
⇒ Son’s age = (y + 2) years =x = 40
Given, difference between the father’s age and the son’s 6
age = 27 years Substitute the value of ‘x’ in eq(2), we get
⇒ x – (y + 2) = x – y – 2 1 1 1
⇒ 29 – 2 = 27 years ⇒ − =
40 y 120
∴ The difference between the ages of father and son, 10
years from now is 27 years. 1 1 1
⇒ − =
14. Let the sides of the square be S1 and S2. 40 120 y
S1 4 3 −1 1
Given, = ⇒ =
S2 5 120 y
Given, sum of the perimeters of the squares = 72 cm

2
=
1 4
= 4S1 + 4S2 = 4(S1 + S2) = 72
120 y
72
⇒ S1 + S2 = = 18 ⇒ 2y = 120
4 ∴ y = 60
⇒ S1 + S2 = 18 ∴ x = 40 km/hr, y = 60 km/hr.
4
But, S1 = S2 16. Let the cost of 1 text book be ₹‘x’
5 and the cost of 1 note book be ₹‘y’
4 According to the problem,
⇒ S2 + S 2 =18
5 2x + y = 255 ... (1)
4S2 + 5S2 x + 3y = 240 ...(2)
⇒ = 18 Substract 2 × eq(2) from eq(1), we get
5 2x + y = 255
⇒ 9S2 = 90
+ 2x + 6 y =
+ 480
90 − − −
∴ S= = 10 cm
2
9 − 5y =−225
Put the value of ‘S2’ in S1 + S2 = 18, we get 225
=y = 45
⇒ S1 = 18 – 10 = 8 cm 5
Hence, the required lengths of the sides of the square is 8 ∴ Cost of 1 note book = ₹45.
cm and 10 cm respectively.
2 1
17. =x y+6
Distance 3 4
15. Time taken to cover a distance =
Speed 2 x y + 24
⇒ =
240 240 3 4
∴ + 10
=
x y ⇒ 8x = 3y + 72
⇒ 8x – 3y = 72 ... (1)
1 1 
⇒ 240  +  =10 3x
− 1 =y
x y 4
1 1 10 1 ⇒ 3x – 4 = 4y
⇒ + = =  ... (1)
x y 240 24 ⇒ 3x – 4y = 4 ... (2)
8 x − 3 y = 72 × 4
240 240
Given, − 2
= 3 x − 4 y =4 × 3
x y
32 x − 12 y = 288
1 1 
⇒ 240  −  = 2 + 9 x −12 y = + 12
x y − + −

1 1 2 1 23 x = 276
⇒ − = =  ... (2) 276
x y 240 120 =x = 12
1 1 1 23
+ = Substitute the value of ‘x’ in eq(2), we get
x y 24
⇒ 3 × 12 – 4y = 4
1 1 1 ⇒ 36 – 4 = 4y
− =
x y 120 ⇒ 4y = 32
2 1 1 5 +1 32
= + = y
⇒ = = 8
x 24 120 120 4
2 6 ∴ x = 12, y = 8.
= ⇒ 6 x = 240
x 120
10 Fundamental of Mathematics

18. Let the other two sides of the triangle be ‘x’ and ‘y’. ⇒ x – y = –21 ... (2)
Substracting eq(2) from eq(1), we get
2 x − y =−2
x y + x − y =− 21
− + +

x = 19
x+y–2
Put the value of ‘x’ in eq(1), we get
∴ Base of the triangle = x + y – 2 ⇒ 2(19) – y = – 2
Given, the perimeter of the triangle is 24 units ⇒ 38 + 2 = y
⇒ Perimeter = x + y + x + y – 2 ∴ y = 40
⇒ 24 = 2x + 2y – 2 Hence, the number of students is 19 and the number of
⇒ 2(x + y – 1) = 24 chocolates is 40.
24
⇒ x + y − 1= = 12 20. The adjacent sides AB and BC differ by 5 units.
2
⇒ x + y = 12 + 1 = 13 ... (1) A B
Given, x – y = 1 ... (2)
13
x+ y =
x− y =1
D C
2 x = 14
Let AB be x ⇒ CD = x
14 Let BC be y ⇒ AD = y
x
= = 7
2 Given, x – y = 5
Put the value of ‘x’ in eq(2), we get Given, the perimeter of the parallelogram = 30 units
⇒ 7–y=1⇒y=6 ⇒ x + y + x + y = 30
∴ Sides of the triangle are 7 cm, 6 cm, (7 + 6 – 2) cm ⇒ 2(x + y) = 30
= 7 cm, 6 cm, 11 cm. ⇒ x + y = 15
5
x− y =
19. Let the number of students be ‘x’
and the number of chocolates be ‘y’ 15
x+ y =
Given, 2x + 2 = y 2x = 5
⇒ 2x – y = –2 ... (1) ∴ x = 10 units
Also given, x + 21 = y
Introduction To Euclid’s Geometry
Answer and Explanations 5
Let the two mid points be R and Q.
STEPWISE MARKING SOLUTION R is the mid point of AB.
⇒ AR = BR (1 Mark)
Objective Type Question: AR + AR = BR + AR (equals are added to equals)
1. (b) Three 2 AR = AB
AB
2. (c) Straight line AR = ... (1) (1 Mark)
2
3. (c) A postulate Similarly Q is a mid point of AB
4. (d) Any polygon AB
⇒ AQ = ... (2) (1 Mark)
5. (a) Triangles 2
 rom (1) and (2) AR = AQ and R,Q are on the same line
F
6. (b) Second axiom AB, implies R and Q coincide.
7. (a) Surfaces and circles ∴ Every line has one and only mid point. (1 Mark) 5
8. (c) Nine 5. We can imagine a ‘plane’ to mean some surface and ‘lines’ to
mean certain curves drawn on the surface. Consider a sphere
9. (b) Deductive Reasoning and the ‘equator’ PQ. Consider two ‘meridians’ MAN and
10. True MBN these are two ‘lines’ perpendicular to the ‘line’ PABQ.
But they are not parallel. The meet at the ‘poles M and N.
11. AC
M
12. 1

Subjective Type Question: P Q


1. (a)  alse, Infinite number of lines can be drawn through a
F A B
single point. (1 Mark)
(b) False, Given two distinct points, there is a unique line
which passes through them. (1 Mark) N
(c) True, It is the 2nd postulate of Euclid. (1 Mark)
(d) True, if the radii are equal, then the circles coincide.
∴ The circles are equal. (1 Mark)
Line segment: A line segment is a part of a line with two
2. (a) 
end points. The other terms that need to be defined in this
definition are
 (4 Marks)
(i) part (ii) line (2 Marks)
(b)  Radius of a circle: The distance from the centre of a
circle to any point on the circle is called its radius. The PROBLEMS FOR ADDITIONAL PRACTICE
other terms that need to be defined are:
(i) circle (ii) distance
Objective Type Questions
(iii) centre (2 Marks)
(c)  Square: The closed figure formed by joining the points 1. (d) DC + DB = AB
of intersection of the circle and two of its perpendicular 2. (c) Three
diameters is a square. The terms that need to be defined
are 3. (c) Two
(i) diameter
4. (a) Public worship
(ii) closed figure. (2 Marks)
5. (c) Greece
3. XY = YZ (given)
XY + XY = YZ + XY 6. (c) Greece
7. (a) Jhales
X Y Z
8. (a) Theorem
(If equals are added to equals, then the wholes are equal).
 (1 Mark) 9. (c) A postulate
2XY = XZ
10. (b) a proof
1
∴ XY = XZ (1 Mark) 11. False
2
4. Let R is the mid point of AB ⇒ AR = BR 12. True
Let us assume that there are two mid points of AB. 13. point
2 Fundamental of Mathematics

14. parallel 7. Yes. Euclids fifth postulate implies the existence of parallel
lines. Let a straight line ‘ℓ’ falls on two straight lines ‘m’ and
15. two ‘n’ such that the sum of interior angles on one side of ‘ℓ’ is two
16. curve right angles. Then, according to Euclid’s fifth postulate they do
not meet on this side of ‘ℓ’.
Since the sum of the interior angles on the other side also is
Subjective Type Question two right angles, the lines ‘m’ and ‘n’ do not intersect on other
1. A solid has shape, size, position, and can be moved from one side either.
place to another. Therefore the lines m and n do not intersect on either side.
Surfaces: Boundaries of a solid are called surfaces. They ∴ They are parallel to each other.
separate one part of space from another, and are said to have 8. PR = QS given
no thickness. The boundaries of surfaces are curves or straight
PQ + QR = PR
lines.
QR + RS = QS
2. (a) Point: A point is that which has no part. ⇒ PQ + QR = QR + RS
(b) Line: A line is breadthless length. ⇒ PQ = RS.
(c) Straight line: A straight line is a line which lies evenly
with the points on itself. S
Q R
(d) Surface: A surface is that which has length and breadth P
only.
(e) Plane surface: A plane surface is a surface which lies
evenly with the straight lines on itself. 9. It can be visualized by anyone for anything in any part of the
world, so this is a universal truth.
3. Starting with his definitions, Euclid assumed certain properties
which were not to be proved. These assumptions are actually 10. Suppoose line ‘p’ falls on two given lines ‘m’ and ‘n’ such that
‘obvious universal truths’. He divided them into two types: the sum of the interior angles on the same side of ‘p’ is 1800.
axioms and postulates. Then the two lines do not intersect, no matter to what extent
they are produced.
He used the term postulate for the assumptions that were
specific to geometry. On the other hand common notions or 11. (i)  alse, Infinite number of lines can be drawn through a
F
assumptions, used through out mathematics and not specifically single point.
linked to geometry are axioms. (ii) True, There exist a set of infinite points in between any
4. (a) True, Reason: Equals are added to equals then the wholes two points.
are equal. (iii) True, AB çç CD and CD çç PQ .
(b) True, Reason: a = b, b = c ⇒ a = c ∴ AB çç PQ
(c) True, Reason: If equals are added to equals then the (“is parallel to” is transitive)
wholes are equal. (iv) False, centres can be different with equal radii.
(d) False, Reason: If equals are subtracted from equals then (v) False, Since only one point exists which is at equidistance
the results are equal. from three distinct points. Only one circle can be drawn
5. In the given figure, PR coincides with PQ + QR. According through three non collinear points.
to Euclids axiom, things which coincide with one another, 12. (i) I f the distance between two lines remains constant then
are equal to one another and a unique line passes through two the two lines are said to be parallel lines.
given points. Q lies on the line joining P and R. (ii) A part of a line is known as line segment.
∴ PQ + QR = PR (iii) A ray starts from a point continuous to infinity.
(iv) A circle is a set of points which are at constant distance
P Q R from a fixed point.

6. Let DE be a given line segment. According to Euclids postulate, 13.  = BD


AC 
a circle can be drawn with any centre and any radius.
Taking ‘D’ as the centre and DE as radius, draw a circle. Taking 
AC  + BC
= AB 
‘E’ as the centre and ED as radius draw another circle. Let the

BD  + CD
= BC 
two circles meet at a point, say F. Draw the line segments DF
and EF.  + BC
 = BC
 + CD

Now DF = EF (Radius of the same circle) ⇒ AB
EF = DE (Radius of the same circle) ⇒  = CD
AB 
By Euclids axiom things which are equal to the same thing are
equal to one another. D
∴ DF = EF = DE. So, DEF is an equilateral triangle. C
O
F

A B

14. Let O and O’ be two points which are common to both the
lines, where AB and CD are two different straight lines.
D E ∴ Only one line can pass through OO’ AB and CD must lie
on the same straight line which is a contradiction.
Answer and Explanations 3

∴ Only one line can pass through two distinct points. Consider AD = BC.
Subtract CD on both sides.
A D ⇒ AD – CD = BC – CD
According to the Euclid’s axiom, things which are
subtracted from equals are equal to one another.
O ⇒ AC = BD.
BD = 6 cm
C B
⇒ AC = cm and AD = 10 cm.
15. Euclid and many other mathematicians thought that the fifth Now AC + CD coincides with AD
postulate can be proved by using just the four postulates According to the Euclid’s axiom, things which coincide
and the axioms. All the attempts to prove 5th postulate with one another are equal to one another.
failed. But these efforts have led to the creation of non ∴ AC + CD = AD
Euclidean geometry. Till then every one believed that 6 + CD = 10
Euclids was the only geometry. Mathematicians are under CD = 10 – 6 = 4 cm
the impression that world itself is Euclidean, but now the ⇒ CD2 = 42 = 16 cm.
geometry of the universe we live in, has been shown to be
a non Euclidean geometry. That is why 5th postulate is most 5. Given, ‘C’ is a point between ‘A’ and ‘B’ such that AC = 3x – 1,
significant. BC = 2x + 1 and AB = 4x + 3. 5
C
HOTS A B
1. 2 litres of water are added to two cans, each having 5 litres of
milk. Now AC + BC coincides with AB
The resulting quantity in one of the cans is 2ℓ + 5ℓ = 7ℓ.= A According to the Euclid’s axiom, things which coincide with
one another are equal to one another.
The resulting quantity in the other can is = 2ℓ + 5ℓ = 7ℓ.= B.
∴ AC + BC = AB.
It can be observed that, A = B.
∴ 3x – 1 + 2x + 1 = 4x + 3
The Euclid’s axiom supporting the answer is, if equals are
added to equals then the wholes are equal. 5x = 4x + 3
5x – 4x = 3
2. Let ABCD be a square of side 5cm. If another square EFGH is ⇒ x = 3.
super imposed on ABCD such that their boundaries coincide,
then it means that the squares ABCD and EFGH are equal. 6. Consider a line ‘ℓ’ and mark a point P not on the line ‘ℓ’. Draw
This means the areas of the squares ABCD and EFGH are equal. a line ‘m’ passing through the point P. Draw another line ‘n’
Area of EFGH = Area of ABCD = 25 cm2. such that it falls on the lines ‘ℓ’ and ‘m’.
∴ Side of square EFGH = 5 cm. n
The Euclid’s axiom which supports the answer is, things
which coincide with one another are equal to one another.
P
(E)A 5 cm B(F) m
‘1

l
‘2

D(H) C(G) The line ‘n’ makes interior angles ∠1 and ∠2 with the lines ‘m’
and ‘ℓ’ on the right side of the line.
3. Given, BC = 5 cm and AB = 14 cm. The sum of the angles can be less than 180°, more than 180°
or equal to 180°.
C According to the Euclid’s Fifth Postulate, if a straight line
A B intersects two other lines and the sum of the interior angles
on one side is less than 180°, then the two lines, if produced
It can be seen from the above figure that, AC + BC coincides sufficiently, intersect on that side on which the sum of the
with AB. interior angles is less than 180°.
According to the Euclid’s axiom, things which coincide with Also, if the sum of the interior angles is more than 180°, the
one another are equal to one another. lines do not intersect on that side when extended sufficiently.
∴ AC + BC = AB Both the above cases do not give rise to equidistant lines.
AC + 5 = 14 ∴ The lines will be equidistant only if the sum of the interior
⇒ AC = 14 – 5 = 9 cm angles is equal to 180°.
⇒ AC2 = 92 = 81 cm. This is possible if only a unique line passes through the
point P such that ∠1 + ∠2 = 180°.
4. Given, AD = 10cm, BD = 6cm and AD = BC. Thus, if there is a line ‘ℓ’ and a point ‘P’ not on ‘ℓ’, a
C D unique line passing through point ‘P’ exists such that the
two lines are equidistant from one another.
A B Hence Proved.
4 Fundamental of Mathematics

7. ‘A’ and ‘B’ are points on a line and ‘C’ is another point.  ccording to the Euclid’s axiom, things which coincide with
A
 et us assume ‘C’ lies between ‘A’ and ‘B’ such that AC = 6cm,
L one another are equal to one another.
BC = 11 cm, AB = 16 cm. ∴ AC + BC = AB
AC + BC = 6 + 11 = 17cm
C
But, AB = 16 cm
A B ∴ AC + BC ≠ AB.
From the figure, AC + BC coincides with AB. ∴ ‘C’ does not lie between ‘A’ and ‘B’.
Lines and Angles
Answer and Explanations 6
6. (a) 80° – 2x = 3x – 10° (Vertically opposite angles)
STEPWISE MARKING SOLUTION
⇒ 5x = 90° ⇒ x = 18°

Objective Type Question 7. False

1. (a) 8. 60°

R S 9. 60°

4x Subjective Type Question


40° 3x Exercise 1
P O Q
1. Let the angle be x°.
Given: POQ is a line segment Its supplement = (180° – x)
∠POR + ∠ROS + ∠QOS = 180° Its complement = (90° – x)
⇒ 40° +4x° + 3x = 180° 180° – x = 4(90° – x) (1 Mark)
⇒ 7x =180° – 40° 4x – x = 360° – 180°
⇒ 7x =140° ⇒ 3x = 180°
140 ⇒ x = 60° 6
⇒ x
= = 20° \ The required angle = 60°. (1 Mark)
7
⇒ x =20° 2. Let the angles be 7x and 11x.
2. (a)
Given, the angles are complementary.
⇒ 7x + 11x = 90° ⇒ x = 5° (1 Mark)
R S \ The bigger angle = 11x = 55°
The supplement of the bigger angle
O P 130° = 180° – 55° = 125° (1 Mark)
T
3. Draw a line n through P and parallel to ℓ.⇒ n || ℓ || m.
110°
A
x ℓ
Q
a
Extend line OP, to intersect RQ at T n
P b
Given OP || RS
And RT is transversal line y
m
⸫ ∠RTP = ∠TRS {Alternative interior angles are equal} B
RQ is a line segment
∠PTQ + ∠RTP = 180° ℓ || m, AP is transversal.
∠PTQ = 180° – ∠RTP = 180° – 130° = 50° From the figure, x = a (alternate interior angles)
⇒ ∠PTQ = 50° Similarly, m || n, BP is transversal. (1 Mark)
In DPQT Then, y = b (alternate interior angles)
∠OPQ = ∠PTQ + ∠PQT x+y=a+b
Sum of two opposite interior angle are equal to exterior But, given a + b = z
angle \ x + y = z (1 Mark)
⸫ 110° = 50° + ∠PQT
4. y = 90° – 30° = 60° (1 Mark)
⇒ ∠PQT = 110° – 50° = 60°
1 1
⇒ ∠PQT = 60° x= y= (60°) = 10°
6 6
3. (d) ∠OPQ = 70° (vertically opposite angles)
\ z = x + y = 10° + 60° = 70° (1 Mark)
∠ABO = 70° (corresponding angles)
∠AOB = 180° – (70° + 40°) = 70° 5. Let the angle be x.
x + 70° = 180° (straight angle) \ Its supplement = 180° – x
⇒ x = 110° Given, x = 3(180° – x) (1 Mark)
x = 540° – 3x
4. (a) 3x + 2x + 80° = 180° (straight angle)
x = 135°. (1 Mark)
⇒ 5x = 100° ⇒ x = 20°
5. (c) 180° – (2x – 40°) = 6x + 20° Exercise 2
(∵ Corresponding angles or alternate interior angles) 1. Given thatAC // DB , ∠GEB = 50° and ∠GBE = 30°
⇒ – 2x + 220° = 6x + 20° ⇒ 8x° = 200°
In DEGB, ∠EGB + ∠GBE + ∠BEG = 180°
⇒ x = 25°
∠EGB = 180° – ∠GEB – ∠GBE
2 Fundamental of Mathematics

= 180° – (50° + 30°) = 100° (11/2 Marks) (corresponding angles)


∠EFC + ∠EGB = 180° (AC // DB and FG is a transversal) y = b = 70° (11/2 Marks)
⇒ ∠EFC = 180°– ∠EGB a = c = 110° and b = 70°.
= 180° – 100° = 80° (11/2 Marks)
Exercise – 3
2.
1. Given that ∠PAC = 30°.
P
A

A O B

30°
Q

Given that ∠BOP = ∠BOQ  ...(1) x


y
and AOB is a straight line.
B P C D
∠AOP + ∠BOP = 180° (1 Mark)
∠AOP = 180 – ∠BOP  ... (2) AP^BC and the bisector of ∠BAC.
Similarly, ∠AOQ + ∠BOQ = 180° In D APC, ∠ACD = ∠APC + ∠PAC
∠AOQ = 180° – ∠BOQ ... (3) (an exterior angle of a triangle)
From (1), (2) and (3), we get ∠AOP = 90° + 30° = 120° (2 Marks)
= ∠AOQ. (Since ∠BOP = ∠BOQ) (2 Marks) Since AP is the bisector of ∠BAC,
3. Given that ∠B = 65o ∠BAP = ∠CAP
P \ ∠BAP = 30°. (1 Mark)
In DABP,∠ABP + ∠BAP = ∠APC
A (exterior angle of a triangle)
30° + y = 90° ⇒ y = 60°
\ x = 120° and y = 60°. (1 Mark)

C 2. Given that AP || BC and ∠ACB = 60°.


B
Q Q

We have∠BAP = ∠B + ∠BCA  … (1)


A P
∠BCQ = ∠B + ∠BAC  … (2)
(11/2 Marks)
On adding (1) and (2), we get
∠BAP + ∠BCQ
= ∠B + ∠BAC + ∠BCA + ∠B
= ∠B + 180° R B C
(∵ ∠BAC + ∠BCA + ∠B = 180o)
AP || BC and AC is a transversal
= 65° + 180° = 245° (11/2 Marks)
\ ∠CAP = ∠ACB(\ alternate angles)
4. a = 180° – 100° (∵ a and 100° forms linear pair) = 80° \ ∠CAP = 60°
b = 100° (11/2 Marks) and ∠CAP = ∠QAP
(∵ b and 100° are alternate interior angles) (∵ AP is the bisector of ∠QAC)
\ b – a = 100° – 80° = 20° (11/2 Marks) \ ∠QAP = 60° (1 Mark)
5.
∠CAQ = ∠CAP + ∠QAP = 60° + 60° = 120°
∠CAQ + ∠BAC = 180° (∵ linear pair)
A B 120° + ∠BAC = 180° (1 Mark)
x y ∠BAC = 60°
C D
a b ∠ABR = ∠BAC + ∠ACB
E F (exterior angle of the triangle)
c = 60° + 60° = 120° (2 Marks)
ℓ m 3. Given that AB || CD.
∠BEG + ∠EGD = 180°
Given that AB || CD || EF. ℓ, m are transversals of AB, CD and
EF, (Interior angles on the same side of the transversal, AB || CD )
and also given x = 110° and y = 70°. \ 2∠FEG + 2∠FGE = 180° (1 Mark)
x = c = a (corresponding angles) (EF and FG bisect ∠BEG and ∠DGE respectively).
AB || CD || EF  (11/2 Marks) ∠FEG + ∠FGE = 90°  ... (1)
∠FEG = ∠FGE + 10°  ... (2)
c = 110°and a = 110°
 (1 Mark)
CD || EF ⇒ y = b
On solving (1) and (2),
Answer and Explanations 3

we get, 2. (c) ∠QRP = 65° (alternate interior angles)


2∠FGE = 80°. y = 40° (alternate interior angles)
⇒ ∠FGE = 40° x + 40° + 65° = 180° (Angles in DPQR)
From (1) ∠FEG + ∠FGE = 90° ⇒ x = 75°
⇒ ∠FEG + 40° = 90°
3. (a) ∠ABC = 180° – 150° = 30°
⇒ ∠FEG = 90° – 40° = 50° (2 Marks)
∠ACB = 180° – 105° = 75°
4. Given that DAE || BC, ∠BAD = 2x – 70°, ∠ACB = x – 20° and In DABC, x + 75° + 30° = 180° ⇒ x = 75°
∠FAG = 2x + 20°
∠DAB = ∠ABC 4. (d) x + 125° = 180° ⇒ x = 55°
y = 125° (corresponding angles)
(∵ Alternate interior angles DE || BC .)
y – x = 125° – 55° = 70°
\ ∠ABC = 2x – 70°
And ∠EAC = ∠ACB(1Marks) 5. (c) Draw OP parallel to ℓ and m.
(∵ Alternate interior angles DE || AC ) A

\ ∠EAC = x – 20° 40°
∠FAG = ∠BAC
O
(∵ Vertically opposite angles) P
\ ∠BAC = 2x + 20° (1 Mark) 45°
In DABC, ∠ABC + ∠BAC + ∠ACB = 180° m
B
(2x – 70°) + (2x + 20°) + (x – 20°) = 180°
5x° – 70° = 180° ∠AOP = 40° (alternate interior angles) 6
x° = 50° ⇒ ∠ACB = (x – 20)° = 30° ∠BOP = 45° (alternate interior angles)
⇒ ∠EAC = 30° (∵ ∠EAC = ∠ACB) (2 Marks) ∠AOB = ∠AOP + ∠BOP = 40° + 45° = 85°
5. Given ∠PSR = 120° 6. True
P 7. False

8. One
2x
x 9. 68°

10. Exterior angle


120° 11. Line
y + 40 60° y
Q R 12. Interior
S

Let ∠SPR = x Subjective Type Question


\ ∠QPS = 2x
Exercise 1
Let ∠R = y
\ ∠Q = y + 40° (1 Mark) 1. Given PQ || RS and ∠GHS = 30°
In DPSR, x + y + 120° = 180° ∠DHS = ∠DGQ = 30° (corresponding angles)
⇒ x + y = 180° – 120° ∠DGQ + ∠DGP = 180°
⇒ x + y = 60° ∠PGD = 180 – 30 = 150°
  
⇒ y = 60° – x .... (1) (1 Mark) 2. AB || CD and EF is a transversal of AB and CD
In ∆PQS, 2x + y + 40° = 120° 3x - 20 + x + 40 = 180o
2x + 60° – x + 40° = 120 (from (1)) 4x = 160o ⇒ x° = 40°
x = 20° and y = 60° – x = 60° – 20°= 40°
3. Let the angle be x.
\ ∠QPR = 3x = 3x (20°) = 60°. (2 Marks) The supplement of x is 180° – x
⇒ 180° – x = 100° ⇒ x = 80°
PROBLEMS FOR ADDITIONAL PRACTICE \ The complement of x is 90° – x = 90° – 80° = 10°
4. Let each of equal angle be x.
Objective Type Question x + x = 180o ⇒ x = 90o
1. (b) Let three angle A, B and C be 2x, 4x and 3x respectively 5. Let the angles be 2x and 3x.
Sum of all the three angles of a triangle is 180° 2x + 3x = 90° ⇒ x = 18°
⸫ ∠A + ∠B + ∠C = 180° The larger angle = (3x) = 54°.
⸫ 2x° + 4x + 3x = 80°
6. Let x, y, z are 3 acute angles.
⇒ 9x = 180°
x + y + z = 180°
180°
⇒ x
= = 20° y - x = 10
9 ⇒ y = x + 10o
Measure of smallest angle 2x = 2 × 20 = 40° z - y = 10o
4 Fundamental of Mathematics

⇒ z = y + 10o x = 48°, y = 82° and z = 50°


= x + 10 + 10o = x + 20o ∵ ∠AOC = ∠BOD [vertically opposite angles]
\ x + x + 10 + x + 20 = 180° Now, ∠AOE + ∠AOC + ∠COF = 180°
⇒ 3x + 30 = 180° ⇒ x = 50° The sum of angles at a point on a straight line on the same
side = 180
7. Given that BC || AD, AB = AD and ∠DFE = 140°
EF is also a straight line
A F D Now ∠AOF = 180 – x° = 180° – 48° = 132°
∠BOF = x° = 48o
140°
∠AOD = x + z = 48° + 50° = 98°
E
2. Given ∠AOD = ∠BOD = 90°.
B C D

∠AFE = 180° – ∠DFE (linear pair) = 180 – 140° = 40° C


∠ADB = ∠AFE = 40° (corresponding angles, BD || EF )
∠ADB = ∠DBC
(alternate Interior angles AD || BC ) A B
O
\ ∠DBC = 40°
∠AOC = ∠AOD + ∠COD
8. Given AB || CD.
∠BOC = ∠BOD – ∠COD
P ______________________
A B
100° ∠AOC – ∠BOC = ∠AOD – ∠BOD + 2 ∠COD
2x + 10° Q ∠AOC – ∠BOC = 2∠COD (∵ ∠AOD = ∠BOD = 90°)
E F
30° 1
∠COD = (∠AOC – ∠BOC)
30° 2
C D
R 3. Given AB || DE, EC || GF and ∠EGF = 100°.

Draw the line EF parallel to AB and CD and passing through Q. A


∠BPQ = 180° – ∠APQ = 180° – 100° ⇒ ∠BPQ = 80°
∠EQP = ∠BPQ = 80°
(alternate angles AB || EF ) E
∠DRQ = ∠EQR = 30° G
10
(alternate angles CD || EF ) 0°
∠PQR = ∠EQR + ∠EQP B C
2x + 10° = 80° + 30° D F
2x = 110° – 10° ⇒ x = 50° ∠ECF = 40°
9. ∠QPS = 180° – 120° = 60° ∠FGD = 180° – 100° = 80°
(∵ ℓ // m and interior angles on same side of the transversal.) ∠GFD = ∠ECF = 40° (corresponding angles)
∠RPT = 180° – 100° = 80° In DGDF, ∠GDF = 180° – (∠GFD + ∠FGD)
(∵ ℓ // m and interior angles on same side of the transversal.) = 180° – (40° + 80°) = 180° – 120° = 60°
\ ∠QPS + ∠RPT = 80° + 60° = 140° ∠ABC = ∠GDF = 60° (corresponding angles)

10. Supplement of x = 180° – x 4. Given, ℓ || m and ℓ1 || m1


Complement of x = 90° – x X P
Given that 180° – x = 6(90° – x)
⇒ 180° – x = 540° – 6x
⇒ 6x – x = 540° – 180° 3
⇒ 5x = 360° ⇒ x = 72° Y
A
Exercise 2 140°
Q
1. Given, AB, CD, EF are passing through the point O. ℓ B
A C m 150°
C
R n
ℓ1 m1
x z
E F ∠1 = 140° and ∠2 = 150°.
O ∠ABQ = ∠1 = 140° (corresponding angles)
y
∠ACR = ∠2 = 150° (corresponding angles)
∠ABC = 180° – ∠ABQ = 180° – 140° = 40°
D B ∠ACB = 180° – ∠ACR = 180° – 150° = 30°
Answer and Explanations 5

In DABC, ∠BAC = 180° – (∠ABC + ∠ACB) ∠AOC = 180o (AOC is Straight line)
= 180° – (40° + 30°) = 180° – 70° = 110° ∠AOB + ∠BOC = 180o
∠XAY = ∠BAC = 110° (vertically opposite angles) ∠BOC + ∠COD = 180 (∠AOB = ∠COD)
∠XAY = ∠3 = 110° (corresponding angles) ∠BOD = 180o
BOD is a straight line i.e., BD is a straight line.
5. Given, ℓ ⊥ m
b
R 9. Given a =
O ℓ 3
C
L
Q S
K a° b°
A B
O
m P
D
PQ and RS are incident and emergent rays.
We have, a + b = 180o (linear pair)
Through Q draw QL ⊥ m and through R draw RK ⊥ ℓ
b 4b
let ∠PQL = q + b = 180° ; = 180°
∠LQR = q (angle of incident = angle of reflection) 3 3
\ ∠OQR = 90° – q 3
 
And ∠OQR = ∠QRK = 90° - q (∵ ∠QOR = 90°)
b = 180   = 45 × 3 = 135°
 4
⇒ ∠KRS = 90° – q 6
a = 180 – 135° = 45°
Now ∠PQR + ∠QRS = 2q + 180° – 2q = 180°
∠AOD = b° = 135°
⇒ PQ || RS
∠BOD = a = 45°
6. Let the first angle, the second angle and the third angle be f, s
and t respectively. 10. Given, ∠ABC = 65°, ∠BCE = 35o, ∠ECD = 30o and ∠CEF = 150o
 90° − f   f A
s 
= = 30° − 
 3  
B
3
 180° − f   f
t 
=

=  90° −  E F
2 2
t = 6s (given)
C D
 f  f
\ 90° − =  6 30° − 
 2  3 \ ∠BCD = ∠BCE + ∠ECD = 30o + 35o = 65o
∠ABC = ∠BCD = 65° and BC is a transversal of AB and
\ 
 f 
⇒  90° − =  (180° − 2 f ) CD and CE is a transversal of EF and CD
2 ⇒ AB || CD ... (1)
180 − f ∠CEF + ∠ECD = 150° + 30° = 180°
= 180 – 3f ⇒ CD || EF  ... (2)
2
\ AB || CD || EF (from 1 and 2)
⇒ 180 – f = 360 – 6f
3f = 180° ⇒ f = 36° Exercise 3
7. a°, b°, c°, d° and e° can be in ascending order or in descending
1. (a) 
Given that QO is the bisector of ∠Q and RO is the bisector
order. of ∠R.
\ b = a + 1, c = a + 2, d = a + 3 and In DOQR, ∠QOR = 180° – (∠OQR + ∠ORQ)
e = a + 4 or b = a – 1, c = a – 2, d = a – 3 and e = a – 4
1
\ a + b + c + d + e = 5a + 10 or 5a – 10. = 180° − (∠PQR + ∠PRQ)
Sum of the angles around a point is 360°. 2
\ a° + b° + c° + d° + e° = 360° 1
= 180° − (180° – ∠P)
\ (5a + 10)° = 360° or (5a – 10)° = 360° 2
\ a° = 70° or 74° P
= 180° – 90° + ∠
8. Given: AC is a straight line and ∠AOB = ∠COD 2
B P
= 90° + ∠
A 2
(b) Given ∠ROQ = 110°
O P
We have ∠ROQ = 90° + ∠
2
E C P P
⇒ 110° = 90° + ∠ ⇒ ∠ = 20°
D 2 2
⇒ ∠P = 40°
6 Fundamental of Mathematics

2. (a) Given that BP is the bisector of ∠ABC and CP is the 5. (a) Given that ∠BAC = 50° and AB // PQ.
bisector of ∠ACD.
B
A P Q

y D

2x
50° x
B C D A C
E
In DABC,
P
∠ACD = ∠A + ∠ABC (exterior angle of a triangle is
equal to the sum of interior opposite angles} ∠BAC + 2x = 180° (interior angles on same side of the
⇒ 
1  1  1 transversal)
 ∠ACD =   ∠A +   ∠ABC  ... (1)
2  2  2 ⇒ 50° + 2x = 180° ⇒ 2x = 180° – 50°
From DPBC, ∠PCD = ∠PBC + ∠P  ... (2) 130°
⇒ x
= = 65°
 1  1 2

  ∠ACD =   (∠ABC + ∠P) ... (3)
2 2 (b) In DABC, ∠BAC + ∠ACB + ∠ABC = 180°
⇒ 50° + x + ∠ABC = 180°
From (1) and (3),
⇒ 50° + 65° + ∠ABC = 180°
 1  1  1 ⇒ ∠ABC = 180° – (115°) = 65°

  ∠A +   ∠ABC =   (∠ABC + ∠P)
2 2 2 ∠ABC + y = 180° (∵ AB // PQ and interior angles on the
(c) 
 1 same side of the transversal)
∠BPC =   ∠A
 2 ⇒ 65° + y = 180° ⇒ y = 180° – 65° = 115°
(b) Let ∠PQS and ∠SQR be 7x° and 5x° respectively. 6. (a) 70°
∠PQS + SQR = 180° (∵ PQR is a straight line).
A
S

120°
x
P R B C D
Q
From the figure ∠BAC = 70°
7x° + 5x° = 180° (vertically opposite angles)
x° = 15° ∠ACD = ∠BAC + ∠ABC (exterior angle of a triangle is
∠SQR = 5x° = 75°. equal to the sum of two interior opposite angles)
3. (a) DE is the bisector of ∠BDF. 120o = 70o + x ⇒ x = 50o
Let ∠EDF = ∠EDB = x ⇒ ∠ADB = 2x (b)
\ x + x + 2x = 180° ⇒ x = 45° A
(∵ ADF is a straight line) ℓ
Also ∠DBA = ∠EDB = x = 45° B C
\ ∠DBC = 135° 50° 80°
(b) In DADB, ∠A + ∠ADB + ∠DBA = 180° m
D E
⇒ ∠A = 180° – (2x + x) = 180° – 3(45°) = 45°.
q p
4. Let ∠PQT = ∠TRS = x, ∠TQS = ∠TRP = y
Since ℓ // m, ∠ABC = ∠BDE = 50°
P ∠ACB = ∠CED = 80° (corresponding angles)
⇒ ∠BAC = 180° – (50° + 80°) = 50°
80° \ In DABC, ∠ABC = ∠BAC = 50°
\ DABC is an isosceles triangle.
T
7.
X S X
y x Y
120°
Q R

∠SQR = a and ∠SRQ = b.


In DSQR, a + b = 180° – ∠S = 60°
In DPQR
2(x + y) + a + b = 100° [\ ∠P = 80°] Y Z
M N
x + y = 20° (∵ a + b = 60°)
In DTQR, ∠QTR = 180° – (x + y + a + b) Given, XM ^ YZ and
= 180° – (20° + 60) ° = 100° XN is the bisector of ∠YXZ
Answer and Explanations 7

∠MXN = ∠YXN – ∠YXM  … (I) 90°


 1 ⇒ 90° = 2x° ⇒ x° = = 45°
∠YXN =   ∠YXZ (given) … (1) 2
 2
The required answer = 180° – 45° = 135°
∠YXM = 90° – ∠Y  … (2) 2. ∠BOC = 100°
Substituting the values of (1) and (2) in I
2
 1 ∠DOC = ∠AOB
∠MXN =   ∠YXZ – (90° – ∠Y) 3
 2
∠AOB + ∠BOC + ∠DOC = 180° (∵ AOD is a line)
 1 2
=   (180° – ∠Y – ∠Z) – 90° + ∠Y ⇒ ∠AOB + 100° + ∠AOB = 180°
 2 3
 1  1  2
= 90° –   ∠Y –   ∠Z – 90° + ∠Y ⇒ ∠AOB  1 +  = 180° – 100° = 80°
 2  2  3
 1  1  1  5
=   ∠Y –   ∠Z =   (∠Y – ∠Z) ∴ ∠AOB   = 80°
 2  2  2  3
8. (a) ∠BGC = 360° – (140° + 90°) = 130° 80 × 3
∴ ∠AOB = = 48°
In DBGC, ∠GBC = 180° – (130° + 30°) = 20° 5
⇒ ∠ABG = ∠ABC – ∠GBC = 60° – 20° 2
In DABG, x = 180° – (90° + 40°) = 50° ∴ ∠DOC = × 48°= 32°
(b) Given x + y = 80° and z – y = 55° 3 6
As BC || QP, ∠OBC = ∠OPQ (alternate interior angles) 3. Given, ℓ || m, AC and BC are the bisectors of the angles ∠PAB
\ ∠OPQ = x (∵ ∠OBC = x) and ∠RBA
z = 180° – (x + y) = 180° – 80° = 100° ∴  ∠PAB and ∠RBA are interior angles on the same side of
z – y = 55° ⇒ 100 – y = 55° ⇒ y = 45° ⇒ x = 35° the transversal and hence they are supplementary.

9. (a) In the figure given below, n

A Q A P

B E C
B
m
20° 15° S R
50° 25°
C P Q D ⇒ ∠PAB + ∠RBA = 180°
1 1
⇒ (∠PAB + ∠RBA) = × 180°= 90°
⇒ ∠BPQ = 70º and ∠EQD = 140º ⇒ ∠EQP = 40º 2 2
\ ∠BAE = 180º - (70º + 40º) = 70º ∠PAB ∠RBA
(b) ∠x + 150° = 180º ⇒ + = 90°
\ x = 30º 2 2
y + 100° = 180° \ y = 80° ∠CAB + ∠CBA = 90° (∵ AC and BC are bisectors of
⇒ 
\ x + y = 110o ∠PAB and ∠RBA)
In D ABC
10. (a) In the given figure ℓ || m and ℓ1 || m1 ∠ACB + ∠CAB + ∠CBA = 180° (angle sum property)
∠PRQ = ∠ACB = 40° ⇒ ∠ACB + 90° = 180°
(∵ Interior alternate angles) ⇒ ∠ACB = 180° – 90° = 90°.
∠RQS = ∠DBC = 110°
(∵ Exterior alternate angles) 4. Since ‘ℓ’ and ‘m’ are perpendicular to the same line ‘n’ they are
parallel to each other.
In DPQR ∠RQS = ∠RPQ + ∠PRQ
∴ ℓ || m, ‘n’ is the traversal.
110° = ∠RPQ + 40°
In DABC ∠ABC = 90° (m ⊥ n)
∠RPQ = 110° - 40° ⇒ ∠QPR = 70°
AB = BC ⇒ ∠CAB = ∠ACB = 45°
(b) Let ∠ACB = xº = ∠DCE (vertically opposite angles)
Now ∠PCA is the exterior angle of DABC
50º + ∠BAC + xº = 75º + xº + ∠CDE
∴ ∠PCA = ∠CAB + ∠ABC = 45° + 90° = 135°.
⇒ ∠BAC - ∠CDE = 35º
5. Let the angle be x°.
HOTS 1
⇒ (180° – x°) = 2.5 (90° – x°)
1. Let the angle be x°.
4
1 25
1 ⇒  (180° – x°) = (90° – x°)
⇒ (90° – x°) = (180° – x°) 4 10
3
1 5
⇒ 270° – 3x° = 180° – x° ⇒  (180° – x°) = (90° – x°)
⇒ 270° – 180° = 3x° – x° = 2x° 4 2
8 Fundamental of Mathematics

360° – 2x° = 1800° – 20x° ⇒ 2∠POQ = ∠BOQ + ∠AOQ – 2∠AOQ


20x° – 2x° = 1800° – 360° ⇒ 2∠POQ = ∠BOQ – ∠AOQ
18x° = 1440 1
1440 ⇒ ∠POQ = (∠BOQ – ∠AOQ).
x°= = 80° 2
18 9. ℓ || m
∴ The angle = 80° ∠PAC = 35°
The compliment of the angle = 90° – 80° = 10°. ∠RBC = 25°
6. ℓ || m, ∠PAC = 70°, ∠RBC = 30° and AC = BC Draw a line s’ parallel to ℓ
In DABC, AC = BC s || ℓ and ℓ || m
∠CAB = ∠CBA = x° ⇒ s || ℓ || m
n n
Q A P Q A P
ℓ ℓ
35°
C U C T s
D
B B 25°
m m
S R S R

∠PAB + ∠RBA = 180° (co interior angles) ∠PAC = ∠ACU (alternate angles)
∠PAC + ∠CAB + ∠RBC + ∠CBA = 180° ∴ ∠ACU = 35°
70° + x° + 30° + x° = 180° ∠RBC = ∠UCB (alternate angles)
∴ ∠UCB = 25°
100° + 2x° = 180°
∠ACB =∠ACU + ∠UCB = 35° + 25° = 60°
2x° = 180° – 100° Now ∠BCD = 180° – ∠ACB = 180° – 60° = 120°.
80°
x°= = 40° ⇒ ∠CAB = ∠CBA = 40° 10. AB || CD, ∠BAE = 50°, ∠DCE = 60°
2 ∠BAE = ∠CDE (alternate angles)
In D ABC ∴ ∠CDE = 50°
∠ACB = 180° – (∠CAB + ∠CBA) ∠DCE = ∠ABE (alternate angles)
= 180° – (40° + 40°) = 180° – 80° = 100° ∴ ∠ABE = 60°

7. Let the least angle of the triangle be x° A C


∴ the angles of the triangle are x°, x° + 15°, x° + 30°
x° + x° + 15°+ x° + 30° = 180° (sum of angles of a triangle)
⇒ 
3x° + 45° = 180° E
3x° = 180° – 45° = 135°
135° B D
x°= = 45°
3 In Δ ABE
∴ Angles of the triangle are 45°, 45° + 15°, 45° + 30° ∠BAE = 50°
⇒ 45°, 60°, 75°. ∠ABE = 60°
  ∴ ∠AEB = 180° – (50° + 60°)
8. OP ⊥ AB .
= 180° – 110° = 70°.
∠AOP = ∠BOP = 90°
∠BOQ + ∠AOQ = 180° 11. Given ℓ || m, ∠BAD = 105°
∠BOP + ∠POQ + ∠AOQ = 180° s t
∠AOP + ∠POQ + ∠AOQ = 180°(∵ ∠AOP = ∠BOP)
Now ∠AOP = ∠AOQ + ∠POQ Q E P

∠AOQ + ∠POQ + ∠AOQ + ∠POQ = 180° A B
2 (∠AOQ + ∠POQ) = 180°
S D C R
m
P
Q

Now ∠ADC +∠BAD = 180


⇒ ∠ADC = 180° – 105° = 75°
A O B Similarly, ∠DCB = 180° – 110° = 70°
∠AEB = ∠DEC
⇒ 2 (∠AOQ + ∠POQ) = ∠BOQ + ∠AOQ In D DEC, ∠DEC = 180° – 70°– 75° = 35°
(∵ ∠BOQ + ∠AOQ = 180°) ∠AEB = 35°.
Answer and Explanations 9

12. AOB is a line 16. x + y = 90°


∴ ∠AOC + ∠BOC = 180°, y + z = 180°
2 z = 4x
° 180°
x° + x=
3
 2
x° 1 +  = 180°
 3
y
 5 x z
x°   =180°
 3
 3
° 180 ×  =
x= 108°
 5 
⇒ ∴ y + 4x =180°
2 (x + y = 90°) × (4)
∠BOC = 108° ⇒ ∠AOC =
∴  × 108° = 72°.
3 4 x + y = 180°
13. a, b, c, d are the angles around a point. 4 x + 4 y = 360°
– –
∴ a + b + c + d = 360°
Let a = x° 3=y 180°
⇒ b = x° + 20°, c = x° + 40° and d = x° + 60° 180°
Now x° + x° + 20° + x° + 40° + x° + 60° = 360° y= = 60°
3
4x° + 120° = 360° 6
4x° = 360° – 120° = 240° ⇒ x = 30°.
240° z = 30°
x°= = 60° The required answer is 30°,60°,120°.
4
∴ The angles a, b, c, d are 17. ℓ || m
a = 60°, b = 80°, c = 100° and d = 120° ∠BAC = 45°
∠BCR = 75°
14. ℓ || m, ∠BAM = 95° ⇒ ∠BAD = 85° (linear pair)

t s t s
K M
Q B A P
ℓ Q A B P

C m
S C D R
m S R
J L
∠BAC = ∠ACS(Alternate angles)
∠ACS=45°
⇒ ∠CDA = 95° (corresponding angles)
Similarly , ∠BCR = 75°
s || t is the transversal
∠ACS +∠ACB + ∠BCR = 180°
∠BAM + ∠KBA = 180°
∠ACB = 180° – 45°– 75° = 60°
⇒ ∠KBA = 180° – 95° = 85°
⇒ ∠KBA = ∠BCD (Corresponding angles) 18. Given, ℓ || m || n
∴ ∠BCD = 85° y : z = 9 : 11
∴ Angles of the quadrilateral are Let y = 9k
∠ABC = 95°, ∠BAD = 85° z = 11k
∠CDA = 95°, ∠BCD = 85°.
S
15. ‘x’ and ‘y’ form a linear pair
x K
∴ x + y =180° Ɛ
1 P A Q
z= x. y
3 m
K B S
x + y + z = 220°
n
z = 220° – (x + y) T C z U
= 220° – 180 = 40°
z = 40° P
1
x 40°
=
3 ∠ABS = ∠BCU (corresponding angles)
⇒ x° = 120° ⇒ y = 60° ⇒ ∠BCU = y
The required answer is 120°, 60°, 40°. Now ∠BCU + ∠UCJ = 180° (linear pair)
10 Fundamental of Mathematics

y + z = 180° 20. ℓ || m
9k + 11k = 180° ⇒ 20k = 180° ∠ABS = 100°
180° ∠ACR = 120°
=k = 9
20
s t
∴ y = 9 × 9 = 81°
z = 11 × 9 = 99° Q A P
∠KAQ = 180° – x = ∠ABS (corresponding angles) ℓ
⇒ 180 – x = y
⇒ x = 180 – y = 180 – 81° = 99°
19. AB || CD S R
∠DCE = 100°∠ABE = 120° m
B C
Draw a line PQ through E parallel to CD
C D
A B ∠ABS + ∠QAB = 180° (Co interior angles)
100° ⇒ ∠QAB = 180° – 100° = 80°
120° ∠ACR + ∠PAC = 180° (Co interior angles)
∴ ∠PAC = 180° – 120° = 60°
Now ∠QAB + ∠BAC + ∠PAC = 180°
P E Q ⇒ ∠BAC = 180° – (80° + 60°) = 180° – 140° = 40°
Now ∠ABS = ∠BAC + ∠ACB (exterior angle of a
∴ PQ || CD, AB || CD
triangle is equal to the sum of interior angles)
∴ PQ || AB || CD
= 100° = 40° + ∠ACB
∠DCE + ∠CEQ = 180° (co interior angles)
∴ ∠CEQ = 180° – 100° = 80° ⇒ ∠ACB = 100° – 40° = 60°
∠ABE = ∠BEQ. (alternate angles) ∠ACR = ∠BAC + ∠ABC
120° = ∠BEC + ∠CEQ 120° = 40° + ∠ABC ⇒ ∠ABC = 120° – 40° = 80°
120° = ∠BEC + 80° ∴ Angles of the triangle are 40°, 60°, 80°.
⇒ ∠BEC = 120° – 80° = 40°
Triangles
Answer and Explanations 7
3. The third angle is 180o - (45o + 90o) = 45o.
STEPWISE MARKING SOLUTION
\ The given triangle is a right isosceles triangle. (1 Mark)

Objective Type Question 4. Given AB = AC ⇒∠B = ∠C


And also given∠A = 3∠B
1. (b) Given AB = FD We have, ∠A + ∠B + ∠C = 180o  (1 Mark)
And ∠A = ∠D 3∠B + ∠B + ∠B = 180° ⇒∠B = 36°
As we know that if two angles and the included side of \ ∠C = 36° (1 Mark)
one triangle are equal to the two angles and side of other
triangle 5. Given DACB @ DACH @ DBCH
\ AC = DE \ ∠ACB = ∠ACH = ∠BCH
Angle at C is 360°
2. (a) ∠ABC = 35° (vertically opposite angles)
\ x = 57° + 35° = 92°
360°
⇒ ∠BCH = = 120°  (2 Marks)
(An exterior angle is equal to the sum of two interior 3
opposite angles). Exercise 2
3. (b) BD = CD (Given AD is perpendicular bisector of BC)
1. In DABC, AB = BC ⇒ ∠A = ∠C = 45o (∵ ∠ABC = 90o)
AD = AD (common side)
In DABD and DCBD, AB = BC (given)
∠ADB = ∠ADC = 90° (Given)
AD = CD(given) and BD is common side.
\ By SAS axiom, ΔADB ≅ ΔADC.
\ DABD ≅ DCBD (SSS congruence property) (11/2 Marks)
4. (d) ∠ABD = ∠CBD (by CPCT)
Q But ∠ABD + ∠CBD = ∠ABC = 90o 7
\ ∠ABD = ∠CBD = 45o
As ∠ABD = ∠A = 45o, AD = BD. (11/2 Marks)
P 70° O 2. Given,BD = DC and AD ^ BC
Proof: In DABD and DACD
∠ADB = ∠ADC = 90°(given)
R AD is common side (11/2 Marks)
DABD @ DACD (SAS congruence property)
180° − 70° AB = AC (by CPCT) (11/2 Marks)
PQ = PR ⇒ ∠Q = ∠R =
2
3. Given, BE ^ AC, CF ^ AB and BE = CF
⇒ ∠Q = ∠R = 55° Proof : in DAEB and DAFC
OQ and OR bisects ∠Q and ∠R respectively. A is common ∠AFC = ∠AEB = 90°and BE = CF (11/2 Marks)
°
55°  1  \ DAEB @ DAFC (AAS congruence property)
\ ∠OQR = ∠ORQ = =  27 
2  2 AB = AC(by CPCT) (11/2 Marks)
 1° 1°  4.
\ ∠QOR = 180° −  27 + 27 =  125° .
 2 2 D
Alternate Method:
∠P
∠ROQ = 90° + = 90° + 35° = 125°
2
5. True A
6. False

7. False

8. One B C
9. One
Given, AB = AC andAC = AD
In DABC, AB = AC
Subjective Type Question
∠ABC = ∠ACB = x (say) and
Exercise 1 ∠ACD = ∠ADC = y (say) (11/2 Marks)
Now in DDBC,x + x + y + y = 180o
1. Given statement is true. (1 Mark) 2 (x + y) = 180o ⇒ x + y = 90o
2. Given statement is true. (1 Mark) ⇒ ∠BCD = 90° (11/2 Marks)
2 Fundamental of Mathematics

Exercise 3 (b)
A
1.

A P

B C
D

Given, AC > AB and ∠BAD = ∠CAD


B C Q R In DABC,∠ABC >∠ACB
D S
(∵ AC > AB) (1 Mark)
Given, AB = PQ,  1  1
∠ ABC +   ∠A >∠ ACB +   ∠A (1 Mark)
 2  2
and AD = PS
⇒ 180o - ∠BDA > 180o - ∠ADC
1 1 
 BC = QR  ⇒ BD = QS -∠BDA > -∠ACD
2 2  ⇒ ∠ADC > ∠BDA (1 Mark)
Now, in DABD and DPQS
AB = PQ, BD = QSand AD = PS (1 Mark) PROBLEMS FOR ADDITIONAL PRACTICE
DABD @ D PQS (SSS congruence property)
∠B = ∠Q (by CPCT) (2 Marks) Objective Type Question
We consider DABC and DPQR 1. (a) D ABC @ D PQR
AB = PQ, BC = QR AB BC
∴ =
and ∠B = ∠Q (by proof) PQ QR
DABC @ D PQR (SAS congruence property). (1 Mark) DABC not congruent to DRPQ then
AB
ÂC
2. ≠
RP QR
P \ BC = PQ is not correct.
180° − 90°
T 2. (a) AD = CD ⇒ ∠DAC = ∠ C= = 45°
2
S
AB = AC ⇒ ∠B = ∠C = 45°
∠BAC = 180° – (∠B + ∠C) = 180° – (45° + 45°) = 90°.
Q R 3. (d) ∠Q = 180° – (∠P + ∠R) = 180° – (55° + 75°) = 50°
\ ∠R > ∠P > ∠Q ⇒ PQ > QR > PR.
Given, S is a point in the interior of DPQR.Construction:
4. False
Extend QS meeting PR at T.
Proof : In DSTR, ST + TR > SR  ... (1) 5. True
(In a triangle the sum of any two sides is greater than third side)
6. True
Similarly in DPQT(1Mark)
PQ + PT > QT  ... (2) 7. Triangle
(In a triangle the sum of any two sides is greater than third side)
8. An equilateral
 (1 Mark)
Adding (1) and (2) 9. Third
⇒ ST + TR + PQ + PT > SR + QT
⇒ ST + PR - PT + PQ + PT
Subjective Type Question
> SR + QS + ST
\ PQ + PR > SQ + SR (2 Marks) Exercise 1
3. (a) Given,∠BAD = ∠CAD 1. The given statement is true.
 1
Proof : in DADC, DADB =   ∠A + ∠C 2. The given statement is true.
 2
3. AB (the side opposite to greatest angle)
(exterior angle is equal to the sum of interior opposite
angles) (1 Mark) 4. Given PQ = AB, QR = BC, ∠Q = ∠B = 90o.
 1 By using SAS congruence property, ∆PQR ≅ ∆ABC.
(In DABD, DADB >   ∠A
 2 5. We have ∠A + ∠B + ∠C = 180o
DADB > DBAD) ∠B = 180o - 50o - 50o = 80o
\ AB > BD (1 Mark) \ AC is the longest side in DABC.
Answer and Explanations 3

6. Given statement is false. A

7. Given statement is true.

8. Given statement is false. (∵ 6 + 7 < 14)

9. In DABC let ∠A = ∠B = 40o B M C


We have, ∠A + B + ∠C = 180o
∠C = 180o - 40o - 40o = 100o
\ The greatest angle is 100o. D
10. In DABC, ∠A = 80o and AB = AC
As AB = AC 4. Given, AB = AC and DB = DC
∠C = ∠B Join AD and BC and mark the point of intersection as ‘M’
We have ∠A + B + ∠C = 180o In DABD and DACD,
\ ∠B + ∠C = 180o - 80o = 100o AB = AC, BD = DC (given) and AD is common.
\ ∠B = ∠C = 50o \ DABD @ DACD (SSS congruence property)
∠BAM = ∠CAM (by CPCT)
Exercise 2 Now in DBAM and DCAM
1. Given, AB = CG, AB = AC and AM is common
EG = BD and ∠AGE = ∠DBC ∠BAM = ∠CAM (by proof)
In DDBC and DEAG DBAM @ DCAM (SAS congruence property)
AB = CG BM = CM and ∠AMB = ∠AMC (by CPCT)
⇒ AB + BG = CG + BG ⇒ AG = BC Also, ∠BMA + ∠CMA = 180° (linear pair)
EG = BD ⇒ 2∠BMA = 180o
∠AGE = ∠DBC (given) ⇒ ∠BMA = 90o
DAGE @ DCBD \ AD bisector BC at right angles. 7
(By SAS congruence property) 5. Given, AB = AC and BP = CQ
2. Given: ∠AOP = ∠BOP In DABP and DACQ,
AB = AC and PB = CQ (given)
A ⇒ ∠ABC = ∠ACB
and ∠ABP = ∠ACQ
DABP @ DACQ (SAS congruency)
A AP = AQ (by CPCT)
P
6.
D P Q

O N B

Draw DM ^ AO and DN ^ BO.


In DDOM and DDON A B
OD is common C
∠MOD = ∠NOD (given) In DAQC and DBPC, AC = BC (given)
∠DMO = ∠DNO = 90° ∠A = ∠B (given)
DDOM @ DDON (AAS congruency) ∠ACP = ∠BCQ (given)
\ DM = DN (by CPCT) ∠ACP + ∠PCQ = ∠BCQ + ∠PCQ
∠ACQ = ∠PCB
3.
A DPCB @ DQCA (ASA congruence property)
\ AQ = BP
7.
F E

B C

Given AB = AC, BE ^ AC and CF ^ AB


In DABE and DACF
AB = AC
∠AEB = ∠AFC = 90°
∠A is common Given, AB = AC and DB = DC
DAFC @ DAEB (AAS congruence property) Join AD
BE = CF (by CPCT) In DABD and DACD, AB = AC
4 Fundamental of Mathematics

BD = CD and AD is common Join A to C and B to D


DABD @ DACD (SSS congruence property) In DABC, ∠BAC > ∠BCA (AB < BC)  ... (1)
∠ABD = ∠ACD (by CPCT) Similarly in DADC,
∠CAD > ∠DCA (CD > AD)  ... (2)
8.
D From (1) and (2)
∠BAC + ∠CAD > ∠BCA + ∠DCA
A P ⇒ ∠A > ∠C
Now in DBAD,
∠ADB < ∠ABD (AD > AB)  ... (3)
And in DBDC,
∠BDC < ∠DBC (CD > BC) … (4)
From (3) and (4), ∠ADB + ∠BDC < ∠ABD + ∠DBC
B C ⇒ ∠D < ∠B.
Given, AP || BC, AP bisects ∠DAC. 4.
AP || BC A
∠PAC = ∠C ...(1) (alternate angles)
∠DAP = ∠B ...(2) (corresponding angles)
∠DAP = ∠PAC (given)
∠B = ∠C ⇒ AB = AC B C
O
Exercise 3

1. Given AB = AC. CF bisects ∠C and BE bisects ∠B. P


In DAFC and DAEB
AB = AC, ∠A is common.
Given, DABC, BD = DC.
 1  1 Extend AD to P such that
∠ABE = ∠ACF   ∠B =   ∠C AD = DP and join PC.
 2  2
In DABD and DPDC
DAFC @ DAEB (SAS congruence property) AD = DP (construction)
BE = CF (by CPCT) BD = DC (given)
2. Given, ∠ADB = ∠PDC (vertically opposite angle)
A DADB @ DPDC (SAS congruence property)
PC = AB (by CPCT)
Now in DAPC,
AC + PC > AP
D AC + AB > 2AD [∵ AB = PC and AP = 2AD]
5. Given, AB = AC and AD ^ BC
B C In DABD and DACD
P
AB = AC (given)
(i) AB = AC ∠ADB = ∠ADC = 90o (given) and
(ii) DB = DC AD is common
Join AD and extend it to meet BC at P. DABD @ DACD (RHS congruency)
In DABD and DACD, AB = AC BD = DC (by CPCT)
DB = DC (given) and AD is common ∠BAD = ∠CAD (by CPCT)
\ DABD @ DACD (SSS congruence property) AD is the median drawn on to BC and bisector of ∠A
∠BAD = ∠CAD (by CPCT)
⇒ AD is the bisector of ∠A 6.
Now in DBAP and DCAP, AB = AC, AP is common T
∠BAP = ∠CAP (by proof)
\ DBAP @ DCAP (SAS congruence property)
\ ∠APB = ∠APC = 90° and BP = CP
S R
⇒ Extended AD is the perpendicular bisector of BC .
3.
D
A P Q

Given, PQ = QR = RS = SP = ST = RT
In DPST and DTRQ
ST = TR (given)
B C
SP = RQ (given)
Given, AB is the shortest side, CD is the longest side. ∠TSP = 90o + 60o = 150o
Answer and Explanations 5

∠TRQ = 90o + 60o = 150o Consider the right angled ∆ABD.


DTSP @ DTRQ (SAS congruence property) According to the Pythagoras theorem,
TP = TQ (by CPCT) 2
AB
= BD 2 + AD 2 Here, AB = 8 cm and BD = 4 cm.
7. 8= 4 + AD 2
2 2

2
A AD= 82 − 4 2
2
AD= 64 − 16
AD 2 = 48.
AD = 4 3 cm
B C
x y 2. Consider a ∆ABC where, BC > AC > AB. From the given data,
BC is the greatest side and AB is the smallest side. So, the
Given, AB > AC. greatest angle is the angle opposite to side BC i.e., ∠BAC. The
AB > AC (given) smallest angle is the angle opposite to AB i.e., ∠ACB.
∠ACB > ∠ABC Let ∠BAC = x°
(the angle opposite to longer side is greater)
∠ACB + ∠A > ∠ABC + ∠A A
⇒  x > y (exterior angle is equal to the sum of its interior
opposite angles)
8.

D B C
7
⇒ ∠ACB = x° – 59° and ∠ABC = x° – 13° 
 (From the given data)
B C
∴ x° + x°– 59° + x° – 13° = 180° (angle sum property)
In DABC, ∠BDC = ∠BDA = 90o 3x° – 72° = 180°
(a) In DABD and DCBD, AB = AC (given) ⇒ 3x° = 180° + 72° = 252°
∠A = ∠C (AB = AC)
∠ADB = ∠CDB = 90o 252°
⇒ x°= = 84°
By AAS congruence property, DABD @DCBD. 3
(b) Given AD = CD Thus,
In DABD and DCBD ∠BAC = x° = 84°
AD = DC (given)
∠ACB = x° – 59° = 84° – 59° = 25°
∠ADB = ∠CDB = 90o
BD is common side (by SAS congruence property) ∠ABC = x° – 13° = 84° – 13° = 71°
DABD @ DCBD 3. In a triangle, the sum of any two sides is greater than the third
side.
HOTS Also, the difference of two sides of a triangle is always lesser
than the third side.
1. Given, AB = AC = 6 cm and BC = 8 cm.
In an isosceles triangle, the altitude drawn from the vertex A
containing the equal sides bisects the third side.
8
∴ BD = DC = = 4 cm 8 cm
2 7 cm
A

B C

∴ AB + AC > BC > AC – AB.


B C
D 7 + 8 > BC > AC – AB
Consider ∆ABD and ∆ADC, 15 > BC > 1
AB = AC = 6 cm (Given) ⇒ 1 < BC < 15.
BD = DC = 4 cm (proved) Thus, the measure of BC is an odd integer less than 15 and
∠ABD = ∠ACD (angles opposite to equal sides are equal) greater than 1.
∴ By the S.A.S congruency property, ∆ABD ≅ ∆ACD. ∴ BC = 3, 5, 7, 9, 11 or 13 (cm).
6 Fundamental of Mathematics

4. Given in the quadrilateral ACDB, AC = BD, ∠ABC = ∠BCD In ∆ BDC, BC > CD.
= 90°. ⇒ ∠BDC > ∠DBC ... (2)
Adding inequalities (1) and (2) ;
A ∠ADB + ∠BDC > ∠ABD + ∠DBC.
⇒ ∠ADC > ∠ABC
Thus, ∠D > ∠B.
7. Let the sides of the triangle be a = 11 cm, b = 13 cm, c = 23 cm.
B C In a triangle, the sum of any two sides should be greater than
the third side.,
Let us consider the sum of two sides at a time.
a + c = 34cm, b = 13cm
⇒ a + c > b, a + b = 24 cm, c = 23 cm
D ⇒ a + b > c, b + c = 36 cm, a = 11 cm
⇒ b+c>a
Consider ∆ABC and ∆BCD. It can also be observed from the given measures, that the difference
∠ABC = ∠BCD = 90° (given) of a any two sides of a triangle is lesser that the third side.
AC = BD (given) Thus, a triangle can be constructed by using the given measures.
BC = BC (common side)
8. In ∆ABC, AD is the median drawn onto BC.
By R.H.S Congruence Property, ∆ABC ≅ ∆DCB
A median divides the side into two equal parts.
∴ AB = CD (corresponding parts of congruent triangles)
∴ BD = DC
5. Given, AC is the greatest side and AB is the shortest side of the In ∆ABD,
∆ABC. AB + BD > AD (sum of two sides is greater than the third
side) ... (1)
A A

B C
B C
⇒ ∠ABC < ∠ACB. D
BO and CO are angle bisectors of ∠ABC and ∠ACB, In ∆ADC,
respectively. AC + CD > AD (sum of two sides greater than the third side)
⇒ ∠ABO = ∠OBC, ∠ACO = ∠OCB.  ... (2)
Let ∠ABO = ∠OBC = x° and ∠ACO = ∠OCB = y° Adding (1) and (2);
∠ABC > ∠ACB. AB + BD + AC + CD > AD + AD
∠ABC ∠ACB AB + BC + AC > 2AD
⇒ > . ∴ 6 + 9 + 7 > 2AD
2 2 ⇒ 2AD < 22
⇒ ∠OBC > ∠OCB ⇒ AD < 11.
⇒ x° > y° ∴ The maximum possible integer value of AD is 10cm.
In ∆BOC, ∠BOC = 100°
9. In the given figure, p || q and l || m.
⇒ x° + y° = 180° – 100° = 80°
If y° = 50°, x° = 30° which contradicts the condition x° > AO, BO are the bisectors of ∠DAB and ∠CBA.
y°. m l
∴ ∠OCB cannot be equal to 50°.
6. In the given quadrilateral ADCB, AB is the greatest side and
D C
p
CD is the shortest side. O
Join BD. BD divides the quadrilateral into two triangles.
q
A A B
D
Let ∠DAB = x°, ∠CBA = y°
Now x° + y° = 180° (Sum of interior angles on the same side
of the transversal)
AO, BO are bisectors of x° and y°
x° y°
∴ ∠OAB = , ∠OAB =
B C 2 2
In ∆ AOB, ∠AOB + ∠OAB + ∠OBA = 180°
In ∆ABD, AB > AD. ∠AOB = 180° – (∠OAB + ∠OBA)
⇒ ∠ADB > ∠ABD ... (1)
Answer and Explanations 7

 x0 ∴ The side opposite to ∠B is AC, which is the greatest side.


y0 
= 180° −  +  Thus, the hypotenuse is the greatest side of a right-angled
2 2 triangle.

 x0 + y 0  11. AD is the altitude which bisects the hypotenuse BC.


= 180° −  
 2  A

 x0 + y 0 
= 180° −  
 2 
1800 
= 180° =  (∵ x° + y° = 180°)
 2  B
D
C
= 180° – 90°
= 90°. BD = DC
Thus, ∠AOB = 90°. Consider ∆ABD and ∆ADC.
∠ADB = ∠ADC = 90° (∵ AD is the altitude)
10. Let us take a right angled ∆ABC in which ∠B = 90°. BD = DC (given)
Let ∠ACB = x° and ∠BAC = y° AD = AD (common side)
Now x° + y° + 90° = 180° (angle sum property of a triangle) By the S.A.S congruency property, ∆ADB ≅ ∆ADC.
x° + y° = 180° – 90° = 90° ⇒ AB = AC (C.P.C.T)
⇒ x° and y° cannot be greater than 90° individually. ∴ ∆ABC is an isosceles right-angled triangle.
⇒ ∠B = 90° is the greatest angle of ∆ABC.

7
Quadrilaterals
Answer and Explanations 8
3. The given statement is false. (1 Mark)
STEPWISE MARKING SOLUTION
4. The given statement is true. (1 Mark)
Objective Type Question 5. The given statement is false. (1 Mark)
1. (d) Isosceles trapezium.
Exercise 2
A D
x 1. Given that ABCD is a rectangle and ∠BAC = 35°.
x
D C
y O
y
B C O

2. (c) Let ∠A = 2x and ∠B = 2y 35°


2x + 2y = 180° A B
⇒ x + y = 90°
In ∆AOB, Let the diagonals AC and BD intersect at O.
∠AOB = 180° – (x + y) = 180° – 90° = 90°. OA = OB (diagonals are equal and bisects each other)
 (1 Mark)
3. (c) Parallelogram.
⇒ ∠OAB = ∠OBA = 35°
4. (c) AO = OC We know that ∠ABC = 90°
AC = AO + OC = 4 + 4 = 8 cm. AC = BD = 8 cm ∠DBC = ∠ABC −∠OBA (1 Mark)
A D ⇒ 90° − 35° = 55° (1 Mark)

O 2.

A
8
B C

5. (d) Every rectangle is a trapezium is a true statement.


B C
6. (a) AB // CD D
A B
30°
X
30° In ∆ABC, AD is the median drawn on to BC.
D C
⇒ BD = DC.
⇒ ∠ABD = ∠BDC = 30°. And also AD = DX.
(∵ alternate interior angles are equal) In quadrilateral ABXC,
AD = DX and BD = DC
7. True
(D is the midpoint of BC )(11/2 Marks)
8. False
Since the diagonals bisects each other,
9. True The quadrilateral ABXC is a parallelogram.  (11/2 Marks)

10. Not 3. Let ABCD be a parallelogram.

D C
11. Square

12. Right

Subjective Type Question


Exercise 1 A B
1. The given statement is true. (1 Mark) ∠B
Let ∠A=
2. The given statement is true. (1 Mark) 3
2 Fundamental of Mathematics

∠A + ∠B = 180° (adjacent angles of a parallelogram) 2.


∠B
+ ∠=B 180° (11/2 Marks) A
3
4∠B P
⇒ = 180° B
3 C
3
⇒ ∠B = 180 × = 45 × 3 = 135° R
4 Q
135° Given, AB = PQ, AB || PQ
∴ ∠A = = 45°
3 BC = QR, BC || QR
∠C = 45° and ∠D = 135° Join A to P, B to Q and C to R.
In quadrilateral ABQP, (1 Mark)
(opposite angles are equal) (11/2 Marks)
AB || PQ and AB = PQ (given)
4. In ΔADB, 7p° + 5p° + 3p° = 180° ∴ Quadrilateral ABQP is a parallelogram
⇒ 15p° = 180° ⇒ p° = 12° (11/2 Marks) ⇒ AP || BQ and AP = BQ ... (1) (11/2 Marks)
Since ABCD is a parallelogram, Similarly quadrilateral BQRC is a parallelogram.
⇒ 7p° = r° (opposite angles are equal) (11/2 Marks) ⇒ BQ || CR and BQ = CR ... (2)
∴ r° = 7(12°) = 84° From (1) and (2) AP || CR and AP = CR
5. x° + (x + 1)° + (x + 2)° + (x + 3)° = 360°
∴ Quadrilateral APRC is parallelogram.
⇒ (4x + 6)° = 360° ∴ AC || PR and AC = PR. (11/2 Marks)
⇒ 4x° = 354° 3.
 1
⇒ x 88  ° (11/2 Marks)
= A B
 2
 1
∴ ∠A == x° 88  ° P Q
 2
 1
∠B = (x + 1)° = 89  °
 2 D R C
 1
∠C = (x + 2)° =  90  ° Given, AB || DC, P and Q are the midpoints of DB and AC
 2 respectively.
 1 Join B to Q and extend to meet DC at R. (1 Mark)
∠D = (x + 3)° =  91  ° (1/2 Mark) In DAQB and DRQC
 2
AQ = QC (given).
Clearly, ∠B + ∠C = 180°
∠AQB = ∠RQC
∠A + ∠D = 180° ⇒ AB || CD.
∴ ABCD is a trapezium. (1 Mark) (vertically opposite angles)
∠BAQ = ∠RCQ
Exercise 3 (alternate angles and AB || CR
1. DAQB ≅ DCQR [AAS congruency]  (1 Marks)
D C AB = RC (CPCT)
Q BQ = RQ (CPCT) (1Mark)
Now in DDBR
O P is the midpoint of DB and Q is the midpoint of BR.
PQ || DR
P 1 1 1
A B
 DR   [ DC − RC] =   [ CD − AB] 
PQ = =
2 2 2
Given that ABCD is a parallelogram and AP = CQ.  (1 Marks)
Join BD meeting AC at O.
In parallelogram ABCD,
AC and BD are diagonals. (11/2 Marks)
AO = CO and BO = OD [Diagonals a parallelogram bisect each PROBLEMS FOR ADDITIONAL PRACTICE
other) (2 Marks)
AP = CQ (given) Objective Type Question
AO – AP = CO – CQ
1. (b) In ∆KLM, KL = LM
⇒ OP = OQ
In the quadrilateral DPBQ, OP = OQ and OB = OD. 180° − 140°
⇒ ∠KML = = 20° .
∴ DPBQ is a parallelogram. (11/2 Marks) 2
Answer and Explanations 3

2. (b) Square. 3. (a) 


In a rhombus, if adjacent angles are equal, then each angle
would be 90°.
3. (c) Given, ∠E = 2∠S
Such a rhombus is a square.
∠E + ∠S = 180°⇒ 2∠S + ∠S = 180°⇒∠S = 60°
(b) A rectangle whose adjacent sides are equal, would have
∠B = ∠S = 60°. all sides equal.
4. (d) Rhombus. ∴ Such a rectangle would be a square.
∴ Both a and b must be squares.
5. (a) Given, ∠DBC = ∠BDA
4. A parallelogram in which the diagonals bisect each other at
A D right angles must be a rhombus.
5. The angles of a quadrilateral are x – 5°, x°, x + 5°
and x + 10°.
The sum of all the four angles in a quadrilateral is 360°.
∴ (x – 5°) + x + (x + 5°) + (x + 10°) = 360°
B C
 1
⇒ ⇒ x 87  °
4x + 10° = 360° ⇒ 4x = 350°=
⇒ Alternate interior angles are equal  2
⇒ AD // BC  1
⇒ ABCD is a trapezium. The smallest angle = x – 5° = 82  °
 2
6. (c)
6. Given statement is true.
A P D
40° 7. Given statement is false.
8. Given statement is false.

40° 9. Given statement is true.


B C 10. Given statement is true.

AD // BC Exercise 2
⇒ ∠APB = 40° (given) ⇒ ∠PBC = 40°
∴ ∠B = 40° + 40° = 80° 1. Given, ABCD is a rhombus and AB = BD
∴ ∠B + ∠C = 180°⇒ 80° + ∠C = 180°
8
D C
⇒ ∠C = 100°.
7. (a) Parallelogram.

8. (b)

P S A B
O In ABCD, AB = BC = CD = AD
[since ABCD is a rhombus]
Q R AB = BD (given)
∴ triangle ABD is an equilateral triangle (∵ AB = BD = AD)
PR = 12 cm ⇒ ∠A = ∠ABD = ∠ADB = 60°
⇒ ∠BAD = 60° and ∠ABC = 120°
12
⇒ PO = = 6 cm [Sum of adjacent angles of a rhombus is 180°]
2 ∴ The angles of the rhombus are 60°, 120°, 60° and 120°
OQ = OP = 6 cm (∵ opposite angles are equal)

9. True 2.
R
D C
10. True

11. False
S Q
12. Equal, parallel

13. Bisect

14. Square. A P B

Given: ABCD is a rhombus where P, Q, R and S are the


Subjective Type Question midpoints of AB, BC, CD and AD respectively.
Exercise 1 To prove: PQRS is a rectangle.
Construction: Join AC and BD.
1. ∠B + ∠D = 360°− (∠A + ∠C) = 360°− 120° = 240° Proof: Consider ΔABC, P and Q are the midpoints of sides AB
1 60° and BC respectively.
2. ∠CAB = (∠BAD) = =°30 As per the midpoint theorem,
2 2
4 Fundamental of Mathematics

1 5. Given KLMN is a rectangle. Also given that P, Q, R and S are


PQ = AC  ... (1) the midpoints of KL, LM, MN and NK.
2 ∴ DKPS, DNRS, DMRQ and DLPQ are congruent triangles
Consider ΔADC, S and R are the midpoints of AD and AC by SAS congruence critarian.
respectively. ∴ ∠SRN = ∠QRM = ∠KPS = 30° (given ∠KPS = 30°)
As per the mid-point theorem, But ∠SRN + ∠SRQ + ∠QRM = 180°
1 ⇒ 30° + ∠SRQ + 30° = 180° ⇒ ∠SRQ = 120°
SR = AC  ... (2)
2 6.
From (1) and (2) we can say that PQ || RS and PQ = RS. D C
One pair of opposite sides are equal and parallel. P
∴ PQRS is a parallelogram.
Thus, PQ || RS and PS || QR.
A Q B
Consider ΔPBQ
PB = BQ (∵ P and Q are midpoints of AB and BC) Given: ABCD is a parallelogram
∠QPB = ∠BQP (Angles opposite to equal sides are equal) AP ⊥ BD, CQ ⊥ BD.
Consider ΔAPS and ΔCQR, In ∆DQC and ∆APB
AP = CQ (Given) AB = CD [Opposite sides of a parallelogram]
AS = CR (Given) ∠QDC = ∠ABP  [Alternate angles]
PS = QR (PQRS is a parallelogram) ∠APB = ∠CQD = 90°
By SSS congruence Property, ΔAPS ≅ ΔCQR ∆APB ≅ ∆CQD (AAS congruency)
∠APS = ∠CQR (By CPCT) AP = CQ (CPCT)
Consider the line AB
7. Given, ABCD is a rectangle,
∠APS +∠SPQ + ∠QPB = 180° (linear pair) ... (1) P, Q, R and S are the midpoints of
Consider the line BC
AB, BC, CD and AD respectively.
∠PQB +∠PQR + ∠RQC = 180° (linear pair) ... (2)
R
From (1) and (2); D C
∠APS +∠SPQ + ∠QPB = ∠PQB +∠PQR + ∠RQC
⇒ ∠SPQ = ∠PQR S Q
But we know, PS || QR and PQ is a transversal
∠SPQ + ∠PQR = 180°
A B
⇒ ∠SPQ = ∠PQR = 90° P
∴ PQRS is a rectangle.
Join A to C and B to D.
3. Given, P, Q, R and S are the midpoints of AB, BC, CD and AD In ∆ADC, S is the midpoint of AD and R is the
respectively. midpoint of DC.
1
R ∴ SR || AC and SR =   AC
D C 2
1 
S Q Similarly, PQ =  AC  and PQ || AC
2 
∴ SR = PQ and SR || PQ
A B  and P are the midpoints of the sides AB and AD of
S
P
∆ABD.
Join A to C 1 
∴ SR =  BD  and SP || BD
In triangle AOC 2 
S is the midpoint of AD and R is the midpoint of DC 1
1 Similarly RS =   BD and RQ || BD
∴ SR || AC and SR =   AC  ... (1) 2
2 As AC = BD (diagonals are equal in rectangle)
P is the midpoint of AB and Q is the midpoint of BC.
1 1
  AC =   BD
2 2
1
∴ PQ =   AC and PQ || AC ... (2) ∴ PQ = QR = RS = SP
2 Hence quadrilateral PQRS is a rhombus.
From (1) and (2), SR || PQ and SR = PQ.
8. In ∆QRS, 3d° + 3d° + 4d° = 180°
∴ quadrilateral PQRS is a parallelogram.
⇒ 10d° = 180° ⇒ d° = 18°
4. MN || OP ∴ ∠QRS = 4d° = 4(18) = 72° ⇒ b° = 4d° = 72°
∠NPO = ∠PNM (alternate angles) (∵opposite angles of a parallelogram)
⇒ ∠PNM = 40° (∵ ∠NPO = 40°) 9. ∠P + ∠Q + ∠R + ∠S = 360°
∠OQN = ∠QMN + ∠QNB (an exterior angle of ∆QMN) ⇒ (x + 1)° + (x + 3)° + (x + 5)° + (x + 7)°= (4x + 16)°
= 30° + 40° = 70°
(4x + 16)° = 360°  ... (1)
Answer and Explanations 5

⇒ x = 86° In ΔDFC and ΔBFP ; CF = FB (given)


∠P = (x + 1)° = 87° ∠DFC = ∠BFP (vertically opposite angles)
∠Q = (x + 3)° = 89° ∠FDC = ∠FPB (alternate angles)
∠R = (x + 5)° = 91° By ASA congruence property
∠S = (x + 7)° = 93° ΔDFC ≅ ΔPFB ⇒ DC = PB and DF = EP (CPCT)
The required sum = 87° + 93° = 180° In ΔADP
E is the midpoint of AD and F is the midpoint of DP.
10. A
∴ EF || AB || AP
1
By the midpoint theorem, in ∆ADP, EF =   ( AP )
2
D 1 1
=   ( AB + BP ) =   ( AB + DC) (∵ DC = BP)
20° 110° 30° 2 2
B 1
C =EF   ( AB + DC) .
2
Reflex ∠BDC = 360°− 110° = 250°
3. Given,  || m
The sum of the angles in a quadrilateral = 360°
A. B and C, D are the points on  and m respectively as shown
∴ ∠A + ∠B + ∠C + Ref ∠D = 3600
in the figure.
∠A + 20° + 30° + 250° = 360°
RM and RN are the bisectors of ∠ARS and ∠BRS.
∠A = 360°− 300° = 60°
SM and SN are the bisectors of ∠RSC and ∠RSD.
Exercise 3 P
1. ABCD is a rhombus R B
A
∴ AB = BC = CD = AD ℓ
D C
M
N
C D
m
O S 8
A B
Q
In ∆ABD and ∆CBD
AB = BC (given) ; AD = CD (given) ∠ARS = ∠RSD (alternate interior angles)
BD is common ∠BRS = ∠RSC (alternate interior angles)
∆ABD ≅ ∆CBD 1 1 1 1
⇒ ( ARS=) (∠RSD) and (∠BRS) =(∠RSC)
(SSS congruence property) 2 2 2 2
∴ ∠ABD = ∠CBD (CPCT) ⇒ ∠MRS = ∠NSR and ∠NRS = ∠MSR
Now, in ∆ABO and ∆CBO and RS is common side.
AB = BC (given) ∆MRS ≅ ∆NSR [ASA congruence]
BO is common ∴ RN = SM and RM = SN  ... (1)
∠ABO = ∠CBO (by proof) ∠ARS +∠BRS = 180° (linear pair)
∆ABO ≅ ∆CBO (SAS congruence property) 1 1
AO = OC and ∠AOB = ∠COB (CPCT)   ∠ARS +   ∠BRS =90°
∠AOB + ∠COB = 180° (linear pair) 2 2
2∠AOB = 180°, ∠AOB = 90° ∠MRN = 90° ... (2)
Similarly ∆ADO ≅ ∆CDO From (1) and (2) quadrilateral RSNM is a rectangle.
BO = OD and ∠AOD = ∠COD = 90°
4. Given, ABCD is a parallelogram,
∴ AC and BD bisects each other at right angles.
E is the mid point of AD
2. Given: AB || DC F is the mid point of BC
D C D C

E F
O E Q
F
A P P
B
A B
E is the midpoint of AD
F is the midpoint of BC AD = BC and AD || BC
Join D to B. DB intersects EF at O. [Opposite sides of a parallelogram]
Join DF and extend to meet extended AB at P.
6 Fundamental of Mathematics

1 1 HOTS


  AD =   BC
2 2 1. Let the angles be k + 2k + 3k + 4k
∴ DE = BF and DE || BF Sum of the angles of a quadrilateral = 360°
∴ Quadrilateral EBFD is a parallelogram ⇒ k + 2k + 3k + 4k = 360°
⇒ 10k = 360
BE || DF (opposite sides of a parallelogram)
360
Now in DCPB, F is the midpoint of BC [given] ⇒ =k = 36
10
FQ || BP (by proof)
∴ The angles of the quadrilateral are
∴ Q is the midpoint of PC
36°, 2 × 36°, 3 × 36°, 4 × 36°,
⇒ PQ = CQ  ... (1) 36°, 72°, 108°, 144°.
Similarly in triangle ADQ, AP = PQ ... (2) 2. Let ABCD be the quadrilateral (∠A, ∠C), (∠B, ∠D) are
From (1) and (2), AP = PQ = CQ. opposite angles (∠A, ∠D), (∠D, ∠C), (∠C, ∠B), (∠B, ∠A)
are adjacent angles.
5. Let ABCD be a quadrilateral.
P, Q, R and S are the mid points of AB, BC, CD and AD A B
respectively.

D
S R
D C
A C
Given, Adjacent angles are in the ratio 7:11
Let ∠A be 7k and ∠D be 11k
P Q ⇒ ∠C = 7k, ∠B = 11k
B Sum of the angles of a quadrilateral = 360°
∴ 7k + 11k + 7k + 11k = 360°
⇒ 36k = 360°
360°
(i) Join P, Q, R and S to form a quadrilateral. ⇒ k= = 10°
(ii) Join A to C. 60°
Proof: In ∆ADC, ∴ 7k = 7 × 10 = 70°, 11k = 11 × 10 = 110°
S is the midpoint of AD and Hence, the required angles of the quadrilateral are
R is the midpoint of DC. 70°, 110°, 70°, 110°.
Here, the adjacent angles are supplementary and the
1 opposite angles are equal.
SR =   AC and SR || AC  ... (1)
2 ∴ ABCD is a parallelogram.
1 3. ar(ABCD) = 48 cm2
Similarly in ∆ABC, PQ =   AC and PQ || AC  ... (2)
2 A B
From (1) and (2) PQRS is a parallelogram.
PR and SQ are the diagonals of a parallelogram.
∴ PR and QS bisect each other. O
6. Given that DEF is a right triangle and ∠DEF = 90°
Complete a rectangle DEFQD and join EQ. D C
Now, DF, EQ are the diagonals of a rectangle which intersect Now BD is the diagonal of the parallelogram.
at P (since P is the midpoint of DF). In a rectangle, diagonals 1
bisect each other and are equal. ∴ ar ( ∆ADB ) =
ar ( ABCD ) (∵ The diagonal divides
2
1 1
∴ DF = EQ and DF = EQ . the parallelogram into the triangles of equal area.)
2 2 1
∴ DP = EP. ∴ ar ( ∆ADB ) = × 48 =24 cm 2 In a parallelogram, the
2
diagonals bisect each other.
D Q O is the mid points of BD
In ΔADB, AO is the median onto BD.
⇒ ar(ΔAOD) = ar(ΔAOB) (∵ Median divides the triangle
into two triangles of equal area)
P ⇒ ar(ΔAOD) + ar(ΔAOB) = ar(ΔADB)
⇒ 2 ar(ΔAOB) = 24 cm2
E F 24
∴ ar(ΔAOB) = = 12 cm 2
2
Answer and Explanations 7

4. In ΔAOD and ΔBOC, AD // BC and AC is the transversal. ∴ ∠DAB + ∠BCD = 108° + 72° = 180°
Alternate method:
A B ∠EBC = 72°
⇒ ∠ABC = 180 – 72° = 108°
⇒ ∠DAB = 108° (base angles are equal in an isosceles
O trapezium)
⇒ ∠BCD = 180° – ∠ABC = 180° – 108° = 72°
∴ ∠DAB + ∠BCD = 108° + 72° = 180°.
D C
7. Given, ABCD is a parallelogram in which the diagonals AC
∠DAO = ∠BCO (Interior alternate angles) and BD bisect each other at right angles.
AD // BC and BD is the transversal A
∠ADO = ∠CBO (Interior alternate angles)
AD = BC (opposite sides of a parallelogram.)
∴ ΔAOD ≅ ΔBOC (A.S.A)
5. In rhombus ABCD, BD is the diagonal.
B D
A E O

C
B D
∴ BO = OD, AO = OC
In ΔAOB and ΔAOD
BO = OD (given)
AO = AO (common side)
C
∠AOB = ∠AOD = 90° (given)
In ΔABD and ΔBCD, ∠A = ∠C (opposite angles of a rhombus) ∴ ΔAOB ≅ ΔAOD (S.A.S) ... (1) 8
AB = BC In ΔAOB and ΔBOC
AD = DC (All sides of a rhombus are equal) AO = OC (given)
∴ ΔABD ≅ ΔBCD (S.A.S) ... (1) BO = BO (common side)
AD is the diagonal of the parallelogram ABDE ∠BOC = ∠BOA = 90° (given)
AE // BD and AD is the transversal
∠EAD = ∠ADB (interior alternate angles) ∴ ΔBOC ≅ ΔAOB (by S.A.S) ... (2)
∠EDA = ∠DAB (interior alternate angles) Similarly, ΔBOC ≅ ΔCOD  ... (3)
AD = AD (Common side) From (1), (2) and (3)
∴ ΔABD ≅ ΔADE (A.S.A) ... (2) ΔAOB ≅ ΔBOC ≅ ΔCOD ≅ ΔAOD
From (1) and (2), we get ∴ AB = BC = CD = DA (C.p.c.t)
ΔBCD ≅ ΔADE ABCD is a parallelogram in which all the sides are equal.
6. ABCD is a trapezium AB // CD and AD = BC ∴ ABCD is a rhombus.

B 8. Let the smallest angle be x°


A
E A B

D C

∴ ABCD is an isosceles trapezium. x°


AECD is a parallelogram
D C
AD = EC (Opposite sides of a parallelogram)
But AD = BC (given). ∠C = x° ⇒ ∠A = x° (∴ Opposite angles of parallelogram)
⇒ BC = EC ⇒ ∠D = ∠B = 2x° + 24
∴ ΔBEC is an isosceles triangle. ∴ x° + x° + 2x° + 24 + 2x° + 24 = 360°
⇒ ∠EBC = ∠CEC = 72° ⇒ 6x° + 48 = 360°
⇒ ∠BEC = ∠AEC = 72° ⇒ 6x = 360 – 48 = 312
∴ ∠ADC = 72° (Opposite angles of a parallelogram) 312
In an isosceles trapezium, the base angles are equal. ⇒ x= = 52°
6
∴ ∠ADC = ∠BCD = 72°
∠DAB = 180 – ∠ADC (∵ adjacent angles of a ⇒ 2x + 24 = 2 × 52 + 24° =128°
parllelogram are supplementary) ∴  he angles of the parallelogram are 52°, 128°, 52° and
T
= 180 – 72° = 108° 128°.
8 Fundamental of Mathematics

9. In a rhombus, the diagonals bisect the angles at the vertex. DE = AF


∴ DEFA is a parallelogram.
A ∴ ΔDFE ≅ ΔADF ... (2)
Similarly, ΔDFE ≅ ΔCFE ... (3)
36° From (1), (2) and (3), we get
ΔADF ≅ ΔBDE ≅ ΔCEF.
B D 12. 2BD = AB
O
A

C
D E
∠OAB = 36° ⇒ ∠OAC = 36°
In ΔAOB, and ∠ABO = 180° – (90° + 36°)
= 180° – 126° = 54°. B C
Now, ∠ABO = 54° ⇒ ∠OBD = 54°
∠A = ∠D and ∠B = ∠C (opposite angles) AB
∠A = ∠OAB + ∠OAC = 36° + 36° = 72° ⇒ BD =
2
∴ ∠D = 72°
2CE = AC
∠B = ∠ABO + ∠OBD = 54° + 54° = 108°
AC
∴ ∠D = 108° ⇒ CE =
Hence, the required angles of the rhombus are 72°, 54°, 72° 2
and 54°. ∴ D and E are mid points of AB and AC respectively.
10. In a rhombus, the diagonals bisect each other at right angles AB = 2BD = 2 × 3 = 6 cm
AC = 2CE = 2 × 4 = 8 cm
A Now AB + AC > BC
6 + 8 > BC
BC < 14
∴ Maximum length of BC = 13 cm
B D 13
O ⇒ Maximum length of DE = = 6.5cm .
2
13. In the given figure, BA = AD and BC = CE

D
C
∴ AO = OC, OB = OD 4 cm
But AO = OB given A
∴ AO = OB = OD = OC
⇒ AO + OC = OB + OD 4 cm
∴ AC = BD
In a rhombus, if the diagonals are equal then it is a square B E
8 cm C 8 cm
∴ ABCD is a square.
∴ A and C are the mid points of BD and BE respectively
11. D, F, are the mid points of AB an AC
1
A ∴ AC // DE, AC = DE
2
⇒ 2AC = DE
∴ DE = 2 × 5 = 10 cm
D F ∴ Perimeter of ΔBDE = BD + DE + BE
= 8 + 10 + 16
= 34 cm.
B C 14. Given, ABCD is a rectangle
E
F
1
∴ DF // BC, DF = BC  (∵ By mid point theorem)
2 A B
i.e., DF = BE
∴ DFEB is a parallelogram
∴ ΔDFE ≅ ΔBDE ... (1)
1 D C E
Similarly, DE // AC and DE = AC (∵ By mnid point
2
BD = 8 cm, CD = 6 cm
theorem)
Answer and Explanations 9

In ΔBCD, 1
BD2 = CD2 + BC2 Now, HG = FE = BD
2
⇒ BC = BD 2 − CD 2 1
∴ BC = 64 − 36 = 28 HE = GF = AC
2
= 2 7 cm But AC = BD (diagonals of an isosceles trapezium).
In ΔDFE, ∴ HG = HE = FE = GF
BD = BF, DC = CE ∴ DEFG is a rhombus.
∴ B and C are the mid points of DF and DE respectively. 17. In ΔABD
1 A
∴ BC = FE ⇒ FE = 2BC = 2 × 2 7
2
= 4 7 cm P
D G
∴ Perimeter of BCEF = BC + BF + FE + CE
= 2 7 +8+ 4 7 +6
B Q S D
(
= 14 + 6 7 =2 7 + 3 7 cm . ) O

15. Given, Perimeter of ABCD = 40 cm E R F


A

C
E
1
DG // BD, DG = BD (by mid point theorem)
H 2
B O D
F In ΔBCD
1
G EF // BD, EF = BD
2
∴ DG // EF and DG = EF 8
C ∴ DEFG is a parallelogram
DG // BD
40  ∠QOP = 90° (∵ Diagonals bisect each other at right
∴ AB = BC = CD = AD = = 10 cm .
4 angles)
1 ∴ ∠DPO = 90° (interior angles on the same side of the
In ΔABO, EF = AB (∵ By mid point theorem) transversal)
2 DE // AC
1 ⇒ ∠DPO + ∠QDP = 180° (interior angles on the same side
= × 10 =
5cm
2 of the transversal)
1 ⇒ ∠QDP = 180° – ∠DPO
In ΔBCO, FG = × BC =5cm (∵ By mid point = 180° – 90° = 90°
2 ∴ DEFG is a parallelogram with one angle equal to 90°.
theorem) ∴ DEFG is a rectangle.
Similarly, GH = 5 cm and EH = 5 cm
∴ Perimeter of EFGH = 5 × 4 = 20 cm. 18. ABCD is square.

16. In ΔADB, D and G are mid points of AB and AD respectively. D


A B
A H B Q
P
G E G E
O
S
D F C R
D C
F
1
∴ DG // BD, DG = BD (∵ By mid point theorem)
2 D, E, F, G are the mid points of AB, BC, CD, DA respectively
I n ΔBDC, F, E are the mid points of DC and BC 1 1
DG // BD, DG = BD FE // BD, FE = BD
respectively. 2 2
1 ∴ DG = FE, DG // FE
∴ FE // BD, FE = BD ∴ HG // FE, DG = FE
2 ∴ DEFG is a parallelogram
Similarly, HE // GF and DE = FG 1 1
DG = FE = BD Similarly, DE = FG = BC
∴ DEFG is a parallelogram 2 2
10 Fundamental of Mathematics

But AC = BD (diagonals of a square are equal) 21. C


∴ DE = EF = FG = GD
∠POQ + ∠DQO = 180° (∴ Interior angles on the same
side of the transversal)
But ∠POQ = 90° (∵ Diagonals bisect at right angles)
∴ ∠DQO = 90° 2 E
F 1
∠PDQ + ∠DQO = 180° (interior angles on the same side
of the transversal)
∴ ∠PQQ = 180° – ∠DQO = 180° – 90° = 90°
∴ DEFG is a square. 3 4
A D B
19. ABCD is a rectangle.
In, ΔABC ∠A = 90° and
H Since, ABC is an isosceles triangle,
A B
⇒ AB = AC  ... (i)
Let ADEF be the square inscribed in an isosceles triangle
G E ABC.
Then, we have,
D C AD = AF = EF = ED  ... (ii)
F
Subtracting equation (ii) from (i),
AB – AD = AC – AF
1 BD = CF
HG // BD, HG = BD (∵ By mid point theorem)
2 Let ΔCFE and ΔEDB,
1 BD = CF
FE // BD, FE = BD (∵ By mid point theorem)
2 DE = EF
∠CFE = ∠EDB = 90° (side of a square)
1
∴ HG // FE, HG = FE = BD ΔCEF ~ ΔEDB (By SAS congruence rule)
2 Hence, CE = BE
∴ EFGH is a parallelogram. Therefore, vertex E of the square bisect the hypotenuse
1 BC.
HE //AC, HE = AC (∵ By mid point theorem)
2 22. F
1
FG //AC, FG = AC (∵ By mid point theorem)
2
1 1
Now, HG = FE = BD HE = FG = AC But AC = BD
2 2 D
E
C
(diagonals of a rectangle are equal).
∴ HG = HE = FE = FG
6 cm
∴ EFGH is a rhombus.
20. AB = BC = AC = 6 cm

A A B
ABCD is a parallelogram
Given:
D F AB = 10 cm
AD = 6 cm.
The bisector of ∠A meets DC at E.
B C AE and BC produced meet at F.
E Since, AF bisects ∠A,
We get,
D, E, F are the mid points of AB, BC, AC respectively.
1 1 ∠BAE = ∠EAD  ... (1)
∴ DF =
BC = × 6 = 3cm (∵ By mid point theorem) ∠EAD = ∠EFB ... (2) [Alternate angles]
2 2
From (1) and (2),
1 1 ∠BAE = ∠EFB
FE = AB = × 6 = 3cm (∵ By mid point theorem)
2 2 Since sides opposite to equal angles are equal,
1 1 BF = AB {Side opposite to equal angles are equal}
BE = BC = × 6 = 3cm Here, AB = 10 cm {Given}
2 2
⇒ BF = 10 cm
1 1
DB = AB =
 × 6 = 3cm ⇒ BC + CF = 10 cm and BC = AD = 6 cm
2 2 6 cm + CF = 10 cm
∴ Perimeter of BEFD = BE + FE + DF + DB ⇒ CF = (10 – 6) cm = 4 cm
= 3 + 3 + 3 + 3 = 12 cm. ⇒ CF = 4 cm
Answer and Explanations 11

23.

30 cm
D C 1 1
= (AB + BP) = (AB + DC)
40 cm 2 2
X Y
1
= (50 + 30)
2
A 50 cm B P
= 40 cm
Since X is the midpoint of AD and BC respectively
ABCD is a trapezium with AB || DC; AB = 50 cm DC = 30 cm \ Trapezium DCYX and ABYX are of same height, h, cm
Construction:  Join DY and produce it to meet AB produced at P. Now
In ∆BYP and ∆CYD 1
(DC + XY) × h
∠BYP = ∠CYD    (vertically opposite angles) area(DCYX) 2
=
∠DCY = ∠PBY (alternate angles) area(ABYX) 1 (AB × XY) × h
BY = CY (Y is mid point of BC) 2
∆BYP ≅ ∆CYD (by ASA cogence criterion)
30 + 40 70 7
\ DY = YP and DC = BP = = =
⇒ Y is the midpoint of AD 50 + 40 90 9
1 ⇒ 9 ar(DCXY) = 7 ar(XYBA)
∴ XY || AP and XY = AP (by midpoint theorem) 7
2 ⇒ ar(DCXY) = ar(XYBA)
1 9
⇒ XY = AP
2

8
9
Areas of Parallelograms
and Triangles
Answer and Explanations

STEPWISE MARKING SOLUTION Subjective Type Question


Exercise 1
Objective Type Question
1. Since ABCD and ABRS lie on the same base AB and between
1. (c) In rectangle ABEM, same parallel lines AB and DR, ar(parallelogram ABCD) =
AB = EM ... (1) [sides of rectangle] ar(parallelogram ABRS) = 25 sq cm. (1 Mark)
In parallelogram ABCD,
CD = AB  ... (2) 2. Given that, DABC and DABD lie on the same base and lie
Adding, equations (1) and (2), between the same parallel lines. Also given that the area of
DABD = 15 sq cm
AB + CD = EM + AB  ... (i)
\ The area of DABC = 15 sq cm (1 Mark)
{Perpendicular distance between two parallel sides of a
parallelogram is always less than the length of the other 3.
parallel sides.}
BE < BC and AM < AD R T U S
[\ In a right angled triangle, the hypotenuse is greater than
the other side]
add above inequalities both sides we get
SE + AM <BC + AD or BC + AD > BE + AM
On adding AB + CD on both sides, we get
AB + CD + BC + AD > AB + CD + BE + AM P Q
⇒ AB + BC + CD + AD > AB + BE + EM + AM
[∴ CD = AB = EM] Given PQ || RS and the area of parallelogram PQUR is 46
 Perimeter of parallelogram ABCD > perimeter of cm2.
rectangle ABEM ΔPTQ, ΔPUQ and ΔPSQ and parallelogram PQUR are on the
2. (a) AO = OD and AD // BC same base and between the same parallels. (1Mark)
\ Area of ΔPTQ = Area of ΔPUQ = Area of ΔPSQ
A O D 1
= × area of parallelogram PQUR
2
1
= × 46 = 23 cm2. (1 Mark) 9
2
B C 4.
A
⇒ Area of DAOB = Area of DCOD
1
=
2 E
( area of DBOC)
1
= (20) = 10 cm2 B D C
2
3. (a) Area of DMKN = Area of DLKM = 32.5 cm2. In triangle ABC, AD is the median.
BD = DC
4. (b) AD = BC and AD // BC
1
\ Ar (DABD) = Ar (DABC)
A P D 2
E is a point on AD. (1 Mark)
Now, ED is a median of DBEC
\ Ar(DBED) = Ar(DCED)
B C [Median divides a triangle into two triangles of equal area].
 (1 Mark)
Area of D ACD 5. Let ABCD be a parallelogram
= Area of DBPC = 25 cm2 Join BD
5. True InDABD and DCDB
BD is common side
6. True
AB = CD and AD = BC
7. Parallel lines (opposite sides of a parallelogram are equal)
\ DABD ≅ DCDB (SSS property) (1 Mark)
8. AD and BC
Ar (DABD) = AR(DCDB)
2 Fundamental of Mathematics

i.e. BD divides the parallelogram into two triangles of equal \ Ar (D DBC) + Ar (DABD)
area. = Ar (DDBP) + Ar (DADB)
Similarly, AC divides it into two triangles of equal area. ⇒ Ar (quadrilateral ABCD) = Ar(DADP) (11/2 Marks)
 (1 Mark)
4. Given that ABCD and AEFD are parallelogram
Exercise 2 E
1. Given, BD = DC and BE = ED
B P A
A
F

C Q D
B C
E D AEFD is a parallelogram
AF is the diagonal
In DABC, AD is the median. (given) \ EF = AD ... (1)
1 ADQP is also a parallelogram, (11/2 Marks)
\ Ar (DABD) = ar (DABC)  ... (1) AD = PQ  ... (2)
2
From (1) and (2) we get
(Median divides the triangle into two triangles of equal
\ EF = PQ
areas) (11/2 Marks)
Subtracting PF on both the sides,
In DABD, AE is the median
EF – PF = PQ – PF
1
\ Ar (DABE) = Ar (DABD)  ... (2) PE = FQ (11/2 Marks)
2
5. (a) ABCD and ABEF are two parallelograms lying on the
From (1) and (2), Ar(DABE)
1 same base AB and between the parallel lines AB and
= Ar (DABC) (11/2 Marks) CF .
4
\ Ar (// gm ABCD)
2. Given that in the quadrilateral ABCD,
= Ar (// gm ABEF)  ... (1)
Ar(D ADC) = Ar(D ABC)
we know that, Ar(DAXF) + Ar(DBXE)
D C 1
= [Ar (// gm ABEF)] (1 Mark)
Q 2
O 1
⇒ 13.5 = [Ar (// gm ABEF) (∵given)
2
P ⇒ [Ar (// gm ABEF) = 27 sq cm
A B ⇒ [Ar (// gm ABCD) = 27 sq cm
(from 1) (1 Mark)
Let P and Q are on AC such that DP ^ AC and BQ ^ AC. (b) 
PQRS is a parallelogram. QS is the diagonal bisecting
Ar (DADC) = Ar(DABC)
the parallelogram.
1 1 
⇒  2 × DP × AC =2 × BQ × AC  S A R
 
⇒ DP = BQ (1 /2 Marks)1

In DDPO and DBQO,


∠DPO = ∠BQO = 90o ⇒ DP = BQ
B
∠DOP = ∠BOQ (vertically opposite angles)
DPOD ≅ DBOQ [AAS property]
DO = OB (CPCT) (11/2 Marks)
P Q
3. Given, ABCD is a quadrilateral and DB || CP. Join D and P
Area of RSQ = 2(Area of D ASQ)
D C
= 40 sq cm. (1 Mark)
1
Area of DARB = [Area of DSRQ]
4
40
= = 10 sq cm. (1 Mark)
4
A B P
Exercise 3
Ar (DDBC) = Ar (DDBP)  ... (1) 1. Given: AD = DB, BE = CE and AF = CF
[Triangles on the same base DB and between same parallel 1
lines DB and CP] (11/2 Marks) R.T.P: (DDEF) = A(DDEF)
4
Answer and Explanations 3

A AQ and DC are the diagonals.


DP = PC
\ Ar(D BCP) = Ar(DDPA) ... (2)(11/2 Marks)
D F and Ar(D DPA) = Ar(D DPQ)  ... (3)
From (2) and (3), we get
Ar (DBCP) = Ar (DDPQ) (11/2 Marks)
B E C
PROBLEMS FOR ADDITIONAL PRACTICE
D is the mid point of AB and E is the mid point of BC
\ DE || AC
(∵ Line segment joining the midpoints of any two sides of Objective Type Question
a triangle is parallel to the third side and half of third side). 1. (b) Area of DKLN = Area of DKLM = 42.5 cm2
Similarly EF || AB and DF || BC K L
\ Ar(DDBE) = Ar(DDEF) (2 Marks)
(∵ DBEF is a parallelogram)  ... (1)
[Diagonal of a parallelogram divides the parallelogram
into the triangles of equal area]
Similarly, Ar(DDEF) = Ar(DADF) (2 Marks) N M
= Ar(DCEF)  ... (2)
From (1) and (2) (\ Triangles are on the same base KL and between the same
1 parallel lines KL and NM)
Ar(DDEF) = ar(DABC) (1 Mark]
4 2. (c) Area of the parallelogram ABCD
2. Given, AD = DB = 2 (area of DABC) = 2 × 20 = 40 cm2
A 3. (d) AP = PB and AB // DC

P A D
D
P Q
B C
Q B C
Join D to C. ⇒ area of DAPQ = area of D BPQ = 50 cm2 9
In DABC,DC is the median. ⇒ area of DABQ = 50 + 50 = 100 cm2
1 Area of DCPD = area of DABQ = 100 cm2
\ Ar(DDBC) = (ABC) ... (1)  (11/2 Marks)
2 4. (b) AD = BP = PC and AD // BC
[Median divides triangle into two triangles of equal areas] ⇒ Now ABPD and APCD are two parallelograms
Ar (DDQC) = Ar(DDQP) ... (2) Area of the parallelogram ABPD = 2 (area of DPCD)
(11/2 Marks) = 2 × 25 = 50 cm2
[Triangles on the same base and between same parallel lines]
A D
Ar(DDQC) + Ar(DDBQ) = Ar(DDQP) + Ar(DDBQ)
Ar (D DBC) = Ar (D PBQ)  ... (3)
(11/2 Marks)
From (3) and (1)
1 B C
Ar(DBPQ) = ar (DABC). (1 Mark) P
2
\ Area of the trapezium ABCD = Area of the parallelogram
3. Given that ABCD is a parallelogram and ⇒ BC = CQ
ABPD + area of DPCD = 50 + 25 = 75 cm2
Join A and C.
A B 5. (c) \ Area of rectangle ABCD = 4 × Area of D AOB
= 4 × 16.5 = 66
6. True
7. False
D C 8. True
P
9. True
1
10. ar ( ABCD )
Q 2
Join A and C. 11. ar(∆ACE)
AD = BC and BC = CQ 12. 1 : 2
⇒ AD = CQ and AD || CQ (2 Marks)
13 Congruent
\ Quadrilateral ADQC is parallelogram.
4 Fundamental of Mathematics

Let P and Q be the midpoints


Subjective Type Question
⇒ AP = PB and DQ = QC.
Exercise 1 ⇒ AP = PB = DQ = QC
AB || DC
1. Since both the parallelograms ABCD and ABRS lie on the ⇒ APQD and PBCQ are parallelograms
same base AB and between same parallel lines AB and DR,  The bases of APQD and PBCQ are equal and the
ar(parallelogram ABRS) parallelograms lie between the parallelograms AB and
= ar (parallelogram ABCD) = 100 sq cm CD.
2. Given, XY || BC, AB || CN and AC || BM \ Ar(parallelogram APQD) = Ar(parallelogram PBCQ)

A 6. Since both the parallelograms lie on the same base AB and


between the parallel lines AB and CS, ar(parallelogram ABRS)
= ar(parallelogram ABCD) = 40 sq cm.

M P Q N 7. DAEB and rectangle ABCD lie on the same base AB and lie
X Y between same parallel lines.
\ Area of rectangle ABCD = 2 × ar(DAEB)
= 2 × 10 = 20 sq cm.
8. Given that, DABC and DABD lie on the same base and lie
between the same parallel lines.
B C Also given, that the area of DABC = 20 sq cm
\ The area of DABD = 20 sq cm.
\ Quadrilaterals BCNP, BCQM are two parallelograms
9. Given, ABCD is a parallelogram
[∵ BC || PN, PB || NC, BC || MQ, MB || QC]
\Ar (parallelogram BCNP) D Q C
= Ar(parallelogram BCQM) ... (1)
[parallelograms on the same base and between same
parallel lines are equal in area] P
1
Ar (D AMB) = Ar (parallelogram BCQM) A B
2
[BM || AC] 1
Ar(DAPD) = Ar(parallelogram ABCD)  ... (1)
1 2
Similarly Ar (DANC) = Ar (parallelogram BCNP]
2 1
 ... (3) Ar(DAQB) = Ar(parallelogram ABCD)  ... (2)
2
[NC || AB] [If a triangle and the parallelogram lie on the same base and
From (1), (2) and (3) between same parallel lines, then the area of triangle is equal to
Ar (DAMB) = Ar (DANC) half the area of parallelogram]
From (1) and (2), Ar(DAPD) = Ar(DAQB)
3. Given that, DABC and DABD lie on the same base and lie \ Ar(DAQB) = 64 sq cm.
between the same parallel lines.
10. Given that in the trapezium ABCD, AB || DC.
Also given that the area of DABC = 10 sq cm
\ The area of DABD = 10 sq cm. A B

4. Area of parallelogram taking AB as base


= AB × DP = 10 × 5
= 50 sq cm  ... (1)
Area of parallelogram taking BC as base = BC × DQ D P C E
\ BC × DQ = 50 (from [1]) Drop BP ^ DC.
8(DQ) = 50 1
⇒ DQ = 50/8 = 6.25 cm Ar(DABD) = AB × BP ... (1)
2
5. Given: ABCD is a parallelogram 1
Ar(DBCE) = × CE × BP ... (2)
2
D Q C 1 1
Since AB = CE, AB × BP = × CE × BP
2 2
\ Ar (D ABD) = Ar (D BCE)

Exercise 2
A B
P
1. Given, Ar(DAOD) = Ar(DBOC)
Ar(DAOB) + Ar (DAOD) = Ar(DBOC) + Ar(DAOB)
Answer and Explanations 5

D C Given that PQ = AB = 5 cm
O As we know that the triangles having equal bases and lying
between the same parallels are equal in area.
1
A B The area of triangle PQR = × (base PQ) × (altitude PR)
2
⇒ Ar(DADB) = Ar(DACB) 1
The perpendicular distance between DC and AB is
\  = × 6 × 6 = 18 sq. cm
remains constant. 2
\ AB || DC = area of triangle ABC
\ ABCD is a trapezium. Area of rectangle PBCR = 14 × 6 = 84 cm2
2. Let P, Q, R and S be the midpoints AB, BC, CD and AD of the
Area of trapezium QACR = 84 – (18 + 18)
parallelogram ABCD. = 84 – 36 = 48 cm2
D P C Alternative method
QA = 14 – 6 – 6 = 2 cm and PB = RC = 14 cm.
1
Area of trapezium QACR = × PR × ( QA + RC )
S Q 2
1
× 6 × ( 2 + 14 ) = 48 cm2.
2
A R B 5. As we know that, the area of the triangle PTR is half of area
Join P and R. of the parallelogram PRSA (∵They lie on the same base and
between the same parallels.)
ARPD and RBCP form parallelograms.
1 A T S
Ar(DPSR) = Ar(parallelogram ARPD)  ... (1)
2
1
Ar(DPQR) = Ar(parallelogram RBCP)  ... (2)
2
By adding (1) and (2), we get
Area of the quadrilateral PQRS R
1 P Q
= Ar(parallelogram ABCD)
2 Given that, ∠PRS = 90°
\ The area of the rectangle PRSA 9
3. Given, ABCD and BPRS are the parallelograms AS || CP
D C = (PR) (SR) = 11 × (7.7) = 84.7 sq. cm
1
The area of the triangle PTR = (area of the rectangle)
2
A P 1
B = × (84.7) = 42.35 sq. cm
2
Similarly area of the triangle AQS = 42.35 sq. cm
S R
6. Given, AB || DC
Join A to C and S to P. D C
Ar (DACS) = Ar (DAPS)
O
[Triangles on the same base AS and between same parallel to
lines AS || CP]
A B
Ar(DACS) - Ar(DABS) = AR(DAPS) - Ar(DABS)
⇒ Ar(DABC) = Ar(DPBS) ⇒ 2Ar(DABC) = 2Ar(DPBS) Ar(DADB) = Ar(DACB)  ... (1)
Ar (parallelogram ABCD) = Ar (parallelogram BPRS) [Triangles on the same base and between same parallel lines
[Diagonal divides the parallelogram into two triangles of equal are equal in area]
area] \ Ar(DADB) – Ar(DAOB) = Ar(D ACB) – Ar (DAOB)
4. Two triangles PQR and GHI are isosceles. So, ⇒ Ar(DAOD) = Ar(DBOC)
R C 7. Given, that AD, BE, and CF are three
Medians meet at G(centroid).
A

F G E
P Q A B

PQ = PR and AB = BC B D C
6 Fundamental of Mathematics

AD is the median of the triangle ABC. ⇒ Ar(DDPC) + Ar(DAPB)


\ Ar (DABD) = Ar (DACD) ... (1) 1
= ar (parallelogram ABCD) ... (4)
GD is the median of DGBC 2
\ Ar (DGBD) = Ar (DGDC) ... (2) From (3) and (4)
\ Ar (DBGC) = 2Ar(DBGD). A(DAPD) + Ar(DBPC) = Ar(DDPC) + Ar(DAPB)
8. Let ABCD be a parallelogram 2. Given:
Join BD Let ABCD be a parallelogram and ABEF be a rectangle.
InDABD and DCDB
BD is common F D E C
side AB = CD and
AD = BC
(opposite sides of a parallelogram are equal)
\ DABD ≅ DCDB (SSS property)
Ar (DABD) = Ar(DCDB)
i.e. BD divides the parallelogram into two triangles of A B
equal area.
Similarly, AC divides it into two triangles of equal area. \ AB || CF
In the above figure, the rectangle and the parallelogram lie
9. In the parallelogram ABCD, if G is any point in the interior of
on the same base and between same parallel lines.
parallelogram, then area of triangle DGC + area of the triangle
\ Ar(rectangle ABEF = Ar(Parallelogram ABCD)
A B AD > AF and BC > BE.
[Perpendicular distance between the parallel lines is the
shortest distance]
D C AB + AD > AB + AF
⇒ 2(AB + AD) > 2(AB + AF)
AGB = Area of the triangle DGA + Area of triangle CGB = Perimeter (Parallelogram ABCD
30 cm2. > Perimeter of rectangle ABEF.
Exercise 3 3. Given, AB || DC and XY || AC

1. Given: ABCD is a parallelogram D C

D R C Y

P
A X B

Join A to Y
A B
S Ar (DADX) = Ar (DACX)  ... (1)
(Triangles on the same base and between same parallel lines
Through P draw a parallel line to AD which meets DC and AB AB and CD).
at R and S respectively. Ar(DACX) = Ar(DACY) ... (2)
In quadrilateral ADRS (Triangles are on he same base AC and lie between same
AD || RS [By construction] parallel lines AC and XY.
AS || DR [∵ AB || DC ] \ From (1) and (2) we get,
ar (DADX) = AR (DACY)
\ ASRD is a parallelogram
1 4. Given that, AD, BE and CF (medians of the triangle) meet at G.
\ Ar(DAPD) = Ar(parallelogram ASRD)  ... (1)
2 A
1
Similarly, Ar(BPC) = Ar(parallelogram RSBC)  ... (2)
2 F G E
[If a triangle and parallelogram lie on the same base and
between same parallel lines, then the area of triangle is
equal to half the area of the parallelogram]
By adding (1) and (2), B D C
1 In triangle ABC, AD is the median
Ar(DAPD) + Ar(DBPC) = [Ar(parallelogram ASRD)]
2 Ar (DABD) = Ar (DACD)  ... (1)
1 (Median divides the triangle into two triangles of equal area)
+ [ArCRSB] \ Clearly GD is the median
2
Ar(BGD)
1 = Ar (DCGD) ... (2)
= Ar (parallelogram ABCD]  ... (3)
2
Answer and Explanations 7

Subtracting (2) from (1) we get Given,


Ar (DABG) = Ar(DAGC) ... (3) PS = 5 cm; Radius of circle = 13 cm
Similarly Ar (DBGC) = Ar(DAGC) ... (4) By using Pythagoras theorem
From (3) and (4), we get
(13) − ( 5) =
2 2
1 SR= 169 − 25= 144= 12 cm
Ar (DABG) = Ar (DBGC) = Ar(DAGC) = (DABC)
3 A is any point on PQ
5. Given that ABCD is a rectangle and ABEF is a parallelogram PA<PQ
and also given that, the perimeter of ABCD is equal to the 1
perimeter of ABEF. ar (DPQR) = × PQ × QR
2
D C
F E 1
= ×12 × 5 = 30 cm2
2
Let PA<PQ,
A ar(DPAS) < ar(DPQR)
P B
ar(DPAS) < 30 cm2
Drop: FP ^ AB Let PA=PQ,
AD = AF (∵ the perimeters are equal) 1
ar (DPAS) = × PQ × PS
\ FP < AF ⇒ FP < AD. 2
AB × AD > AB × FP 1
= ×12 × 5 = 30 cm2
Ar(rectangle ABCD) > Ar(parallelogram ABEF) 2
2.
6. Given that LM || CB, PQ || AB and
ABCD is a parallelogram
As CQ || LO and CL || OQ,
OQCL is a parallelogram.
As PO || AM and PA || OM,
AMOP is a parallelogram.
L
D C

Let the diagonals AC and BD intersect at O. 9


P Q
O ar (∆ADB) = ar (∆BDC) (∵ diagonal divides the parallelogram
A B into two triangles of equal area.)
M
⇒ ar (∆AOD) + ar (∆AOB) = ar (∆COD) + ar (∆BOC)
Ar (DABC) = Ar (DADC)  ... (1) In ∆ADB, ‘O’ is the mid point of BD (∵ diagonals bisect
[AC is the diagonal of parallelogram ABCD] each other)
Ar (DOQC) = Ar (DLOC)  ... (2) ∴ AO is the median ∆ADB.
(∵ OC is the diagonal of the parallelogram OQCL) Similarly, in ∆BDC, CO is the median.
Ar (DAOP) = Ar (DAOM) ... (3) ∵ AO is the median, we get
From (1), (2) and (3) ar (∆AOD) = ar (∆AOB) ... (1)
Ar (parallelogram DLOP) = Ar (parallelogram MOQB) ∵ CO is the median, we get
ar (∆COD) = ar (∆BOC) ... (2)
HOTS Now ar(∆AOD) + ar(∆AOB) = ar (∆COD) + ar (∆BOC)
2ar(∆AOB) = 2ar(∆BOC)
1. False, Since A is any point on PQ, then ar (PAS) ≠ 30 cm2
⇒ ar (∆AOB) = ar (∆BOC) ... (3)
But, the statement can be true if PA is equal to PS. Similarly, ar (∆AOD) = ar (∆COD)  ... (4)
Justification: From (1), (2), (3) and (4), we get
PQRS is a rectangle inscribed in a quadrant of a circle of radius ar (∆AOB) = ar (∆BOC) = ar (∆COD) = ar (∆AOD)
13 cm.
Hence, proved.
3.
S 12 cm R
13 cm
5 cm

P AQ
8 Fundamental of Mathematics

1 6.
ar (∆DCE) = ar ( ABCD )
2
1
= × 96 = 48sq cm
2
Now, ar (∆ADE) + ar (∆DEC) + ar (∆BEC) = ar (∆ABCD)
∴ ar (∆ADE) + ar (∆BEC) = 96 – ar (∆DEC)
= 96 – 48
= 48 sq cm.
4. In parallelogram BDCE, BC is the diagonal.
∴ ar(∆BCE) = ar(∆BDC) ... (1)
In parallelogram ABCD, BD is the diagonal
∴ar(∆BDC) = ar(∆ADB) ... (2)
From (1) and (2)
ar(∆BCE) = ar(∆BDC) = ar(∆ADB)
∴ ar(ADCE) = ar(∆ADB) + ar(∆BDC) + ar(∆BCE)
= 3 ar(∆BCE)
= 3 × 24 = 72 cm2.
Given, BC = 8 cm, DF = 3(AF)
In parallelogram ABCD, 7.
AD = AF + DF
= AF + 3AF = 4 AF = 8
8
⇒ AF= = 2cm
4
∴ AF = 2 cm, DF = 6 cm
⇒ DE = 2 cm (∵ FE = BC = 8 cm)
∴ AE = 2 + 6 + 2 = 10 cm
In trapezium ABCE, AE = 10, BC = 8 cm
Perpendicular distance between AE and BC = 5 cm AE = 6 cm, CF = 8 cm
1 Area of parallelogram ABCD
∴ ar(ABCE) = × 5 × (10 + 8) = CD × AE = AB × AE
2 = AD × CF = BC × CF
1 ∴ AB × AE = BC × CF
= × 5 × 18 =45sq cm .
2 ⇒ 6AB = 8BC
5. AB 8 4
∴ = =
BC 6 3
∴ AB : BC = 4 : 3.
8.

Since BD is the diagonal of the parallelogram ABCD, we have


ar(∆ABD) = ar(∆BCD)
ar(∆ABD) + ar(∆BCD) = ar(∆ABCD)
2 ar(∆ABD) = ar(∆ABCD)
1
ar(∆ABD) = ar ( ABCD )  ... (1) E and F are the midpoints of AB and CD.
2 ∴ AE = BE
In ∆ADB, ‘E’ is the mid point of BD DF = FC
∴ AE is the median of the triangle ADB AB = CD (∵ Opposite sides of a parallelogram)
⇒ ar(∆AED) = ar(∆ABE) ∴ AE + BE = DF + FC
⇒ 2 ar(∆ABE) = ar(∆ADB) ... (2) ⇒ 2AE = 2DF
From (1) and (2) we get ⇒ AE = DF and AE || DF
1 ∴ AEDF is a parallelogram.
ar(∆ABD) = ar ( ABCD ) 1
2 ⇒ ar(∆AEFD) = ar ( ABCD )
1 2
2 ar(∆ABE) = ar ( ABCD ) By theorem,
2
1
∴ ar(∆ABCD) = 4 ar(∆ABE) ar(∆EFG) = ar ( ADFE )
2
Answer and Explanations 9

1 1 11.
⇒ ar(∆EFG) = × ar ( ABCD )
2 2
1
⇒ ar(∆EFG) = ar ( ABCD )
4
ar ( ∆EFG ) 1
∴ =
ar ( ABCD ) 4
Hence, the required ratio = 1 : 4.
9.

Given, DF = BC and DF || BC
∴ DBCF is a parallelogram.
ABCD is a trapezium.
AB || CD
A is extended to E and CE is joined
∴ AE || CD
In parallelogram DBCF, DC is the diagonal.
EP = ‘p’ units EQ = ‘q’ units 1
AD = ‘x’ units ∴ ar ( ∆DBC ) = ar ( ∆DBFC )  ... (1)
2
Now ar(∆ABCD) = ar(∆AFGD) + ar(∆BCGF)
Now ∆DBC and ∆DEC are two triangles on the same base
Given, EP ⊥ AD, EQ⊥ BC ⇒ PQ ⊥ AD
CD and between the same parallels AE and CD.
= AD × EP + BC × EQ
∴ ar(∆DBC) = ar(∆DEC) ... (2)
∴ ar(∆ABCD) = AD × PQ
From (1) and (2), we get
= x × p + BC × q (but BC = AD)
1
= x × (p + q) sq units. ar ( ∆DEC ) = ar ( ∆DBFC )
∴ ar(∆ABCD) = x × p + x × q 2
= x(p + q) sq units. ar ( ∆DEC ) 1
∴ =
10. ar ( ∆DBFC ) 2
∴ Required ratio is 1: 2.
9
12.

AB = 6 cm
CD = 12 cm
AB || CD
⇒ AB || PD
and BP || AD (given)
∴ ABPD is a parallelogram AD is the median
∴ AB = PD = 6 cm ‘E’ is any point on the median
BA = AQ = 6 cm ar (∆ABC) = 42 sq cm ar(∆BDE) = 8 sq cm
and PD = 6 cm and AQ || PD ∵ AD is the median
∴ AQDP is a parallelogram ar(∆ABD) = ar(∆ADC) (∵Median divides the
CD = 12 cm triangle into 2 triangles of equal area)
⇒ PD + PC = 12 ∴ 2 ar(∆ABD) = ar(∆ABC)
⇒ 6 + PC = 12 42
∴ ar (∆ABD) = = 21 cm 2
PC = 12 – 6 = 6 cm 2
Now, AB = PC ⇒ AB || PC ∴ ar(∆ADC) = 21 cm2
∴ ABCP is a parallelogram. In ∆BEC, ED is the median.
Let the area of (1) be ‘x’ sq units. ∴ ar(∆BDE) = ar(∆DEC)
∴ Area of (1) = Area of (2) = Area of (3) = Area of (4) = ‘x’ ∴ ar(∆DEC) = 8 sq cm.
sq units Now, ar(∆ADC) = ar(∆AEC) + ar(∆DEC)
x + x 2x ⇒ 21 = ar(∆AEC) + 8
∴ Required ratio = = = 1
x + x 2x ∴ ar(∆AEC) = 21 – 8
= 1:1. = 13 sq cm.
10 Fundamental of Mathematics

13. BD is the diagonal of the parallelogram.


1
ar ( ∆ADB ) = ar ( ∆ABCD )
2
In ∆ADB AO is the median
1
∴ ar(∆AOD) = ar(∆AOB) = ar ( ∆ADB )
2
1
∴ ar ( ∆AOD ) = ar ( ∆ADB )
2
1 1
= × ar ( ∆ABCD )
2 2
1
AD, BE, CF are the medians ar ( ∆AOD ) = ar ( ∆ABCD )
4
ar(∆ABC) = 40 cm2
The 3 medians divide the triangle into 6 triangles of equal area. ar ( ∆AOD ) 1
∴ =
∴ ar(∆AGF) = ar(∆AGE) = ar(∆GEC) = ar(∆CDG) = ar ( ∆ABCD ) 4
ar(∆BDG) = ar(∆BGF)
Hence, the required ratio is 1: 4.
⇒ ar(∆AGF) = ar(∆AGE) 48 = ar(∆GEC) = ar(∆CDG) =
ar(∆BDG) = ar(∆BGF) = = 8sq cm 16.
6
∴ ar(DGEC) = ar(∆GEC) + ar(∆GDC)
= 8 + 8 = 16 cm2.
14.

ABCD is a trapezium
(AB || CD), E is the mid point of CD
AB = 8 cm,CD = 12 cm
Ar(trap. ABCD) = 50 sq cm
1
⇒ h ( AB + CD ) =
50
2
AD is the median. Where ‘h’ is the ⊥ distance between the parallels AB and
ar(∆ABC) = 36 sq cm CD.
1
‘E’ is the mid point of AD. h ( 8 + 12 ) =
50
∵ AD is the median 2
1 20h = 100
ar(∆ABD) = ar(∆ADC) = ar ( ∆ABC )
2 100
h
= = 5 cm
1 20
= 18cm 2
× 36 =
2 ∴ AP = EQ = BR = 5 cm
In ∆ABD, ‘E’ is the mid point of AD 1 1
ar(∆ABE) = AB × EQ = × 8 × 5 =20 sq cm
∴ BE is the median 2 2
1 1 1
∴ ar(∆BED) = ar(∆ABE) = ar ( ∆ABD ) ar(∆ADE) = DE × AP = × 6 × 5 = 15 sq cm
2 2 2
1 1 1
= 9cm 2
× 18 = ar(∆BEC) = EC × ER = × 6 × 5 = 15 sq cm
2 2 2
∴ ar(∆ABE) = 9 sq cm. ratio of ar(∆ABE) and ar(∆ADE) + ar(∆BEC)
15. 20 20 2
∴ Required ratio = = = = 2 : 3.
15 + 15 30 3
17. In ∆ABD, BO is the median and ‘O’ is the mid point of AD
∴ ar(∆BOD) = ar(∆ABO) ... (1)
In ∆ACD, CO is the median
ar(∆AOC) = ar(∆DOC) ... (2)
from (1) and (2), we get
ar(∆BOD) + ar(∆DOC) = ar(∆ABO) + ar(∆AOC)
⇒ ar(∆BDC) = ar(∆ABC)
⇒ ar(∆BDC) = 30 sq cm.
Answer and Explanations 11

20. Given, ABCD is a parallelogram


(AB || CD, AD || BC)
AE, CF are medians of ∆ADC and ∆ABC respectively.

Since, AE is the median, we get


∴ The required area of ∆BDC is 30 sq cm. ar(∆ADE) = ar(∆AEC) = 18 sq cm(given)
18. ∴ ar(∆ADC) = 2 ar(∆ADE) = 18 × 2 = 36 sq cm
Now, AC is the diagonal of the parallelogram ABCD, we
get
ar(∆ADC) = ar(∆ABC)
∴ ar(∆ABC) = 36 sq cm
Since, CE is the median of ∆ABC, we get
ar(∆ACF) = ar(∆BCF)
∴ 2 ar(∆ACF) = ar(∆ABC)
ar ( ∆ABC )
⇒ ar ( ∆ACF ) =
2
36
= = 18 sq cm
2
Given, BD = DE = EC and ar(∆ABD) = 17 sq cm ∴ ar(∆ADCF) = ar(∆ADE) + ar(∆AEC) + ar(∆ACF)
In ∆ABE, BD = DE = 18 + 18 + 18 = 54 sq cm.
⇒ D is the mid point of BE
21.
∴ AD is the median of the ∆ABE.
∴ ar(∆ABD) = ar(∆ADE) ... (1)
In ∆ADC, DE = EC
⇒ E is the mid point of CD 9
∴ AE is the median.
∴ ar(∆ADE) = ar(∆AEC) ... (2)
From (1) and (2), we get
ar(∆ABD) = ar(∆ADE) = ar(∆AEC)
Since, ar(∆ABD) = 17 sq cm (given), we have
ar(∆ADE) = ar(∆AEC) = 17 sq cm
ar(∆ADC) = ar(∆ADE) + ar(∆AEC) [REPLACE ‘B’ WITH ‘D’ and ‘D’ with ‘B’]
= 17 + 17 = 34 sq cm. The figure ABCD has one pair of opposite sides parallel to each
other(AD || BC)
19. ∴ It is a trapezium.
Let ‘h’ be the height of the trapezium.
Given, AD = 4 cm and BC = 6 cm.
1
Area of the trapezium = h ( AD + BC )
2
For the area of ABCD to be less than 84 s cm, the height
of the trapezium ABCD should be
1
h ( 4 + 6 ) < 84
2
Given, AB || CD ⇒ 10 h < 168
∆ADC and ∆BDC are an the same base CD and between the ⇒ h < 16.8
same parallels AB and CD ∴ The required maximum integer value of height is 16 cm.
∴ ar(∆ADC) = ar(∆BDC)
⇒ ar(∆AOD) + ar(∆COD) = ar(∆BOC) + ar(∆COD)
∴ ar(∆AOD) = ar(∆BOC)
Hence, proved.
Circles
Answer and Explanations 10
⇒ ∠OCD = 360° – (110° + 140° + 50°)
STEPWISE MARKING SOLUTION = 60° (11/2 Marks)
Since B, O, D are collinear,
Objective Type Question BD becomes a diameter
1. (b) ∠AEB = 60° = ∠CDF ( AB = CF) ⇒ ∠BCD = 90° ⇒ 60° + ∠OCB = 90°
In ΔFCD, ∠FCD = 90° ⇒ ∠OCB = 30°. (11/2 Marks)
⇒ ∠CFD = 30°
Exercise 2
2. (b) ∠PQR = 80°
⇒ ∠PSR = 100° 1. (a)
∠SPR = 50° ⇒ ∠SRP = 30°
C
⇒ ∠SQP = 30° = ∠SRP
3. (d) ∠MKL = ∠MNL = 40° 65°
⇒ ∠KNM = 60° + 40° = 100° O
⇒ ∠KLM = 180° – 100° = 80°
130°
4. (d) OD = CD and OD = OC A B
\ ∠COD = 60° ⇒ ∠AEB = 60°
\ ∠AOB = 2 × 60° = 120°
Since ∠AOB = 130°,
5. True 1
∠ACB   ∠AOB = 65° and
6. False 2
7. False ∠OAB = 25°
Since AC = BC;
8. Rectangle
 1 °
9. Centre ∠CAB = ∠ABC =  57   (1 Mark)
 2
10. Centre
\ ∠OAC = ∠CAB - ∠OAB
 1 °  1 °
Subjective Type Question =  57  – 25° = 32   (11/2 Marks)
 2  2
Exercise 1 (b)
1. The given statement is false. (1 Mark) A
10
2. The given statement is true. (1 Mark)
3. As ∠BAC = 70°, ∠BOC = 2 ∠BAC = 140° O
4. B D
A D
C

O Given ∠ABC = 80° and AB || CD


∠ABC + ∠C = 180°
120°
B C ( AB || CD and BC is a transversal)
\ ∠C = 100° (1 Mark)
Reflex ∠BOD = 200°
∠AOC = 120° and ∠BOD = 50° (given) ⇒ ∠BOD = 360° – 200° = 160° (1/2 Mark)
Join BO, AO, OC, OD. AB = CD
⇒ ∠BOA = ∠COD = x (1 Mark) 2.
⇒ x + x + 50 + 120 = 360 ⇒ x = 95°
∠AOD = ∠AOB + ∠BOD = 95° + 50° = 145°. (1 Mark) A E

5. ABCD is a cyclic quadrilateral F


∠ABC = 70° ⇒ ∠ADC = 110°
∠AOC = 2 × ∠ABC = 140°
∠DAO = 50°(given) B D
AOCD is a quadrilateral. C
2 Fundamental of Mathematics

(a) Given ∠BAF = 60° 5.


⇒ ∠BCD = 120°
( Opposite angles of cyclic quadrilateral) (1/2 Mark) A D
∠BFD = 100° (given)
EBCD is a quadrilateral.
\ ∠FBC + ∠BCD + ∠FDC + ∠DFB
= 360° B C
⇒ ∠FBC + ∠FDC
= 360° - (∠BCD + ∠BFD)
Let ABCD is a trapezium and AD || BC
\ ∠FBC + ∠FDC
∠CBD = ∠ADB (alternative angles)
= 360° – (100 + 120°) = 140° (11/2 Marks)
2∠CBD = 2∠ADB (1 Mark)
(b) ∠BAD = ∠BED ⇒ ∠BED = 60°
Angles made by the chords at the centre are equal. Here
( ∠BAD = ∠BAF = 60°) (1 Mark) 2∠CBD and 2∠ADB are the angles subtended by CD and AB
3. respectively at the centre of the circle.
\ AD = CD
A \ Non parallel opposite sides of a trapezium are equal in
length.  (2 Marks)

Exercise 3

1.
B D C
A
(a) Given, AB is the diameter
Join A to D
As AB is the diameter, O Oc
M
∠ADB 90 [Angle in a semi circle]
\ AD ^ BC (11/2 Marks) B
(b) Given ∠AOB = 60°
A Given,
O and O’ are the centres of the circle.
120° AB is the common chord.
60°
B C Join OA, OB, O’A and O’B. (1 Mark)
O
Proof: In ΔOAO’ and ΔOBO’
OO’ is common.
OA = OB (radii) and O’A = O’B (radii)
In ΔAOB, OA = OB (radii) ΔOAO’ ≅ ΔOBO’
120° [SSS congruence property]
∠OBA = ∠OAB = = 60°  (1/2 Mark) ∠AOO’ = ∠BOO’ [CPCT] (1 Mark)
2
Now, in ΔAOM and ΔBOM
\ OA = OB = AB
OM is common.
But AO = OC (radii)
∠BOM = ∠AOM (by proof)
⇒ OC = AB (1 Mark)
OA = OB (radii)
4. Given, ∠AOC = 120° ΔAOM ≅ ΔBOM
[SAS congruence property] (1 Mark)
A AM = MB
∠OMA = ∠OMB (CPCT)
But ∠OMA + ∠OMB = 180°
O 120° B \ ∠OBA = ∠OMB = 90°
OO’ is the perpendicular bisector of AB. (1 Mark)
2.
C
C
B
Reflex ∠AOC = 360° – 120° = 240° (1 Mark) M
1 O P
\ ∠ABC = (Reflex ∠AOC) N
2 D
1 A
=   240=
° 120°
2 Given, AB = CD
∠A + ∠C = 360° - (120° + 120°) Draw OM ^ CD
= 120° (1 Mark) ON ^ AB and Join OP. (1 Mark)
Answer and Explanations 3

Proof : In ΔOPM and ΔOPN 5.


OP is common B
∠OMP = ∠ONP = 90° D
OM = ON
[equal chords are equidistant from centre of a circle] O
∠OPM ≅ ∠OPN N M
[RHS congruence property] (11/2 Marks)
MP = NP (CPCT)
C
1 1 A
AB = CD ⇒   AB =   (CD)
2 2
⇒ AN = CM ⇒ AN + NP = CM + MP Given, AB, CD are two chords and AB > CD
⇒ AP = CP Draw ON ^ AB and OM ^ CD.
⇒ PB = PD ( AB = CD) (11/2 Marks) Join O to A and O to C.
3. AB > CD (given)
1 1
  AB >   CD
2 2
O  (2 Marks)
A B
AN > CM
AN2 > CM2
C D AN2 – OA2 > CM2 – OA2 (1 Marks)
AN2 – OA2 > CM2 – OC2
( OA = OC)
P OA2 – AN2 < OC2 – CM2
ON2 < OM2
Given, AOB is a diameter ON < OM (1 Marks)
Join OD and AD (1 Mark)
OC = OD (radii)
and OC = CD (given) PROBLEMS FOR ADDITIONAL PRACTICE
\ ∠OCD is an equilateral triangle. (1 Marks)
\ ∠COD = 60° Objective Type Question
1 1. (a) AC is the diameter ⇒ ∠ABC = 90°
∠CAD =   ∠COD = 30°
2 OB = BC
∠ADB = 90° (1 Marks) ⇒ ∠OBC = ∠C = ∠BOC = 60°
(Angle in a semi-circle) \ ∠AOB = 180 – ∠BOC = 120°
⇒ ∠ADP = 90°
\ ∠APD = 180 – ∠ADP – ∠PAD
2. (b) ∠CAD = 40° = ∠CBD 10
⇒ ∠ACB = 180° – (80° + 40°)
= 180 – 90
– 30 = 60° (1 Mark) = 60°

4. 3. (b) ∠DAC + ∠DBC + ∠ACB + ∠ADB = 360°


C ⇒ 100° + 50° + 140° + ∠ADB = 360°
B ∠ADB = 70°
M
O P ⇒ ∠AEB = 70° ( ∠AEB = ∠ADB)
N \ ∠ADB + ∠BEA = 2 × 70° = 140°
D
A 4. (c) ∠ABC = 125°, ∠ACE = 50°
∠ACE = ∠ABE = 50°
Let O be the centre of the circle. ⇒ ∠EBC = ∠ABC – ∠ABE
Given AB = CD = 125° – 50° = 75°
Draw ON ^ AB and OM ^ CD (1 Mark)
5. True
In ΔOMP and ΔONP
ON = OM 6. False
(Equal chords are equidistant from the centre). (1 Mark)
∠OMP = ∠ONP = 90° 7. True
By construction, OP is common
8. Circular
ΔOMP ≅ ΔONP (1 Marks)
[RHS congruence property] 9. Circumference
∠MPO = ∠NPO
\ Centre of the circle lies on the bisector of ∠APC. 10. segment
 (1 Marks)
4 Fundamental of Mathematics

Subjective Type Question 9. ∠DAB = 45°


⇒ ∠ADB = 180° – 2 (45°) = 90°
Exercise 1 ⇒ Reflex ∠ADB = 270°
270°
1. The given statement is false. ⇒ ∠CDA = = 135° ( AC = BC))
2
2. The given statement is false.
(since O need not the centre of the circle)
180°−135°   1 °
⇒ ∠CAD =   =  22 
 2   2
3.
10. ∠ADB = 50° = ∠ACB (angles in the same segment)
P (180o − 50o )
O AD = BD (given) ⇒ ∠ABD = ∠BAD = = 65°
2
∠ABC = 100° (given)
Q R In ∆CAB, ∠CAB = (180° – 100° – 50°) = 30°
⇒ ∠CAD = ∠BAD - ∠CAB = 65° - 30° = 35°
∠PQR = 95°
Exercise 2
The angle made by PR at the centre = 2(95°)
= 190° = Reflex ∠POR 1. ∠APB = 30° (given)
\ ∠POR = 360° – 190° = 170° Join AB. In ∠APB, AP = BP
4.
⇒ ∠PAB = ∠PBA = x
U T ⇒ x + x + 30 = 180°
130° ⇒ x = 75°
∠PAB + ∠PEB = 180°
60° (opposite angles of a cyclic quadrilateral)
P S 75° + ∠PEB = 180°
X
⇒ ∠PEB = 105°.
R
Q 2. Given that ∠EAD = 25° and ∠CBD = 20°
∠EAD = ∠EBD = 25°
Join PU (angles in the same segment are equal)
⇒ UPST is a cyclic quadrilateral ∠EBC = ∠EBD + ∠CBD
Given, ∠UTS = 130° = 25° + 20° = 45°
⇒ ∠UPX = 50° EBCD is a cyclic quadrilateral.
( opposite angles of a cyclic quadrilateral are ∴ ∠EBC + ∠EDC = 180°
supplementary) (opposite angles of a cyclic quadrilateral)
In ΔUPX, ∠PXU = 60° and ∠UPX = 50° ∠EDC = 180° – 45° = 135°.
⇒ ∠PUX = 180° – (60° + 50°) = 70°
⇒ ∠PQR = 180° – 70° = 110° 3.

5.
O Oc

A B C D
O
Given that the circles with the centres O and O’ are congruent
P S circles and AB and CD be their equal chords
Q R In ΔOAB and ΔO’CD
AB = CD [given]
Join PS and OQ, OR OA = OB = O’C = O’D
∠OPS = 15° [radii of congruent circles]
⇒ ∠OSP = 15° ( OP = OS, radii) ΔOAB ≅ ΔO’CD [SSS congruency]
⇒ ∠POS = 180° – 15° – 15° = 150° \ ∠AOB = ∠CO’D [CPCT]
PQ, QR, RS subtends equal angles at the centre.
4.
Let each angle be x.
A
⇒ 3x = 150° ⇒ x = 50°
In ΔOQR, ∠OQR + ∠ORQ = 180° – 50° = 130° m
130°
\ ∠OQR = ∠ORQ = = 65° ( OQ = OR) B O
2

6. False
C
7. True

8. If ∠ABC = 40°, then ∠AOC = 2 × ∠ABC = 80° Given, that


\ Reflex ∠AOC = 360° – 80° = 280° A, B and C are on the circumference of a circle.
Answer and Explanations 5

Join A to B and draw a perpendicular bisector of AB i.e., ‘ℓ ’. ⇒ ∠OBC = 60° ⇒ ∠EBC = 60° – 20° = 40°
Join B to C and draw a perpendicular bisector of BC i.e., ‘m’. Given ∠AED = 120°
ℓ and m can have only one point in common i.e., O. ⇒ ∠BED = 180° – 120° = 60°
The perpendicular bisectors of the chords of a circle passes In ΔEFB, ∠EFB = 180° – (∠BEF + ∠EBF)
through the centre of the circle. = 180° – (100°) = 80°
\ O is the centre of the given circle. ⇒ ∠CFD = 80°
5. (a) 8.
A

O Oc
O
A B C D
B C
Given, that circles with centres O and O’ are congruent.
Let AB and CD are their chords such that ∠AOB = ∠CO’D
Given, ∠BAC = 55° and ∠AOB = 150°. In ΔAOB and ΔCO’D
∠BOC = 2∠BAC = 2 × 55° = 110° ∠AOB = ∠CO’D (given)
[Angle subtended by an arc at the centre of circle is OA = OB = O’C = OD = radius
double the angle subtended by the arc at any point on the [radii of two congruent circles] ΔAOB ≅ ΔCO’D
remaining part of circle] [SAS congruence property]
∠AOB = 150° \ AB = CD
∠AOC = 360° – (∠BOC + ∠AOB) = 100°
1 1 9.
∠BCA =   ∠AOB =   × 150° = 75°
2 2 O C
1 X°
∠ABC =   ∠AOC × 100° = 50°
2
(b) A B
C
Given, that OA = AB
In ΔOAB
OA = OB (radii) OA = AB (given)
D \ ∠AOB is an equilateral triangle.
80° \ ∠AOB = 60°
40° 1
A B x =   ∠AOB = 30°
2
Given, ∠DBC = 80° and ∠BAC = 40° [ The angle subtended by an arc at any point on the remaining 10
∠BAC = ∠BDC = 40° [Angles in the same segment] circle is half the angle subtended by the arc at the centre]
\ In ΔDBC, ∠DCB + ∠DBC + ∠CDB = 180° 10. (a)
∠DCB = 180° – 80° – 40° = 180° – 120° = 60°
6. Given that PQ = QR
⇒ Let ∠POQ = ∠QOR = x O
A B
⇒ ∠POR = 2x
∠POR = 2 × ∠PSR = 2 × 40° = 80°
C
\ ∠POQ = 40°
∠OPQ + ∠OQP = 180° – 40° = 140° D
140°
\ ∠OPQ = ∠OQP = = 70°  ( OP = OQ) Given, OD ^ AB Join D to B
2 ∠DOB = 90°(given)
7. 1 90°
∠DCB =   ∠DOB = = 45°
2 2
O (b)
A E B
D C
F
D
C
A B
Given, ∠OA = 20°
Join OB, OC Given that
∠OAB = ∠OBE = 20° ABCD is a parallelogram inscribed is a circle.
In ΔOBC, OB = OC = BC (Given AO = BC) ∠A = ∠C (opposite angles of a parallelogram)
6 Fundamental of Mathematics

But ∠A + ∠C = 180 (sum of opposite angles of a cyclic Given that ∠QRS = 150°.
quadrilateral) \ ∠A = ∠C = 90° PQRS is a cyclic quadrilateral.
\ ABCD is a rectangle.
∠QRS + ∠QPS = 180°
Exercise 3 150°+ ∠QPS = 180°
1. Given, A, B and C are the three non-collinear points. ∠QPS = 180° – 150° = 30°
∠QOS = 2 ∠QPS ( The angle subtended by an arc at the
A centre of a circle is double the angle subtended by the same arc
m
at any point on the remaining part of the circle)
O
ℓ ∠QOS = 2(30°) = 60°
B C ∠QOS = ∠AOB = 60°
4. (a)
(i) Draw a perpendicular bisector of AB i.e., ‘ℓ’
R S
(ii) Draw a perpendicular bisector of BC i.e., ‘m’ T
Every point on ‘ℓ’ is equidistant from A and B similarly 50°
every point on ‘m’ also equidistant from B and C. 25°
 ℓ and m can have only one common point. Let it P Q
be O. O
\ There exist only one point which is at equidistant from all
the three points.
Hence, only one circle can be drawn with centre O, such
that it passes through A, B and C.
Given ∠QTS = 50° and ∠TQS = 25°
2. Given that AB = 24 cm and CD = 10 cm In ΔQTS.
Let PQ be the distance between AB and CD ∠TQS + ∠TSQ + ∠STQ = 180°
PQ = 17 cm (given) \ ∠TSQ = 180° – 50° – 25° = 105°
∠PSQ = ∠PRQ = 105° (angles in the same segment).
Q
C D \ Reflex ∠POQ = 2 × 105 = 210° [ Angle subtended by
an arc at the centre is double the angle subtended by the
arc at any point on the remaining part of the circle].
O (b)
A P B
A F
AB || CD D
90°
Let OQ = x and the radius be r. E
As OQ = x, OP = 17 - x C
In ΔOQD, OD2 = OQ2 + QD2 B
1
r2 = x2 + (5)2 ... (1) [QD =   CD] Given, ∠ADC = 90° and ∠FAE = 25°
2
∠D + ∠B = 180°
Similarly, in ΔOPB [opposite angles of a cyclic quadrilateral]
OB2 = OP2 + PB2 ∠B = 90°
1 CB || AE (given)
r2 = (17 – x)2 + 122 [PB =   AB]  ... (2) ∠B = ∠BAE = 90° [alternative angles]
2
∠BAF = 90 + 25° = 115°
From (1) and (2) ∠BCD = ∠BAF = 115°
x2 + 25 = (17 – x)2 + 144 [exterior angle of a cyclic quadrilateral is equal to its
x2 + 25 = 289 – 34x + x2 + 144 opposite interior angle]
34x = 408 5. ABCD is a cyclic quadrilateral.
x = 12 \ ∠BCF = ∠BAD = 70°
( The exterior angle of a cyclic quadrilateral is equal to
\ r= 144 + 25= 169= 13 its interior opposite angle)
Radius of the circle = 13 cm. BEFG is a cyclic quadrilateral.
3. ∠ABG = ∠EFG = 120°
∠BCF = ∠BGF = 70°
B ( Angles in the same segment are equal)
P
\ ∠BAD + ∠ABG + ∠BGF
S = 70° + 120° + 70° = 260°
O R
6. Given, AB = AC = 6 cm
Join O to B and O to C and join B to C.
Q
BC intersects OA at M.
A BM = CM
Answer and Explanations 7

Given, that AB = AC.


O AB and AC are diameters.
5 Mark intersecting point (other than A) as D.
B C R.T.P: B, D and C arc collinear points.
M
6 6 Construction: Join AD, BD and CD
Proof:
A ∠ADB = 90° (Angle in a semi circle)
∠ADC = 90° (Angle in a semi circle)
Let OM = x and BM = y = CM ∠ADB + ∠ADC = 90° + 90°
In ΔOMC, OC2 = OM2 + MC2 \ ∠BDC = 180°
25 = x2 + y2 \ B, D and C are collinear points.
y2 = 25 – x2  ... (1) i.e., the circles intersect on BC.
Similarly in ∠AMC
9.
AC2 = AM2 + MC2
36 = (5 – x)2 + y2  ... (2) D
From (1) and (2), 25 – x2 = 36 – (5 – x)2
65
⇒ 25 – x2 = 36 – 25 – x2 + 10x C
14 a
14 = 10x ⇒ x =
10 b 40 P
49 A c B
Now y2 = 25 −
25
625 − 49 576 Q
= y 2 = =
25 25 In ΔPDA, ∠PDA + ∠PAD + ∠P = 180°
24 ∠PAD = 180° – 105° = 75°
⇒ y=
5 ∠DAB + ∠BCD = 180° [Sum of opposite angles of a cyclic
BC 24 quadrilateral]
But= y = a = 180° – 75° = 105°
2 5 b = ∠CDQ = 65°
48 [Exterior angle of a cyclic quadrilateral is equal to its interior
⇒ BC = = 9.6 cm
5 opposite angle]
BC = 9.6 cm ∠DQB + a + 65° = 180°
\ ∠DQB = 180° – 65° - 105°
7. Given, = 180° – 170° = 10°
\ c = 10°
10.
O
10
A
A B P C D

circles are concentric circles B C


AD is the chord of the bigger circle and BC is the chord of the D
smaller circle.
Drop OP ^ AD
Given, O is circum centre of the triangle ABC
In the outer circle OP ^ AD
OD ^ BC
AP = PD  ... (1)
In ΔOBD and ΔOCD
[A perpendicular drawn from the centre to a chord bisects the
OB = OC
chord]
∠ODB = ∠ODC
Similarly, in the smaller circle
OD is common
OP ^ BC.
ΔODB ≅ ΔODC [RHS congruence property]
BP = PC ... (2)
∠BOD = ∠COD
From (1) and (2)
1
AP – BP = PD – PC ⇒ AB = CD ∠BOD =   ∠BOC  ... (1)
2
8.
A 1
∠A =   ∠BOC  ... (2)
2
[Angle subtended by an arc at any point on the remaining circle
is the half of the angle subtended by the same arc at the centre].
B C From (1) and (2) ∠BOD = ∠A
D
8 Fundamental of Mathematics

∠CDB and ∠CAB are angles made by the arc BC


HOTS ∴ ∠CDB = ∠CAB (angles in the same segment.).
1. AB = CD = 8 cm.(given) Now ∠CAB = ∠OAB
∠AOB = 70° (given) In ∆OAB
∴ ∠COD = 70° (equal chords subtend equal angles at the ∠AOB = 110°(given)
centre) OA = OB (radii of the same circle)
In ∆AOB, 180 –110° 70°
OA = OB (radii of the same circle) ∠OAB = ∠OBA =
∴  = = 35°
2 2
∠AOB = 70°
∠OAB = ∠OBA (angles opposite to equal sides) ∴ ∠OAB = ∠OBA = 35°
180 – 70 110 ∴ ∠CAB = 35°
∴ ∠OBA = = = 55° ∴ ∠CDB = 35°
2 2 ∴ ∠DCA + ∠CDB = 50° + 35° = 85°
Similarly, in ∆OCD
OC = OD (radii of the same circle) 6. ∠DCB = 95°, ∠AOC = 170° (given)
∠COD = 70° ABCD is a cyclic quadrilateral
180 – 70 110 ∴ ∠DCB + ∠DAB = 180° (opposite angles of a cyclic
∴ ∠OCD = = = 55° quadrilateral are supplementary)
2 2 ∴ ∠DAB = 180 – ∠DCB
∴ ∠OCD + ∠OBA = 55° + 55° = 110°. = 180 – 95° = 85°
2. ∠AOB = 60° (given), ∠COD = 54° (given) ∠AOC is the angle subtended by (
 ADC ) ° at the centre
Now ∠AOB + ∠BOC + ∠COB = 180°
‘O’.
∴ ∠BOC = 180 – (60 + 54) = 180 – 114 = 66°
170°
∴ angle subtended by arc BC at the centre ‘O’ is 66° ∴ ∠ABC = = 85°
66 2
∴ ∠BEC = = 33° . ∠ABC = 85°⇒ ∠ADC = 180° – 85° = 95°
2
∴ The angles of the quadrilateral ABCD are ∠ADC = 95°,
3. arc AB = arc BC ∠DAB = 85°, ∠ABC = 85°, ∠DCB = 95°.
∠AOB = ∠BOC (equal arcs subtend equal angles at the
∴ 
centre.) 7. AB = 12cm, CD = 8 cm
Let ∠AOB = ∠BOC = x° Let the distance of the chord AB from centre ‘O’ be ‘x’ cm.
∴ ∠AOC = x° + x° = 2x° 3
∴ x.
the distance of chord CD from centre ‘O’ =
1 1 2
Now ∠ADC = 41.5° = ∠AOC = × 2x = x°
2 2 1 1 1 1
Now DE = CD = × 8 = 4 cm and BF = AB =
⇒ x = 41.5° 2 2 2 2
∴ ∠AOC = ∠BOC = 41.5° × 12 = 6 cm
∠AEC = ∠ADC = 41.5° (angles in the same segment). ( the ⊥ar from the centre bisects the chord)
∴ ∠BOC + ∠AEC = 41.5° + 41.5° = 83°
E
4. AC = BC (given) C D
∠OAB = x°, ∠OAC = y° (given)
AC = BC O
∴ ∠CAB = ∠CBA
∠OAC + ∠OAB = ∠OBA + ∠OBC  ...(1) A F B
In ∆OAB
OA = OB (radii of the same circle) In ∆OED
∴ ∠OAB = x° = ∠OBA  ...(2) DE 2 + OE 2 = OD2
∴ from (1),  3 2
∠OAC = ∠OBC = y° 42 +  x  = OD2
Now in ∆ABC
2 
∠ACB = 180 – (∠CAB + ∠CBA) In ∆ OFB
= 180 – (x° + y° + x° + y°) = 180 – 2 (x° + y°) OF2 + BF2 = OB2
∴ ∠AOB = 2 (∠ACB) = 2(180 – 2(x° + y°)) x 2 + 62 = OB2
= 360° – 4(x° + y°).
Now OB = OD (radii of the same circle)
5. ∠DEC = 140°(given) ∴ OB2 = OD2
ADEC is a cyclic quadrilateral. 9
∴ 16 + x 2 = OD 2
∴ ∠DAC = 180° – 140° = 40° (opposite angles of a cyclic 4
quadrilateral are supplementary)
9 2
 A, O, C are collinear, AC is the diameter. ⇒ x − x 2 = 36 – 16 = 20
∴ ∠ADC = 90° (angle in a semicircle) 4
In ∆ADC 9 x 2 –4 x
2

∠ADC = 90°, ∠DAC = 40° = 20


∴ ∠DCA = 180°– (90° + 40°) = 180° – 130° = 50° 4
Answer and Explanations 9

= 360° (sum of angles around a point)


5x2
= 20 13k + 13k + 10k = 360°
4 36k = 360°
5x2 = 80 ⇒ x2 = 16 ⇒ x = 4 cm. k = 10
∴ ∠AOC = 13 × 10 = 130°, ∠BOC = 13 × 10 = 130°
( 4) + ( 6)
2 2
∴ Radius of the circle =
∠AOB = 10 × 10 = 100°
= 16 + 36 1 1
Now ∠ABC = × ∠AOC = × 130°= 65°
= 52 = 2 13 cm . 2 2
8. In OAB, ∠AOB = 100°, ∠EAO = 10°, ∠EBO = 20° 1 1

∠ACB = × ∠AOB = × 100°= 50°
OA = OB (radii of the same circle) 2 2
180° –100° 80°
∠OAB = ∠OBA =
∴  = = 40° 1 1
2 2 
∠CAB = × ∠BOC = × 130°= 65°
2 2
∴ ∠OAB = ∠OBA = 40°
1 1 12. AB = 12 cm, CD = 16 cm, PR = 4 cm.

∠ACB = ∠AOB = × 100 =50° Let the distance of the chord CD from the centre ‘O’.
2 2
∠CAB = ∠EAO + ∠OAB = 10° + 40° = 50° P
A B
In ∆ABC, 4cm
∠ACB + ∠CAB + ∠CBA = 180° (angle sum property) C D
Q
∠CBA = 180° – (∠ACB + ∠CAB) = 180° – (50° + 50°)
= 180° – 100° = 80°. O
∴ ∠CBD + 20° + 40° = 80°
⇒ ∠CBD = 80° – 60° = 20°.
Now ∠CBD and ∠CAD are angles made by the arc CD
∴ ∠CBD = ∠CAD be ‘x’ cm.
∴ ∠DAC = 20°. ⇒ OQ = x cm
1 1
9. ∠AFB = 100°, ∠AOB = 130° PB = ×12 =
6 cm , QD = ×16 =
8cm (⊥ar drawn
∠FAO = ∠FBO. (given)
2 2
from centre to the chord bisects the chord)
D C In ∆OPB
F OP 2 + PB2 = OB2
(4 + x)2 + 62 = OB2
O In ∆OQD
OQ 2 + QD 2 = OD2
A B
x 2 + 82 = OD2
10
In ∆AOB OB = OD (radii of the same circle)
1 OB2 = OD2
∠AOB = 130°, ∠ADB = ∠AOB = 65° ⇒ (4 + x)2 + 36 = x2 + 64
2
16 + x2 + 8x + 36 = x2 + 64
∠AFD = 180°– ∠AFB = 180°– 100° = 80° 8x + 52 = 64
In ∆AFD, 8x = 12
∠DAC = 180° – 65° – 80° = 35°. 12 3
x
= = cm
= 1.5cm .
10. (
AB ) ° = (
BC ) ° = (
CA ) ° ⇒ AB = BC = CA (If the arcs of 8 2
a circle are congruent then their corresponding chords are 13. Given ∠D is the greatest angle
congruent) Let ∠D = x
∴ ∆ABC is an equilateral triangle. ∠D and ∠A differ by 40°
∴ ∠ABC = ∠ACB = ∠CAB = 60° ∴ ∠A = x – 40°
∠AOB = 2 × ∠ACB = 2 × 60° = 120° ∠D and ∠C differ by 20°
∠BOC = 2 × ∠CAB = 2 × 60° = 120° ∴ ∠C = x – 20
∠AOB ∠BOC 120° 120° Now ∠A + ∠C = 180° (opposite angles of a cyclic
 + = + = 40° + 60° = 100°. quadrilateral)
3 2 3 2
C
11. ∠AOC: ∠BOC = 1: 1 and ∠BOC: ∠AOB = 13: 10 D
Let ∠AOC = x, ∠BOC = y, ∠AOB = z.
x: y = 1: 1, y: z = 13: 10 O
x: y = 13: 13 y: z = 13: 10
⇒ x: y: z = 13 :13: 10 B
Let ∠AOC = 13k, ∠BOC = 13k and ∠AOB = 10k A
∴ ∠AOC + ∠BOC + ∠AOB
∴ x – 40 + x – 20 = 180°
10 Fundamental of Mathematics

2x – 60 = 180° ⇒ 2x = 240° ∴ ABCD is a rhombus in which each angle is equal to 90°


240° ∴ ABCD is a square.
x
⇒ = = 120°
2 17. In ∆OSQ
∴ ∠D = x = 120° ∠OSQ = 55° (given)
∠C = x – 20 = 120° – 20° = 100° OS = OQ (radii)
∠A = x – 40 = 120° – 40° = 80° ∴ ∠OSQ = ∠OQS = 55°.
∠B = 180 – ∠D = 180° – 120° = 60° ⇒ ∠SOQ = 180 – (55° + 55°) = 180° – 110° = 70°
1 1
14. AB = CD = 6 cm (given) ∠SPQ =
⇒  × ∠SOQ = ×70°= 35°
OP = 4 cm (given) 2 2
In ∆OPB, ∠OPB = 90° Now SPQR is a cyclic quadrilateral.
OP = 4 cm (given) ∠SPQ + ∠SRQ = 180° (opposite angles of a cyclic
∴ 
AB quadrilateral)
PB = ∴ ∠SRQ = 180° – ∠SPQ = 180 – 35° = 145°.
2
(perpendicular from the centre to the chord bisects the chord) 18. ∠QTR = 110°
6 ∠TQP = 25°
∴ PB= = 3cm ∠TQP and ∠TRS are angles made by the arc PS.
2
∴ ∠STR = 180 – 110° = 70°.
OB2 = OP 2 + PB2 (by Pythagoras theorem) In ∆STR, ∠STR = 70°, ∠TRS = 25°
∴ OB2 = OP 2 + PB2 = 42 + 32 = 16 + 9 ∴ ∠TSR = 180° – (70° + 25°) = 180° – 95° = 85°
∴ ∠QSR = 85°
= 25 = 5cm
⇒ The radius of the circle = 5 cm 19. Radius of inner circle = 11 cm .
∴ The diameter of the circle = 2 × 5 = 10 cm. ⇒ OQ = 11 cm
15. OP = OQ = 5 cm Radii of outer circle = 38 cm .
  P and OQ are perpendiculars from the centre ‘O’ to the
O ⇒ OP = 38 cm
chords AB and CD respectively and OP = OQ.
We can say that AB = CD (since the chords which are In ∆OPT, PT = OP 2 – OT 2
equidistant from the centre are equal)
( 38 ) − ( 2 )
2 2
The diameter = 26 cm. = = 38 – 2 = 6 cm
26
∴ The radius of the circle = = 13cm . OT = 2 cm
2
∴ OB = OC = OA = OD = 13 cm In ∆OQT, QT = OQ 2 – OT 2
In ∆OPB, ∠OPB = 90°
( 11) − ( 2 )
2 2
OP = 5 cm = = 11 – 2 = 3cm
OB = 13 cm
PQ = PT – QT = 6 – 3 = 3 cm.
OB2 = OP 2 + PB2
20. ∠AOB = 100°; ∠AFB = 80°; ∠CAD = 30°.
⇒ PB = PB = OB2 – OP 2
∠FAO = ∠FBO
(13) – (5) = 169 – 25 = 144 = 12 cm
2 2
= In ∆AOB
∠AOB = 100°, OA = OB (radii)
AB
But PB = (The perpendicular from the centre to the 180° – 100° 80°
2 ∠OAB = ∠OBA =
∴  = = 40°
chord bisects the chord) 2 2
∴ AB = 2PB = 2 × 12 = 24 cm In ∆ AFB,
⇒ AB = CD = 24 cm. ∠AFB + ∠FAO + ∠OAB + ∠ FBO + ∠ OBA = 180°
16. AB = BC = CD =DA, AB || CD, (Angle sum property)
∴ ABCD is a rhombus. Let ∠FAO = ∠FBO = x
∠ADC + ∠ABC = 180° (opposite angles of a cyclic 80° + x + 40° + x + 40° = 180°
quadrilateral) 160 + 2x = 180°
∠ADC = ∠ABC ( ABCD is a rhombus.) 180° – 160° 20°
180° x= = = 10°
∠ADC = ∠ABC = = 90° 2 2
2
∠CAD and ∠CBD = 30°. (angles in the same segment)
Similarly, ∠DAB = ∠DCB = 180°
∴ ∠DAB = ∠DCB (ABCD is a rhombus) 
∠ABD = ∠OBA + ∠FBO + ∠CBD = 40° + 10° + 30° =
80°.
180°
∴ ∠DAB = ∠DCB = = 90°
2
Constructions
Answer and Explanations 11
(3) With P as centre and with the same radius which intersect
STEPWISE MARKING SOLUTIONS the previous arc at X. Join AX ∴∠XAB = 60
(4) Taking P and X as centres draw two arcs which intersect
Objective Type Questions at Y.
(5) Join AY which bisects the angle XAB ie, ∠YAB = 30°.
1. (d).
 (2 Marks)
2. (a) 1
4.
3. (d) Ruler and Compass.

4. True P X
5. Sides

Subjective Type Questions


A
1. Steps of construction:
(1) With the help of protractor construct∠ABC = 28°.
(2) Draw a line segment PQ of reasonable length.
(3) Taking P as centre and with a reasonable radius cut an arc D
which meets PQ at X. 2.5 cm R
(4) With the same radius and B as the centre cut an arc
45 m
meeting AB at M and AC at L. 4.8 c
(5) Now taking X as the centre and LM as radius cut an arc Q
which meets previous arc at N. B
(6) Join PN and extend to R.
(7) ∠RPQ is the required angle whose measure is 28°.  (2 Marks)
 (2 Marks) Steps of consumption:
A R (1) Draw QR = 4.8 cm.
(2) At Q construct ∠XQR = 45°.
(3) Mark a point D on QX such that QD = 2.5 cm = PQ – PR.
M N
28° (4) Join RD and draw perpendicular bisector AB of RD.
B L C P X Q (5) Let the perpendicular bisector AB intersect QX at point P.
 (2 Marks) (6) Join PR, PQR is the required triangle. (2 Marks)
2. X 5.

A B 11

P B C Q
Y
 (2 Marks)
Steps:  (2 Marks)
(1) Draw a line segment AB = 8.8 cm.
(2) With A as centre and radius equal to more than half of AB Steps:
draw two arcs on either side of AB. (1) Draw a line segment PQ whose length is same as perimeter
(3) With the same radius as in step (2) with ‘B’ as centre draw of ∆ABC which is 6 cm.
two more arcs which intersect the previous arcs at two (2) At P and Q construct the angles of 40° and 60° respectively
points X and Y. (3) Bisect the angles drawn at P and Q which intersect
(4) Join XY which is the required bisector of AB. (2 Marks) at A.
(4) Draw the perpendicular bisectors of AP and AQ which
3. Steps: (2 Marks) intersect PQ at B and C respectively.
(1) Draw a line segment AB of any suitable length. (5) Join AB and AC. Now ABC is the required ∆ABC.
(2) With A as centre and with any suitable radius draw an arc  (2 Marks)
which intersect AB at P.
2 Fundamentals of Mathematics

6. 2. Steps of construction:

X y
P M

X
P

Y Z Q R
Steps: (2 Marks) (1) Draw a line segment QR = 5 cm
(1) Draw a line segment (2) Construct ∠RQY = 50
YZ = 3.8 cm. (3) With Q as centre and with radius equal to 8 cm draw an
(2) Construct an angle ∠MYZ = 3 5°. arc which intersect QY at X such that QX = 8 cm
(3) Mark the point P on YM such that YP = XY + XZ = 8.4 cm (4) Draw the perpendicular bisector of RX to meet QX at P.
(4) Join PZ and draw the perpendicularbisector of PZ which (5) Join PR which gives the required ∆PQR
meet PY at X.
(5) Join XY and XZ which gives the required ∆XYZ. 3. Construction steps:
 (2 Marks)
A X

PROBLEMS FOR ADDITIONAL PRACTICE

Objective Type Questions


1. (d) D
2. (d)

3. (d) B C
4. False

5. False
(1) Draw a line segment BC = 6 cm.
6. Can not (2) Draw ∠XBC = 55°.
(3) With B as centre and with radius equal to AB – AC = 2 cm
7. Two,an
draw an arc which intersect BX at D.
(4) Join CD and draw the perpendicular bisector of DC to
Subjective Type Questions meet BD produced at A.
(5) Join A and C ∴ABC is the required triangle.
Exercise – 1
4. Construction steps:
1. Steps of construction:
X
X D

A B
A

Y B C

(1) Draw a line segment BC = 8cm


(1) Draw a line segment AB = 5.5 cm.
(2) Construct ∠X BC = 90° and with B as centre and radius
(2) With centre A and radius equal to more than half of AB
AB + AC = 16 cm draw an arc on the perpendicular which
draw two arcs on either side of AB.
intersects at D
(3) With B as centre and with the same radius draw two arcs
(3) Join CD and draw the perpendicular bisector of CD which
intersect the arcs drawn in step 2 at X and Y.
intersect BD at A
(4) Join XY which is the required perpendicular bisector of
(4) Join AC to obtain the required ∆ABC.
AB.
Answer and Explanations 3

5. (a) Steps of construction: (3) With any radius at A as centre draw an arc XY and with the
(1) Draw a line segment PQ same radius with P as centre also draw an arc to intersect
(2) Taking P as centre and with a reasonable radius cut an arc PQ at M.
meeting PQ at X (4) With M as centre and radius equal to XY draw an arc
(3) X as centre and with the same radius draw two arcs which which meet the previous arc at N.
intersect previous arc at Y and Z such that XY = YZ (5) Join PN and produce ∠QPN is the required angle which is
(4) Taking Y and Z as centres and with a reasonable radius equal to ∠BAC = 75°.
draw two arcs meeting at R 8. Steps of construction:
(5) Join RP which makes an angle 90° at P i.e. ∠RPQ = 90.
Y A X
R

L Y 22½ 30°
Z
P B 10 cm C Q
P X Q

(b) (1) Taking L and X as centres and with a reasonable (1) Draw a line segment PQ = 10 cm.
radius cut two arcs meeting at S  1
(2) \Construct ∠X PQ =  22  ° and ∠Y QP = 30°
(2) Join PS meeting the arc at M  2
(3) ∠SPQ = 45°
(3) Mark the intersecting point of PX and QY as A.
R (4) Draw the perpendicular bisector of PA meeting PQ at B.
(5) Draw the perpendicular bisector of QA meeting PQ at C.
Join AB, AC. Now ABC is the required triangle.
S
x 9. Steps:
L y X
Z A
M
P X Q

6. Steps:
X D

B C

A B
(1) Draw a line segment BC = 5.2 cm.
(2) Construct ∠XBC = 50°
(3) Draw an arc with a radius of 1.5 cm from B which intersect
Y
BX at D 11
(4) Join DC and draw a perpendicular bisector DC to meet
(1) Draw a line segment AB = 7.5 cm BD produced at A.
(2) With A as centre and with radius more than half of AB (5) Join AC, ∆ ABC is the required triangle.
draw two arcs on either side of AB. 10. Steps:
(3) With the same radius as in step 2 with centre B draw two
more arcs to cut the above two arcs at two points X and Y X
P
(4) Join X and Y which is the required perpendicular bisector
of AB.
R
7. Steps:

C
S T
Y N
(1) Draw a line segment ST = 4.2 cm.
(2) Construct ∠X ST = 40°
(3) With S as centre and radius 
equal
 to RS + RT = 7.3 cm,
A draw an arc which intersect SX at P.
X B P M Q
(4) Join PT and draw the perpendicular bisector of PT to meet
(1) Construct an angle BAC = 75° using protractor SP at R.
(2) Draw a line segment PQ with any measurement (5) Join RT, SRT is the required triangle.
Heron's Formula
Answer and Explanations 12
STEPWISE MARKING SOLUTIONS Exercise – 2

1. Sides of the triangle are 8 m, 6 m and 6 m.


Objective Type Questions Let a = 8m, b = 6 m and c = 6m
3 8+6+6
=s = 10 cm
( 4 ) = 4 3 cm 2
2
1. (a) Area = 2
4
3 A = s ( s − a )( s − b)( s − c)  (1 Mark)
( a ) = 16 3
2
2. (b) Area =
4 A= 10(10 − 8)(10 − 6)(10 − 6)
⇒ a=8m A= 10 × 2 × 4 × 4 = 2 × 4 5
∴ Perimeter = 3(8) = 24 m A = 8 5 sq m = 8 × 2.23 sq m
3. True
= 17.84 sq m = 178400 sq cm (1 Mark)
Total cost for planting the grass
4. False 5
= × 178400 = `8920. (1 Mark)
5. True 100
6. 15 cm
2. Given the perimeter of the triangle =42 cm
7. 90 cm2 42
s
= = 21cm
8. Area if a triangle 2
a = 12 cm, 2b = 42 – 12 (as the two other sides are equal).
Subjective Type Questions 2b = 30 ⇒ b = 15
∴ c = 15 (11/2 Marks)
Exercise – 1 A = s ( s − a )( s − b)( s − c)  (1/2 Mark)
1. Let a = 26 cm, b = 28 cm and c = 30 cm = 21(21 − 12)(21 − 15)(21 − 15)
26 + 28 + 30 84
s
= = = 42 cm = 21× 9 × 6 × 6 = 3 × 6 21 = 18 21sq m (1 Mark)
2 2
Area of the triangle 3. Sides of the land (triangle) are 18 m, 24 m and 30 m.
Let a = 18, b = 24 and c = 30
= s ( s − a )( s − b )( s − c )  (1 Mark)
a + b + c 18 + 24 + 30 72
S= = = = 36 m  (1 Mark)
= 42 ( 42 − 26 )( 42 − 28 )( 42 − 30 ) 2 2 2
A = s ( s − a )( s − b)( s − c)
= 42 ×16 ×14 ×12 = 62 × 7 2 × 82 = 6 × 7 × 8 = 336 sq cm.
= 36(36 − 18)(36 − 24)(36 − 30)
 (1 Mark)
2. a = 15 cm, b = 52 cm and c = 41 cm
= 36 × 18 × 12 × 6
15 + 52 + 41 108 = 36 × 6 × 3 × 4 × 3 × 6  (1 Mark)
=s = = 54 cm
2 2 = 6 × 6 × 3 × 2 = 216 sq m
Area of the triangle ∴ Cost of levelling the land = 216 × 30 = `6480 (1 Mark)
= s ( s − a )( s − b)( s − c)  (1 Mark) 4. Join Q and S. ∆PQS is right triangle and PQ = 8 cm and PS = 6 12
cm
= 54 (39) (2) (13) = (13) 2 × 22 × 92
14 cm R
= 13 × 2 × 9 = 234 sq cm (1 Mark)
S
3. a = 25 cm, b = 39 cm and c = 40 cm 12 cm
25 + 39 + 40 104 6 cm
=s = = 52 cm
2 2 P 8 cm Q
Area of the triangle
= s ( s − a )( s − b )( s − c )  (1 Mark) ∴ QS2 = PS2 + PQ2
QS2 = 62 + 82 = 100
= 52 ( 52 − 25 )( 52 − 39 )( 52 − 40 ) = 52 × 27 ×13 ×12 ∴ QS = 10 cm (1 Mark)
Area of ∆PQS
= 42 ×132 × 32 × 32 1 1
= × PQ × PS = × 8 × 6 =24sq m  (1 Mark)
= 4 × 13 × 3 × 3 = 468 sq cm. (1 Mark) 2 2
2 Fundamentals of Mathematics

QR + RS + SQ 3
In ∆RSQ, s = 2. (b) Altitude = a
2 2
12 + 14 + 10 a
= = 18 3. (d) Area = 4b 2 − a 2
2 4
Area of ∆QRS 4. True
= s ( s − a )( s − b)( s − c)
5. False
A= 18(18 − 12)(18 − 14)(18 − 10)
6. True
= 18 × 6 × 4 × 8 = 2 × 9 × 6 × 4 × 4 × 2 7. 10 cm
= 2 × 3 × 4 6 = 24 6 sq cm  (1 Mark)
8. Isosceles
∴ Area of quadrilateral PQRS
9. 27 cm
= Area of ∆PQS + Area of ∆QRS
(
= 24 + 24 6 = 24 1 + 6 sq cm  ) (1 Mark) Subjective Type Questions
5. Exercise – 1
A 1. a = 24 cm, b = 25 cm and c = 7 cm
20 cm 20 cm 24 + 25 + 7 56
s= = = 28cm
O 2 2
B D
15 cm 15 cm Area of the triangle = s ( s − a )( s − b)( s − c)
40 cm 40 cm = 28 ( 28 − 24 )( 28 − 25 )( 28 − 7 )
= 28 × 4 × 3 × 21 = 7 × 4 × 4 × 3 × 7 × 3
C = 7 × 4 × 3 = 84 sq cm.
Let ABCD be the given kite. 2. a = 25 cm, b = 25 cm and c = 30 cm
Let AC and BD intersect at O. 25 + 25 + 30 80
In triangle ABD, s= = = 40 cm
2 2
20 + 20 + 30
=s = 35 Area of the triangle = s ( s − a )( s − b)( s − c) =
2
40 × 15 × 15 × 10
A = s ( s − a )( s − b)( s − c)  (1 Mark)
= 4 × 10 × 10 × 15 × 15
A= 35(35 − 20)(35 − 20)(35 − 30)
= 2 × 10 × 15 = 300 sq cm.
= 35 × 15 × 15 × 5 3. a = 10 cm, b = 17 cm and c = 21 cm
= 5 × 15 7 = 75 7 sq m 10 + 17 + 21 48
s= = = 24 cm
∴ Ar(∆AOB) = Ar (∆AOD) 2 2
75 Area of the triangle = s ( s − a )( s − b)( s − c)
= 7 sq cm  (1 Mark)
2
= 24 ( 24 − 10 )( 24 − 17 )( 24 − 21)
40 + 40 + 30
In ∆BCD, s
= = 55
2 = 24 × 14 × 7 × 3 = 3 × 8 × 2 × 7 × 7 × 3
A = s ( s − a )( s − b)( s − c) = 3 × 3 × 16 × 7 × 7 = 3 × 4 × 7 = 84 sq.cm
A= 55(55 − 40)(55 − 40)(55 − 30)  (1 Mark) 4. Let a = 13 cm, b = 21 cm and c = 20 cm
a + b + c 13 + 21 + 20 54
= 55(15)(15)(25) s= = = = 27 cm
2 2 2
= 5 × 15 55 = 75 55 sq cm
Area of the triangle = s ( s − a )( s − b)( s − c)
Area (∆BOC) = Area (∆DOC)
= 27 ( 27 − 13)( 27 − 21)( 27 − 20 )
75 55
= sq cm  (1 Mark)
2 = 27 × 14 × 6 × 7
= 9 × 3× 2 × 7 × 3× 2 × 7
PROBLEMS FOR ADDITIONAL PRACTICE = 9×9× 2× 2×7×7
= 9 × 2 × 7 = 126 sq cm.
Objective Type Questions 5. Let a = 25 cm, b = 39 cm and c = 56 cm
1 25 + 39 + 56 120
1. (c) Area = × 6 × 6 =18cm 2 s= = = 60 cm
2 2 2
Answer and Explanations 3

Total cost of polishing at the rate of 0.25 sq cm


Area of the triangle = s ( s − a )( s − b)( s − c)
= 7200 × 0.25 = `1800
= 60 ( 60 − 25 )( 60 − 39 )( 60 − 56 ) 4. Area of ∆ABC
= 60 × 35 × 21× 4 = 4 × 15 × 7 × 5 × 3 × 7 × 4 s ( s − a )( s − b)( s − c)
= 4 × 3× 5× 7 × 5× 3× 7 × 4 AB + BC + CA 9 + 10 + 13
Where s = = = 16
= 4 × 3 × 5 × 7 = 420 sq cm. 2 2
6. Let a = 17 cm, b = 25 cm and c = 28 cm Area = 16 (16 − 9 )(16 − 10 )(16 − 13)
17 + 25 + 28 70
s= = = 35cm = 16 × 7 × 6 × 3 = 4 × 3 × 14 = 12 14 sq m
2 2
In triangle ACD
Area of the triangle = s ( s − a )( s − b)( s − c) AC + CD + DA 13 + 7 + 10
s= = = 15
= 35 ( 35 − 17 )( 35 − 25 )( 35 − 28 ) 2 2
D 7m C
= 35 × 18 × 10 × 7
= 7 × 5× 2× 9× 2× 5× 7 13 m
10 m 10 m
= 7 × 5 × 2 × 3 = 210 sq cm.

Exercise – 2
A 9m B
1. a + b + c = 84 cm, a = 28 cm, b = 15 cm
∴ c = 84 – (28 + 15) Area = s ( s − a )( s − b)( s − c)
84
s = 42 cm
c = 41 cm, = = 15 (15 − 13)(15 − 7 )(15 − 10 )
2
Area of the triangle = s ( s − a )( s − b )( s − c ) = 15 × 2 × 8 × 5 = 5 × 4 3 = 20 3 sq m .
∴ Area of the quadrilateral
A= 42 ( 42 − 41)( 42 − 28 )( 42 − 15 )
(
= 12 14 + 20 3 = 4 3 14 + 5 3 sq m . )
= 42 × 1× 14 × 27
5. a = 1.2 b = 1.8 m, c = 1.2 m
= 14 × 3 × 14 × 9 × 3 1.2 + 1.8 + 1.2 4.2
= 14 × 3 × 3 = 126 sq cm. s= = = 2.1cm
2 2
2. Each diagonal of a rhombus divides the rhombus into two A = s ( s − a )( s − b)( s − c)
congruent triangles.
A= 2.1(2.1 − 1.2)(2.1 − 1.8)(2.1 − 1.2)
D
= 2.1× 0.9 × 0.3 × 0.9 = 0.3 × 0.9 7 = 0.27 7 sq m
45 m 45 Area of the card board = 0.27 7 × 100 × 100sq m

A 30 C = 2700 7 sq cm = 2700 × 2.65 sq m = 7155 sq cm


Cost of decoration per 100 sq cm = `10
45 45 7155
∴ Total cost of decorating the card board = × 10
100
45 + 45 + 30 120 = ₹715.5 12
s= = = 60
2 2 Exercise – 3
Area of the rhombus shaped field
1. Perimeter of an equilateral triangle = 3a = 1050 cm
= 2 × s ( s − a )( s − b )( s − c )
1050
∴ Length of each side (a) = = 350 cm
= 2 60 ( 60 − 45 )( 60 − 45 )( 60 − 30 ) 3
Area of a triangle by Heron’s formula
= 2 × 60 × 15 × 15 × 30 = 2 × 15 × 30 2 = 900 2 sq m
A= s ( s − a )( s − b)( s − c)
3. Sides of each tile are 15 cm, 20 cm and 25 cm.
Let a = 15 cm, b = 20 cm, and c = 25 cm
a + b + c 1050
s=
 = = 525 , a = b = c = 350
a+b+c 60 2 2
s= = = 30
2 2 A= 525 ( 525 − 350 )( 525 − 350 )( 525 − 350 )
A = s ( s − a )( s − b)( s − c) = 525 × 175 × 175 × 175
= 30 (15 )(10 )( 5 ) = 15 × 10 = 150 sq cm = 175 × 3 × 175 × 175 × 175
Area of the 48 tiles = 48 × 150 = 7200 sq cm = 175 × 175 3 sq cm = 30625 3 sq cm .
4 Fundamentals of Mathematics

2. Let the sides of the triangle be 6x, 5x, 4x respectively perimeter 12 + 14 + 16


of the triangle = 600 cm. =5. s = 21
2
P = 6x + 5x + 4x = 600
⇒ 15x = 600 Area of the triangle, A = s ( s − a )( s − b)( s − c)
x = 40 = 21( 21 − 12 )( 21 − 14 )( 21 − 16 )
∴ The sides of the triangle are
a = 6 × 40 = 240 cm = 21× 9 × 7 × 5 = 3 × 7 × 9 × 7 × 5
b = 5 × 40 = 200 cm
= 3 × 7 × 15 = 21 15 sq cm
c = 4 × 40 = 160 cm
600 ∴ Area of the parallelogram = 2 × 21 15 sq cm
=s = 300
2 base× height = 42 15
A = s ( s − a )( s − b)( s − c) 16 × h =42 15
= 300(300 − 240)(300 − 200)(300 − 160) 42
h= 15
= 300 × 60 × 100 × 140 = 6000 7 sq cm 16
21
3. a = 30, b = 40, c = 50 ⇒ h= 15 cm .
30 + 40 + 50 120 8
s= = = 60
2 2 HOTS
Area of each triangle piece,
1. Let the sides of the triangle be 2k, 3k and 4k.
A= s ( s − a )( s − b)( s − c)
Given, Perimeter of the triangle = 36 cm
= 60(60 − 30)(60 − 40)(60 − 50) ⇒ 2k + 3k + 4k = 9k = 36 ⇒.
= 60 × 30 × 20 × 10 =
600sq cm ∴ Sides of the triangle are ( 2 × 4 ) cm, ( 3 × 4 ) cm, ( 4 × 4 ) cm

Total area of the tent = 300 × 600 sq cm. i.e., 8cm, 12cm, 16cm.
= 180000 sq cm Here, a = 8 cm, b = 12 cm, c = 16 cm
Total cloth required (in sq m) a + b + c 8 + 12 + 16
Semi perimeter (s) . = = 18
180000 2 2
= = 18sq m
10000 ∴ Area of the tringle = s ( s − a )( s − b )( s − c )
4. Mark the point E on AB such that CE is parallel to AD
AECD is a parallelogram
= 18 (18 − 8 )(18 − 12 )(18 − 16 )

D 90 cm C = 18 × 10 × 6 × 2
= 6 × 3× 2 × 5× 6 × 2

50 cm = 12 15 sq cm .
40 cm 40 cm
2. Let the equal sides be ‘x’ cm each
Given, base of the isosceles triangle = (x + 5) cm
Perimeter of the isosceles triangle = 26 cm
A 90 cm E F B ⇒ x + x + x + 5 = 26
30 cm ⇒ 3x + 5 = 26
⇒ 3x = 21
In ∆BEC,BE = 30 cm, BC = 50 cm, CE = 40 cm
∴ x=7
30 + 50 + 40 Let the sides of the isosceles triangle be a, b and c
=s = 60
2 ∴ The sides of the isosceles triangle are 7 cm, 7 cm and 12 cm.
A = s ( s − a )( s − b)( s − c) 7 + 7 +12 26
Semi perimeter (s) = = = 13
= 60 ( 60 − 30 )( 60 − 50 )( 60 − 40 ) 2 2
∴ Area of the isosceles triangle = s ( s − a )( s − b )( s − c )
= 60 × 30 × 10 × 20 = 20 × 3 × 3 × 10 × 10 × 20
= 20 × 3 × 10 = 600 sq cm = 13 (13 − 7 )(13 − 7 )(13 − 12 )
In ∆BEC,
BE2 + EC2 = BC2
= 13 × 6 × 6 × 1
∠BEC = 90° = 6 13 sq cm .
∴ ∠EAD = 90°
3. Let the three sides of the triangle be a, b and c.
∴ AECD is a rectangle.
Given,
⇒ Area of rectangle AECD = AE × AD
a + b = 2c  ... (1) ⇒ a + b = 30 – c
= 90 × 40 = 3600 sq cm
b + c = 2a - ... (2) ⇒ b + c = 30 – a
∴ Area of the trapezium = Ar (∆BEC) + Ar (rectangle
c + a = 2b - ... (3) ⇒ c + a = 30 – b
AECD) = 600 + 3600 = 4200 sq cm
From (1), 30 – c = 2c
Answer and Explanations 5

⇒ 3c = 30 7 + 6 + 5 18
⇒ c = 10 cm Semi perimeter (s) = = = 9
From (2), 30 – a = 2a 2 2
⇒ 3a = 30 ∴ Area of the triangle = s ( s − a )( s − b )( s − c )
⇒ a = 10 cm
From (3), we get = 9 ( 9 − 7 )( 9 − 6 )( 9 − 5 )
c = 10 cm
= 9 × 2 × 3× 4
= 36 × 6= 6 6 sq.cm
10 +10 +10 30
Semi perimeter (s)= = = 15
2 2 6. Given, ∠ABC = 90°, AB = 12 cm, BC = 5 cm, CD = 11 cm, AD
= 8 cm.
∴ Area of the triangle = s ( s − a )( s − b )( s − c )
A 12 cm B
= 15 (15 − 10 )(15 − 10 )(15 − 10 )

= 15 ( 5 )( 5 )( 5 ) = 25 3 sq.cm
8 cm 5 cm
a+b =
4. Given, 22 cm  ... (1)
b+c =30 cm  ... (2)
a+c =28 cm  ... (3) C
Adding (1), (2) and (3), we get 11 cm
D
2(a + b + c) =80 cm
80 DABC is a right-angled triangle.
∴ a+b+c= = 40 ∴ By Pythagoras theorem
2
(12 )
2
a+b+c 40 AC = + 52= 169= 13 cm
s= = = 20 cm
2 2 Area of quadrilateral ABCD = Area of triangle ABC +
a+b+c = 40 Area of triangle ADC
⇒ 22 + c = 40 [∵ From (1)] The sides of the triangle ABC are 12 cm, 5 cm and 13 cm.
∴ c = 40 − 22 = 18 cm 12 + 5 + 13 30
s
= = = 15
b+c = 3 2 2
⇒ b 30 − 18
= ∴ Area of DABC = 15 (15 − 12 )(15 − 5 )(15 − 13)
∴ b = 12 cm
= 15 ( 3)(10 )( 2 )
a+c = 28 cm
⇒ a 28 − c
= = 5× 3× 3× 5× 2 × 2
⇒ a 28 − 18
= = 30 sq cm
∴ a = 10 cm The sides of the triangle ADC are 8 cm, 11 cm and 13 cm.
Here, a = 10 cm, b = 12 cm and c = 18 cm 8 + 11 + 13 32
s
= = = 16
∴ Area of the triangle = s ( s − a )( s − b )( s − c ) 2 2
∴ Area of DADC = 16 (16 − 8 )(16 − 11)(16 − 13)
= 20 ( 20 − 10 )( 20 − 12 )( 20 − 18 )
= 16 ( 8 )( 5 )( 3)
= 20 × 10 × 8 × 2
= 2 × 10 × 10 × 4 × 2 × 2 = 4× 4× 2× 2× 2× 5× 3
= 4 120 =4 4 × 30 =4 × 2 30 12
= 40 2 sq.cm
= 8 30 sq cm
5. Let the longest side be ‘x’ cm
Given, shortest side = ( x − 2 ) cm ∴ (
Area of quadrilateral ABCD = 30 + 8 30 sq cm . )
Third side = ( x − 1) cm 7. The 3 medians of the triangle divide it into 6 triangles of equal
Perimeter of the triangle = 18 cm areas.
⇒ x + x −1+ x − 2 = 18 A
⇒ 3x − 3 = 18
21
⇒ x=
3 F E
∴ x = 7 cm
∴ The sides of the triangle are 7 cm, (7–1) cm and (7–2) cm
Let the sides of the triangle be a, b and c B D C
Here, a = 7 cm, b = 6cm, c = 5cm
6 Fundamentals of Mathematics

We have a = 40 m, b = 60 m and c = 80 m
a + b + c 40 + 60 + 80 180 = ( 36 − 12 )(12 − 4 ) = 24 × 8
s= = = = 90
2 2 2 = 3× 2 × 4 × 4 × 2
∴ Area of the triangular field = 2× 2× 2× 3 = 8 3 sq cm.
= 90 ( 90 − 40 )( 90 − 60 )( 90 − 80 )
10. Area of quadrilateral ABCD = Area of DABC + Area of DADC
= 90 ( 50 )( 30 )(10 ) = 900 × 15 × 100
A
= 30 × 10 15 =
300 15 sq cm
8 cm
∴ The area in which each types of the flower plant can be
B
300 15 9 cm
grown = = 50 15 sq m .
6 6 cm
8. Let the sides of the triangle be a, b and c
a = 100 m, b = 80 m, c = 60 m C
a + b + c 240 D 9 cm
=s = = 120
2 2
∴ Area of park = 120 (120 − 100 )(120 − 80 )(120 − 60 ) Since, ABC is right angled triangle,

AC = 82 + 62= 100= 10 cm
= 120 ( 20 )( 40 )( 60 ) = 2400 × 2400 =
2400 sq cm
In DABC, a = 8 cm, b = 6 cm, c = 10 cm
Perimeter of the park = 240 m. 8 + 6 +10 24
Since, 5 m is left for the gate, we have s
= = = 12
Cost of fencing at ₹25/m = (240 – 5) × ₹ 25 = ₹ 5875 2 2
9. Given, AB = 4 3 , ∠BAC = 30°, ∠ACB = 60° ∴ Area of DABC = 12 (12 − 8 )(12 − 6 )(12 − 10 )
A = 12 ( 4 )( 6 )( 2 )

30° = 6× 2× 2× 2×6× 2

= 62 × 24 =6 × 22 =24 sq cm .
In DADC, a = 9 cm, b = 9 cm, c = 10 cm
9 + 9 +10 28
s
=
 = = 14
60° 2 2
B C ∴ Area of DABC = 14 (14 − 9 )(14 − 9 )(14 − 10 )
The ratio of the sides of a right triangle with angles 30°, 60°, = 14 ( 5 )( 5 )( 4 )
90° is 1: 3 : 2
Let the sides of the DABC be k, k 3, 2k = 5 × 2 14 =
10 14 sq cm .

Now, k 3= 4 3 ⇒ k= 4 ∴ Area of quadrilateral ABCD = 24 sq cm + 10 14 sq cm

Sides of the right triangle are 4 cm, 4 3 cm , 8 cm (


= 24 + 10 14 sq cm )
Let the sides of the triangle be a, b and c
11. Let the side of the park be ‘a’ m
Here, a = 4 cm, b = 4 3 cm , c = 8 cm
Perimeter of the park = a + a + a = 3a = 120

s=
4 + 8 + 4 3 12 + 4 3
= =
(
4 3+ 3 ) =
120
= 40 m
2 3 2 3
(
= 2 3+ 3 =
6+2 3 ) Semi perimeter =
120
2
= 60
∴ Area of DABC = s ( s − a )( s − b )( s − c ) ∴ Area of the park = 60 ( 60 − 40 )( 60 − 40 )( 60 − 40 )
=
= 60 × 20 × 20 × 20
( )(
(6 + 2 3) 6 + 2 3 − 4 6 + 2 3 − 4 3 6 + 2 3 − 8 )( ) = 20 × 3 × 20 × 20 × 20
= ( 6 + 2 3 )( 2 + 2 3 )( 6 − 2 3 )( 2 3−2 ) = ( 20 ) × ( 20 )
2 2
×3

= ( 6 + 2 3 )( 6 − 2 3 )( 2 + 2 3 )( 2 3 − 2) = 20 × 20 3
= 400 3 sq cm .
Answer and Explanations 7

12. Given, ABC is right angled isosceles triangle.


Area of DABD = s ( s − a )( s − b )( s − c )
A
= 28 ( 28 − 20 )( 28 − 20 )( 28 − 16 )

= 28 ( 8 )( 8 )(12 )
x = 7 × 4 × 8 × 8 × 4 × 3 = 82 × 4 2 × 7 × 3
= 8 × 4 21 = 32 21 sq m .
∴ Area of the field = 2 × 32 21 sq m = 64 21 sq m .

B x C
14. In the figure, BE is drawn parallel to AD
By Pythagoras theorem, A 12 m B
2 2 2
AC = x +x = 2 x = x 2 cm
8m
( ) 8m 10m
x + x + x 2 2x + x 2 x 2 + 2
s= = =
2 2 2 D 12 m E F C
∴ Area of triangle 4m

 2x + x 2   2x + x 2  ∴ ABED is a parallelogram
   − x  AB = DE = 12 m
 2  2 
= AD = BE = 8m
 2x + x 2   2x + x 2  In DBCE, CE = 16 – DE = 16 – 12 = 4 m
 − x   − x 2  BE = 8 m, BC = 10 m
 2  2  ∴ In DBCE, a = 4 m, b = 8 m, c = 10 m
 2x + x 2   x 2   x 2   2x − x 2  4 + 8 + 10 22
s
= = = 11
=         2 2
 2
  2  2  2 
∴ Area of DBCE = s ( s − a )( s − b )( s − c )
2
x 2  2x + x 2   2x − x 2 
=       = 11(11 − 4 )(11 − 8 )(11 − 10 )

 2   2  2 
= 11( 7 )( 3)(1)
2
 x 2   4x2 − 2x2 
=  = 231 sq m
 2   4

    Let BF be the altitude of DBCE
2 1
 x 2   2x  2x 2x2
2x x 2 2 2 2
∴ Area of DBCE = × CE × BF
 2   4  = 4 × 4 = 4 = 2 sq cm.
=  2
    1
⇒ × 4 × BF = 231
13. Area of rhombus, ABCD = 2 × Area of DABD 2
A 231
∴ BF = m
2
∴  rea of trapezium = Area of parallelogram ABED + Area
A
20˝m 20˝m of DBCE 12
= DE × BF + 231 sq m
16˝m
B D 231
= 12 × + 231 sq m
2
= 6 231 + 231 =7 231 sq m .
20˝m 20˝m

15. Area of parallelogram ABCD = 2 × Area of DABD


C A 10 cm B
(∵ Diagonal BD divides ABCD into two equal triangles)
16 cm
In DABD, AB = AD = 20m, BD = 16 m 8 cm
Here, a = 20 m, b = 20 m, c = 16 m
20 + 20 +16 56
s
= = = 28 C
2 2 D
8 Fundamentals of Mathematics

(∵ Diagonal divides the parallelogram into two triangles of 3× 6 9


equal area) = 3= 3 sq cm .
In DABD, a = 10 cm, b = 8 cm, c = 16 cm 4 2
10 + 8 + 16 34 17. Let the sides of the triangle be a, b and c
s
= = = 17cm
2 2 Given, a = 3 3 cm , b = 6 3 cm , c = 5 3 cm
∴ Area of DBCE = s ( s − a )( s − b )( s − c ) 3 3 + 6 3 + 5 3 14 3
s= = =7 3
2 2
= 17 (17 − 8 )(17 − 10 )(17 − 16 )
∴ Area of triangle = s ( s − a )( s − b )( s − c )
= 17 ( 9 )( 7 )(1)
= 3 119 sq m
= ( )(
7 3 7 3 −3 3 7 3 −6 3 7 3 −5 3 )( )
∴ Area of parallelogram ABCD = 2 × 3 119 = 7 3 ( 4 3 )( 3 )( 2 3 )
= 6 119 sq cm . = 7 × 4 × 2 × 3× 3
16. Given, ABC is an equilateral triangle.
= 7 × 22 × 32 × 2 = 2 × 3 14 = 6 14 sq cm .
$
18. Given, ABC is an isosceles triangle.

A
FP FP
 FP
6 cm

% FP ' FP &

AB =BC = AC = 6 cm
The altitude AD bisects the base BC B D C
BCD 6 8 cm
∴ DC = = = 3cm
2C 2 Let AB =BC = x cm, BC = 8 cm
In DADC, a = 3 3 cm , b = 6 cm, c = 3 cm By Pythagoras theorem,

( )
2
3 3 +6+3 9+3 3 AC = 42 + 2 5 =36 =
6 cm
=s =
2 2
In the figure, AD is the median
∴ Area of DADC = s ( s − a )( s − b )( s − c ) 8
∴ DC = = 4 cm (∵ The median drawn from any vertex
= 2
 9 + 3 3  9 + 3 3  9 + 3 3  9 + 3 3  of the triangle bisects the opposite side)
   − 3 3   − 6   − 3  Let a = 8 cm, b = 6 cm, c = 6 cm
 2  2  2  2  8 + 6 + 6 20
s
= = = 10
 9 + 3 3  9 − 3 3  3 3 − 3  3 3 + 3  2 2
=         Area of DABC = s ( s − a )( s − b )( s − c )
 2  2  2  2 
 81 − 27   27 − 9  = 10 (10 − 8 )(10 − 6 )(10 − 6 )
=   
 4  4  = 10 ( 2 )( 4 )( 4 )
54 18 9× 6× 6×3 = 8 5 sq cm.
= × =
4 4 16
Surface Areas and Volumes
Answer and Explanations 13
STEPWISE MARKING SOLUTIONS 5. Curved surface area of the cone = πrℓ (1/2 Mark)
22 4.2 22
= × × 10 = × 2.1 × 10
Objective Type Questions 7 2 7
= 22 × 0.3 × 10 = 66 sq. cm.(11/2 Marks)
volume 216π
1. (c) Height = = =6 6. Volume of each cubical box
base area 36π
= 5 × 5 × 15 = 125 cm3. (1 Mark)
Base area = πr2 = 36π ∴ volume of the box containing 8 such boxes = 8 × 125
r=6 = 1000 cm3. (1 Mark)
⇒ CSA = 2πrh = 2π 6 × 6
= 72π sq cm Exercise 2
2. (b) Surface area of a sphere = 4πr 2
1. ℓ = 32 cm, b = 28 cm, h = 35 cm
= 4π × (3.5)2 = 49π sq cm Area of the glass = 2(ℓb + bh + hℓ) (1 Mark)
3. True = 2 (32 × 28 + 28 × 35 + 35 × 32)
= 2 (896 + 980 + 1120) = 5992 sq. cm. (1 Mark)
4. False Length of the tape = 4 × ℓ + 4 × b + 4 × h
5. True = 4 × 32 + 4 × 28 + 4 × 35
= 128 + 112 + 140 = 380 cm.  (1 Mark)
6. Three
2. Total surface area of a cylinder
7. Capacity
= 2πr (h + r) (1 Mark)
8. 2772 cm2 22 7  7
=2× × 19 + 
7 2 2
Subjective Type Questions 45
 
= 22   = 495 sq. cm (1 Mark)
Exercise 1  2
Let x be the paper used.
1. Area of four walls = 2h(ℓ + b)(1/2 Mark)
495 sq. cm is equal to 90% of the paper used.
ℓ = 15 m, b = 12 m, h = 5 m
90
A = 2 × 5 (15 + 12) = 10 (27) ⇒ x = 495
A = 270 sq. m. (1 Mark) 100
∴ Cost of painting the 4 walls x = 550 sq. cm. (1 Mark)
= 270 × 25 = Rs.6750.(1/2 Mark) 3. Surface area of a sphere = 4πr 2

2. Total surface area of cylinder (i) r = 6.3 cm


= 4752 sq. cm, r = 21 cm. 22
∴ Surface area = 4 × × 6.3 × 6.3
2πr (h + r) = 4752 7
22 = 4 × 22 × 0.9 × 6.3
2× × 21 (h + 21) = 4752 (1 Mark)
7 = 88 × 0.9 × 6.3 = 498.96 sq cm.
132 (h + 21) = 4752 (ii) r = 14.7 cm
h + 21 = 36 22
∴ Surface area = 4 × × 14.7 × 14.7
h = 36 – 21 7
h = 15 cm. (1 Mark) = 4 × 22 × 2.1 × 14.7
h
3. Given r = = 2716.56 sq cm
2 (iii) r = 21 cm
Total surface area of a cylinder
= 2πr (h + r) 22 13
∴ Surface area = 4 × × 21 × 21
22 h  h 7
=2× × × h+   (1 Mark)
7 2  2 = 4 × 22 × 3 × 21 = 5544 sq. m.
2
22  3h  33h 4. Given, 4πr12 : 4πr22 = 9 : 16
= × h   = 231 sq. cm =
7  7  7 ⇒ r12 : r22 = 9 : 16 (1 Mark)
2
= 231 r12 9 (r1 ) 2  3 
⇒ = ⇒ = 
⇒ h2 = 7 × 7 ⇒ h = 7 cm (1 Mark) r22 16 (r2 ) 2  4 
4. Curved surface area of water tank = 2πrh r1 3
⇒ =
22 4.2 r2 4
=2× × × 19.6 (1/2 Mark)
7 2 ⇒ r1 : r2 = 3 : 4 (1 Mark)
= 2 × 22 × 2.1 × 2.8 Ratio of their diameters = 2r1 : 2r2
= 258.72 sq. m.(11/2 Marks)
2 Fundamentals of Mathematics

⇒ 2r1 : 2r2 = 2 × 3 : 2 × 4 = 3 : 4. (1 Mark) Ratio of their volumes


5. Volume of cuboid = ℓbh = 5 × 3 × 2.5 = 37.5 cm3
4 3 4
= πr : π × 64r3
 (2 Marks) 3 3
Volume of 15 cuboids = 15 × 37.5 ⇒ 1 : 64.(11/2 Marks)
= 562.5 cm3. (1 Mark)
Exercise 3
6. Number of detergent boxes
80 × 40 × 40 1. Length of the table = 1.5 m.
= = 1600 Breadth of the table = 0.7 m.
8×5× 2
Height of the table = 0.8 m
∴ The maximum number of detergent boxes = 1600.
Area of the top face = 1.5 × 0.7
 (3 Marks)
= 1.05 sq. m.
7. (i) Given, 2πrh = 1848 Area of the two faces (on the left and right) (1 Mark)
Where, r = radius of the base =2×b×h
h = height of the cylinder = 2 × 0.7 × 0.8 = 1.12 sq. m. (1 Mark)
22 Area of the back face = ℓ × h
2× × r × 21 = 1848 (1 Mark) = 1.5 × 0.8 = 1.2 sq. m. (1 Mark)
7
∴ Total area of the decolam sheet required
⇒ 2 × 22 × r × 3 = 1848
= 1.05 + 1.12 + 1.2
1848
r= = 14 cm (1 Mark) = 3.37 sq m. (1 Mark)
2 × 22 × 3
2. (i) h = 3 m, ℓ = 5 m
(ii) Volume of the cylinder = πr2h
22 ℓ = h2 + r2 ⇒ r =
2
2 − h2
= × 14 × 14 × 21 (1 Mark)
7 = 52 − 32 = 4 (1 Mark)
= 22 × 2 × 14 × 21 Surface area of the heap = πrℓ
= 22 × 28 × 21 = 12936 cm3  (1 Mark) 22 440
= ×4×5= sq m = 62.85 sq m (1 Mark)
1 2 7 7
8. Volume of right circular cone = πr h
3 (ii) The total quantity of water
(i) r = 9 cm, h = 21 cm = 120 cm × 90 cm × 1.2 km × 12 hr (1 Mark)
1 22 = 120 × 90 × 120000 cm × 12
∴  olume =
V × × 9 × 9 × 21 = 1782 cm3 = 15552000000 cm3
3 7
= 15552000 ℓ = 15552 kℓ (1 Mark)

(11/2 Marks)
(ii) r = 7 cm, h = 14 cm 3. (i) Volume of juice in tin ℓbh
1 22 = 15 × 10 × 20= 3000 cm3 (1 Mark)
∴ Volume = × × 7 × 7 × 14 = 718.67 (ii) Volume of juice in cylinder
3 7
= π × 3.5 × 3.5 × 20
 (11/2 Marks)
= 22/7 × 3.5 × 3.5 × 20  (1 Mark)
π 2 = 22 × 0.5 × 3.5 × 20
9. Volume of conical pit = rh
3 = 770 cm3 (1 Mark)
 (1 Mark) ∴ Difference of their capacities
π 22 = 3000 – 770 = 2230 cm3. (1 Mark)
= (3.5)2 × 10 = × 3.5 × 3.5 × 10
3 7 1 2
4. Volume of right circular cone = πr h (1 Mark)
220 × 12.25 2695.00 3
= = π 2
21 21 ⇒ r h = 154 π (1 Mark)
= 128.33 m (∵ 1 m = 1 kilolitre)
3 3 3
= 128.33 kilolitres. (2 Marks) r2h = 154 × 3 ⇒ r2 × 14 = 154 × 3
r2 = 11 × 3r2 = 33 (1 Mark)
10. Radius of the moon = r
Radius of the earth = R ⇒ r = 33
R ∴ Diameter = 2 × 33
Given, r = ⇒ R = 4r
4 = 2 33 cm. (1 Mark)
4 3
Volume of the moon = πr 5. When the triangle revolved about the side 7 cm,
3
4 3 P
Volume of the earth = πR (11/2 Marks)
3
25 cm
4 24 cm
= π(4r)3
3
4 R 7 cm Q
= × π × 64 × r3 (1 Mark)
3
Answer and Explanations 3

1 1 22 6. Four
volume = × π × r2 × h = × × 7 × 24 × 24 = 4224 cm3
3 3 7 7. 2(lb + bh + hl)
 (1 Mark) 8. Cuboid
When the triangle the revolved about the side 24 cm,
1 1
volume = × π × R2 × h = × π × 7 × 7 × 24 (2 Marks) Subjective Type Questions
3 3
1 22 Exercise 1
× × 7 × 7 × 24 = 1232 cm3
3 7 1. ℓ = 48 c, b = 30 cm, h = 25 cm
∴ The ratio of their volumes = 24 : 7. (1 Mark) A = 2 (ℓb + bh + hℓ)
6. Internal curved surface area of the hemisphere = 2πr 2 = 2(48 × 30 + 30 × 25 + 25 × 48)
Given, 2πr2 × 4 = 4928 = 2(1440 + 750 + 1200)
22 = 2 × 3390
2× × r2 × 4 = 4928 = 6780 cm2
7 ∴ Area of the sheet = 6780 cm2.
4928 × 7 56 × 7
r2 = = = 28 × 7 2. Total surface area of the brick = 2 (ℓb + bh + hℓ).
88 × 2 2 = 2 (21 × 10 + 10 × 7 + 7 × 21)
r2 = 14 × 14 = 2(210 + 70 + 147)
⇒ r = 14 m (11/2 Marks) = 2 × 427 = 854 sq cm
(i) The inner surface area of the dome = 2πr2 Area of 100 bricks = 854 × 100
22 = 85400 sq cm
=2× × 14 × 14
7 85400
= sq m = 8.54 sq m
= 44 × 2 × 14 100 × 100
= 44 × 28 = 1232 m2 (11/2 Marks) 8.54
(ii) volume of the air inside the dome Number of tins required = = 14.
0.61
1 4 3
= × πr 3. Total surface area of a cylinder = 2πr (h + r)
2 3
Diameter of the cylindrical log = 70 cm
1 4 22 ∴ Radius (r) = 35 cm.
= × × × 14 × 14 × 14
2 3 7 Height of the log = (h) = 6 m = 600 cm.
44 22
= × 28 × 14 Total surface area of the log = 2 × × 35 (600 + 35)
3 7
17248 = 44 × 5 × 635 = 220 × 635 = 139700 sq cm
= = 5749.33 m3 (1 Marks) 1.5
3 Cost of applying wood primer = 139700 ×
7. Shape of capsule is sphere.
1000
Radius R of the sphere = 3.5 mm(11/2 Marks) = ` 209.55
Drug needed to fill the capsule completely = volume of the 22  4.2 
4 3 4. Curved surface area of pipe = 2 × ×  × 3.5 × 100 cm
sphere = π r (11/2 Marks) 7  2 
3 22
4 22 =2× × 2.1 × 3.5 × 100 cm = 46.2 × 100 cm = 4620 sq cm.
= × × 3.5 × 3.5 × 3.5 7
3 7 ∴ Area of the surface to be painted = 4620 sq cm.
88
= × 0.5 × 3.5 × 3.5 = 179.66 mm3 (1 Mark) 5. Curved surface area of a cone = 165 m2
3 ℓ = 10 cm
πrℓ = 165
PROBLEMS FOR ADDITIONAL PRACTICE 22
× r × 10 = 165 13
7
Objective Type Questions 165 × 7
r=
1. (b) Volume of a cylinder = Area of base × h 22 × 10
49 ∴ r = 5.25 cm
= π × 10 = 4.9π cu cm
100 Total surface area = πr (ℓ + r)
2. (d) Volume = lbh 22
= × 5.25 (10 + 5.25) = 251.625 sq cm
v 640 7
= = 16 cm
l=
bh 4 × 10 6. Total surface area of a hemisphere = 3πr2
3. True 22
=3× × 15 × 15
4. True
7
= 3 × 3.14 × 225 = 2119.50 sq.m.
5. False
4 Fundamentals of Mathematics

7. Area of the four walls = 2h (ℓ + b) Exercise 2


Cost of white washing the four walls = 9000
Height of the wall = 3 m and cost per m2 = ` 12 1. (i) Total surface area of a cube = 6 × a2
12 × 2 × 3 (ℓ + b) = 9000 = 6 × 122
9000 = 6 × 144 = 864 sq m.
2 (ℓ + b) = = 250 m. Total surface area of a cuboid = 2 (ℓb + bh + hℓ)
36 = 2 × (15 × 12 + 12 × 8 + 8 × 15)
8. Height of the cylinder = 1.2 m. = 2 × (180 + 96 + 120) = 2 × 396 = 792 sq m.
Diameter of the drum = 0.7 m. The cubical box has the greater total surface area it is (864
∴ Radius = 0.35 m – 792) = 72 sq cm more.
Total surface area of the drum = 2πr(h + r) Lateral surface area of cube = 4a2
22 = 4 × 122
= 2 × × 0.35 (1.2 + 0.35) = 44 × 0.05(1.55) = 4 × 144 = 576 sq m.
7
Lateral surface area of cube = 2h (ℓ + b)
= 2.2 × 1.55 = 3.41 sq m.
= 2 × 8 (15 + 12) = 16 × 27 = 432 sq m.
9. (1) Depth of the circular tank = 14 m. ∴  The cuboidal box has the smaller lateral surface area and
Inner diameter = 2.8 m it is (576 – 432) = 144 sq cm less.
Inner radius = 1.4 m.
2. (1) Inner diameter of cross section of the pipe = 5 cm.
Inner curved surface area = 2πrh
∴ Inner radius = 2.5 cm
22 Inner curved surface area = 2πrh = 1320 sq cm
2πrh = 2 × × 1.4 × 14 = 123.2 sq m.
7 22
(2) Cost of painting = 2.8 per m2 2× × 2.5 × h = 1320
7
Total cost of painting the curved surface area
1320 × 7
= 123 .2 × 2.8 = 344.96 sq m h=
2 × 2 × 2.5
10. h = 16 m, r = 12 m
h = 84 cm
ℓ= h 2 + r 2 = 162 + 122 ∴ Length of the pipe = 84 cm.
(2) Outer curved surface area
= 256 + 144
Length = 84 cm = h
∴ ℓ = 400 = 20 Diameter = 5 cm + 2 × 0.3 = 5.6 cm
Curved surface area cone = πrℓ radius = 2.8 cm = R
22 22
= × 12 × 20 = 754.28 sq m 2πRh = 2 × × 2.8 × 84
7 7
11. Curved surface area of a hemisphere = 2πr2
= 1478.4 sq cm.
r = 14 cm (3) Total surface area
Inner curved surface area + Outer curved surface area +
22
∴ 2πr2 = 2 × × 14 × 14 Base rings on both sides.
7 = 1320 + 1478.4 + 2 π (R2 – ν2)
= 2 × 22 × 2 × 14 = 44 × 28 = 1232 sq.cm. 22
∴ Curved surface area = 1232 sq.cm. = 1320 + 1478.4 + 2 × × (2.8 + 2.5) (2.8 – 2.5)
7
4 3 44
12. Volume of a sphere = πr = 2798.4 + × 5.3 × 0.3
3 7
(i) Diameter = 28 cm = 2798.4 + 9.99 = 2808.39 sq cm
⇒ Radius r = 14 cm
4 22 3. Surface area of 20 cones = 20 × πrℓ
∴ Volume of the sphere =
× × 14 × 14 × 14 44
3 7 = 20 × × 1.4 × 2.1
4 × 22 × 2 × 14 × 14 34496 7
= = = 11498.66 cm3 = 440 × 0.42 = 184.80 sq m.
3 3
(ii) Diameter = 0.49 m 4. r1 = 3.5 cm and r2 = 7.7 cm
0.49 m Ratio of surface area of a sphere = 4πr12 : 4πr22
⇒ Radius = = 4 × π × (3.5)2 : 4 π (7.7)2
2 = (3.5)2 : (7.7)2
3
4 22  0.49  = 3.5 × 3.5 : 7.7 × 7.7
∴ Volume of the sphere =
× × 
3 7  2  = 0.5 × 0.5 : 1.1 × 1.1
= 0.25 : 1.21 = 25 :121
4 22 0.49 × 0.49 × 0.49
= × × 5. r = 3.5 cm
3 7 8
Curved surface area of hemisphere = 2πr2
11 × 0.07 × 0.49 × 0.49 22
= = 0.0616 m3
3 =2× × 3.5 × 3.5 = 44 × 0.5 × 3.5 = 77 cm2
7
Answer and Explanations 5

20 × 77 77 22
∴ Cost of painting = ` = = `15.40 = × 9.8 (9.8 + 21)
100 5 7
6. Given, thickness = 1 cm and outer diameter = 28 cm = 22 × 1.4 × 30.8 = 948.64 sq m.
⇒ Inner diameter = (30 – 2) cm. = 28 cm 13. Surface area of sphere = 4πr2
∴ Inner radius = 14 cm (i) 4πr2 = 616 sq cm
∴ Inner curved surface area of the bowl 22
= 2πr2 = 2 π × 14 × 14 4×× r2 = 616
22 7
=2× × 14 × 14 = 1232 sq cm. 616 × 7 154 × 7
7 r2 =
 × = 7 × 7 = 49
4 × 22 22
7. Volume of the book = 1875 cm3 = 1875000 mm3
Let the number of pages in the book be n r2 = 49 ⇒ r = 7 cm
∴ 1875000 = 250 × 150 × 0.1 × n (ii) 4πr2 = 154
1875000 22
⇒ n= 4×× r2 = 154
250 × 150 × 0.1 7
18750000 154 × 7 7×7 49
⇒ n= r2 =
 = =
250 × 150 4 × 22 4 4
⇒ n = 500 49
∴ The number of pages in the book = 500. r2 =
4
8. Volume of the cylinder = π r2 h ⇒ 7
r= = 3.5m
22 2 radius of the cylinder = r
= π × 21 × 21 × 30 cm3 = × 21 × 21 × 30 14. Let the base
7 3
= 22 × 3 × 21 × 30 = 41580 cm3 ∴ The Radius of the sphere = r
2
9. Diameter of the cylindrical bowl = 14 cm 2
3 
⇒ Radius of the cylindrical bowl (r) = 7 cm Surface area of sphere = 4π  r 
2 
Height (h) = 7 cm 9 2
Volume of milk in each bowl = πr2h = 4π r = 9πr 2

22 4
= × 7 × 7 × 7 = 22 × 49 = 1078 cm3. Surface area of cylinder = 2πrh = 2πr2
7
(Given, h = r)
∴ Total volume of milk required = 300 × 1078 cm3 ∴ The required ratio = 9πr2 : 2πr2 = 9 : 2
= 323400 cm3 = 323.4 litres.
15. Volume of cube = (side)3
10. Amount of water displaced = volume of the sphere (ball) Let the side of the cubes be x cm.
4 Given, x3 = 1728 m ℓ
= × π × r3
3 x3 = 1728 cm3 (∵ 1 mℓ = 1 cm3)
(i) r = 21 cm x3 = (12 cm)3
4 22 x = 12 cm ⇒ height = 12 cm.
⇒ Volume = × × 21 × 21 × 21 = 38808 cm3.
3 7 16. Volume of cube with edge 15 m
(ii) r = 0.7 m. = 15 × 15 × 15 m3
4 22 Let the edge of new cube = x m
Volume = × × 0.7 × 0.7 × 0.7 ∴ Volume of new cube = x3
3 7
⇒ 15 × 15 × 15 = 27x3
88 × 0.343
= = 1.437 m3 15 × 15 × 15
21 ⇒ x3 = = 5 × 5 × 5 = 53
27
11. Curved surface area of the roller = 2πrh ⇒ x=5m
22 70 ∴ The edge of the new cube = 5 m 13
=2× × × 140 = 220 × 140 = 30800 sq cm.
7 2 17. Volume of milk = 4.356 ltrs
30800 = 4356 cm3
= sq m = 3.08 sq m.
10000 4356 = πr2h
Area of the play ground 154 sq m. 4356 × 7
⇒ r2 = ⇒ r = 3 cm
∴ Number of revolutions to move 22 × 154
154 1 Let the radius of the milk can be r cm and height = 154 cm
= sq m. = = 50
3.08 0.02 ∴ area of the metal sheet required = 2πr (r + h)
12. ℓ = 21 m. r = 9.8 m. 22
=2× × 3 (3 + 154)
Total surface area of cone = πr + πrℓ
2
7
= πr(r + ℓ) = 2960.57 cm2
6 Fundamentals of Mathematics

1 2 = 108 sq cm
18. Volume of conical vessel = πr h Total area required for the bag of bigger size = 2808 sq cm
3 Total surface area of the smaller bag
and h =  2 − r 2 and r = 2 − h2 2 (ℓb + bh + hℓ) – ℓb where ℓ = 20 cm, b = 5 cm, h = 25 cm
(i) r = 6 cm, ℓ = 10 cm = 2 (20 × 5 + 5 × 25 + 25 × 20) – 20 × 5
= 2 (100 + 125 + 500) – 100
⇒ h = 102 − 62 = 100 − 36 = 64 = 8 = 1450 – 100 = 1350 sq cm.
∴ h = 8 cm Area of the sheet required for the handle
1 2 1 22 = 4% of 1350 sq cm
Volume = πr h = × × 6 × 6 × 8 = 301.71 cm3
3 3 7 = 54 cm sq cm
(ii) h = 4 cm, ℓ = 5 cm ∴ Total area of the sheet required for the smaller sized bag
= 1404 sq cm.
r = 2 − h2 =
 52 − 4 2 ∴ Total area of the plastic sheet required for 200 bags of
each size = 200 × 2808 + 200 × 1404
= 252 − 162 = 9 = 3
= 561600 + 280800 = 842400 sq cm.
1 2 1 22 Cost of 842400 sq cm plastic sheet at the rate of `5 per
 olume =
V πr h = × ×3×3×4
3 3 7 1
10000 sq cm = 842400 × 5 ×
= 37.71 cm3 10000
19. Given, h = 14 cm 84.24 × 5 = ` 421.20
1 2 2. Length of the paper used = (30 + 10) = 40 cm
Volume of cone = πr h = 1100
3 Diameter of cylinder = 7 cm
1 2 Circumference = 2πr
⇒ πr h = 1100 22
3 =2× × 7 = 44 cm
1 22 7
⇒  × × r2 × 14 = 1100 2 m extra is taken length wise.
3 7
∴ circumference = 44 + 2 = 46 cm
1100 × 7 × 3 Total area of the paper = circumference × height
⇒ r2 =
22 × 14 = 46 × 40 = 1840 sq cm.
r2 = 75 3. Let the number of days be n
⇒ r = 5 3 cm 35 × 30 × 10 × (100)3
1 4 3 =
20. Volume of hemisphere = × πr 3000 × 35 × 1000
2 3 35 × 30 × 10 × 100 × 100 × 100
Given, r = 14 cm. =
3000 × 35 × 1000
1 4 22 (∵ 1000 cm3 = 1000 mℓ and 1 ℓt = 1000 mℓ)
∴ Volume = × × × 143
2 3 7 ⇒ n = 100 days
1 4 22 The Number of days = 100.
= × × × 14 × 14 × 14
2 3 7 4. Thickness of the pipe = 1 cm
44 × 2 × 14 × 14 Outer diameter = 14 cm
= ∴ Inner diameter = (14 – 2) cm = 12 cm
3
∴ Outer radius (R) = 7 cm
88 × 14 × 14
= Inner radius (r) = 6 cm
3 Volume of the iron pipe = π (R2 – r2)h
88 × 196 22 22
= = × 2400 (49 – 36) cm3 = × 2400 × 13 cm3
3 7 7
17248 22
= ∴ Weight of the iron pipe = × 2400 × 13 × 7.87 gm
3 7
= 5749.3 cm3 686400 × 7.87
= (∵ 1 cm3 of iron = 7.87 gm)
Exercise 3 7
5401968.0
1. Total surface area of the Bigger bag = = 771709.7
7
2 (ℓb + bh + hℓ) – ℓb where ℓ= 25 cm, b = 10 cm, h = 35 cm
= 2 (25 × 10 + 10 × 35 + 35 × 25) – 25 × 10 ∴ Weight of the pipe = 771709.7 gm = 771.7 kg.
= 2 (250 + 350 + 875) – 250 5. Diameter of the pencil = 8 mm
= 2 (1475) – 250 radius of the pencil (R) = 4 mm
= 2950 – 250 Diameter of the graphite (r) = 2 mm
= 2700 sq cm. radius of the graphite (r) = 1 mm
Area of the sheet required for the handle Height of the pencil = 21 cm
= 4 % of 2700 sq cm Volume of graphite = πr2h
Answer and Explanations 7

22 x + 53.2224 = 8x
= × 1 × 1 × 21 × 10 mm3 7x = 53.2224 sq m
7 x = 7.6032 sq m.
= 22 × 30 = 660 = 660 mm3
volume of wood = πh (R2 – r2) 10. Curved surface area of a cone = πrℓ
22 = 3.14 × 6 × 82 + 62
= × 21 (42 – 12) × 10
7 = 3.14 × 6 × 10
22 = 188.4 sq cm.
= × 21 × 150 = 22 × 3 × 150 = 9900 mm3 Area of the paper = ℓ × b = 188.4 + 11.6
7 ℓ × 4 = 200.0
6. The solid thus formed is a cone of height 7 cm and base radius ℓ = 50 cm.
24 cm. ∴ length of the paper required = 50 cm.

25 cm Volume of the wall


11. Number of bricks =
7 cm Volume of the brick
40 m × 3m × 30 cm
24 cm =
30 cm × 20 cm × 10 cm
1 4000 cm × 300 cm × 30 cm
∴ The volume of the cone = × πr2h = = 6000.
3 30 cm × 20 cm × 10 cm
1 22 ∴ Number of bricks required to construct the wall = 6000.
= × × 7 × 24 × 24 = 4224 cm2
3 7 12. (i) Inner curved surface area of cylinder
7. The radius of the metallic ball = 6.3 cm = 2πrh
4 22
Volume of the metallic ball = × π × r3 =2× × r × 10
3 7
4 22 22 30
= × × 6.3 × 6.3 × 6.3 Given, 264 = 2 × × 100 × r ×
3 7 7 100
88 × 0.9 × 6.3 × 6.3 264 × 7
= = 1047.81 cm3 ⇒ r= = 1.4 cm
3 2 × 22 × 30
Mass = volume × density ∴ Radius of the base = r = 1.4 cm
= 1047.81 × 7.8 = 8172.96 grams. ∴ Inner curved surface area
8. Inner diameter = 2 m 22
=2× × 1.4 × 10
2 cm 7
r=1m = 2 × 22 × 0.2 × 10 = 88 cm2
(ii) Radius of the base = 1.4 cm
(iii) Capacity of the vessel = π × r2 × h
22
= × 1.4 × 1.4 × 10
∴ Inner radius r = 1 m = 100 cm 7
Thinkness of the iron sheet = 2 cm = 22 × 0.2 × 1.4 × 10 = 61.60 cm3
∴ Outer radius = (100 + 2) cm = 102 cm
13. (i) 
Given, volume of the right circular cone = 392 π cm3 and
2 r = 7 cm
Volume of the iron sheet used = π (R3 – r3)
3 1 2
2 22 ⇒ πr h = 392 π
= × (1023 – 1003) 3
3 7 ⇒ r2 h = 392 × 3
44 7 × 7 × h = 392 × 3 13
= × (102 – 100) [(102)2 + (100)2 + (102) (100)]
21 392 × 3
h= = 24 cm.
44 49
= × 2 (30604) = 128245.33 cm3. ∴ The height of the cone = 24 cm
21
(ii) h = 24 cm, r = 7 cm
9. Total surface area of cylinder = 2πr (h + r)
22 ⇒ ℓ= h2 + r 2 = 242 + 7 2
=2× × 0.28 (3.5 + 0.28)
7 625 = 25.
22 ∴ The slant height of the cone = 25 cm.
=2× × 0.28 × 3.78 = 6.6528 sq m. (iii) The curved surface area of the cone = πrℓ
7
22
Let x be the actual area of the sheet used = × 7 × 25 = 22 × 25
x 7
+ 6.6528 = x = 550 cm2.
8
8 Fundamentals of Mathematics

14. Given, h = 7 cm and r = 24 cm = Curved surface area of the cylinder + Base area of the cylinder
2 2 = 2πrh + πr2
ℓ= h +r
= 7 2 + 242 = 49 + 576 = 625
∴ ℓ = 25 cm 14 cm
1
Volume of the heap = πr2h
3 21 cm
1 22
= × × 24 × 24 × 7 = 22 × 8 × 24 = 4224 cm3
3 7
Canvas required = πrℓ
22 13200
= × 24 × 25 = = 1885.71 cm2.
7 7
15. Given, surface area of sphere = 616 cm2
⇒ 4πr2 = 616 22
   22 
616 × 7 = 2× × 14 × 21 +  × 14 × 14
⇒ r2 =  7   7 
22 × 4
22
616 × 7 = × 14 × ( 42 + 14)
= =7×7 7
88 = 44 ×56
∴ r2 = 7 × 7 = 2464 cm2.
⇒ r = 7 cm. ∴ The total surface area of the cylindrical container = 2464
4 3 cm2.
Volume of the sphere = πr
3 2. Let r1 be the radius of the base of the cylinder, r2 be the radius
4 22 of the base of the cone and
= × ×7×7×7
3 7 ‘h’ be the height of the cone and that of the cylinder.
88 × 49 Volume of the cylinder 3
= Given, =
3 Volume of the cone 4
4312 2
πr1 h 3
= = 1437.33 cm3 =
3 1 2 4
πr2 h
4
3
 7 4 3
Volume of the smaller sphere = π×   = π
3 7 3 3r 2 3
⇒ 12 =
∴ Number of spheres can be made r2 4
4 r12 1
π × 73 ⇒ =
= 3 = 343 r2 2 4
4
π r1 1 1
3 ⇒ = = .
r2 4 2
4 3
16. Volume of each iron sphere = = πr ∴  he ratio of the radii of the cylinder and the cone = r1 : r2
T
3
= 1 : 2.
4
Volume of new sphere = 8 × × πr3 3. Given, a cone of maximum volume is carved out from a
3 wooden cylinder.
4
= × π × (2r)3
3
⇒ Radius of the new sphere = 2r
(i) Diameter of the new sphere = 2 × 2r = 4r
(ii) 
Surface area of each iron sphere is s = 4πr2 and surface
area of the new sphere with radius 2r is
S = 4π (2r)2 = 16 πr2 14 cm
∴ Ratio of s and S
= 4πr2 : 16πr2 7 cm
= 4 : 16 = 1 : 4.

For the cone to be of maximum volume, the base radius of the


HOTS cone should be equal to the base radius of the cylinder and the
1. Given, for a cylindrical container height of the cone should be equal to the height of the cylinder.
h = 21 cm, r = 14 cm. Height of the cylinder = 14 cm, radius of the cylinder = 7 cm.
Total surface area of the cylindrical container ⇒ Height of the cone = 14 cm, radius of the cone = 7 cm.
Answer and Explanations 9

Volume of the cylinder = πr2h. Slant height (l)


1 2 = r 2 + h2 = 52 + 122 = 25 + 144 = 169 = 13 m .
Volume of the cone = πr h .
3 The curved surface area of a cone = πrl sq. units.
1 2 22 1430 2
Volume of the remaining wood = πr2h – πr h ⇒ C.S.A = πrl = × 5 × 13 = m
3 7 7
2 2 The cost of canvas for 1 m2 is ` 42
= πr h
3 1430 2 1430
⇒ Cost of canvas for m = × 42 = ` 8580
2 22 2 7 7
= × × 7 × 7 × 14 = × 2156 = 1437. 33 cm3
3 7 3 ∴ Cost of the canvas at ` 42 per m2 = ` 8580.
∴ Volume of the wood remaining = 1437.33 cm3. 6. Let the lengths of the two cuboids be 2p, 3p.
4. Total surface area of the hollow metallic pipe Let the breadths of the two cuboids be 4q, 5q.
Let the heights of the two cuboids be r, 2r.
Volume of a cuboid = l × b × h.
r
R l1b1h1
⇒ Ratio of the volumes of the two cuboids =
l2 b2 h2
=l1 2=
p, b1 4=
q, h1 r
98 cm
=l2 3=
p, b2 5=
q, h2 2r
2 p × 4q × r
=
3 p × 5q × 2r
8 pqr 8 4
= = =
30 pqr 30 15
= Inner curved surface area + outer curved surface area + 2 ×
Area of the ring. ∴ The ratio of the volumes of the two cuboids = 4: 15.
= 2πrh + 2πRh + 2π R − r ( 2 2
) a 3.
7. The longest diagonal of a cube of side ‘a’ is
Let the sides of the two cubes be a1 and a2 respectively.
= 2πh ( R + r ) + 2π R − r( 2 2
) Given, ratio of the longest diagonals =
= 2π ( R + r )  h + ( R − r )  a1 3 4
⇒ =
Substituting the values, a2 3 5
2.8
r = 1.4 cm
= a1 4
2 ⇒ =
a2 5
4.2
R = 2.1cm
= Ratio of L.S.A of the two cubes
2 2 2
h = 98 cm 4a12 a12  a1   4
= = 2
= 2  =  
T.S.A of the hollow metallic pipe 4 a2 a2  a2  5
2 × 22
= × ( 2.1 + 1.4) × ( 98) + ( 2.1 − 1.4)  ∴ Ratio of the L.S.A of the two cubes = 16 : 25 .
7 3
a13  a1 
2 × 22 = =
Ratio of volume of the two cubes
= × 3.5 ×98.7 a23  a2 
7
3
= 2171.4 cm2  4 64
= = 
∴ T.S.A. of the metal pipe = 2171.4 cm2.  5 125
5. The canvas covers the curved surface area of the conical tent. ∴ Ratio of the volumes of the two cubes = 64 :125 .
The radius of the tent (r) = 5 m, height of the tent (h) = 12 m.
8. Three cubes of side 4 cm are joined end to end to form a cuboid.
13

12 m
3a

The dimensions of the cuboid, l = 4 + 4 + 4 = 12 cm,


5m b = 4 cm
c = 4 cm
10 Fundamentals of Mathematics

On considering the algebraic identity.


T.S.A of a cuboid = 2 ( lb + bh + lh )
( l + b + h)2 = l 2 + b 2 + h 2 + 2 ( lb + bh + lh )
= 2 (12 × 4 + 4 × 4 + 12 × 4)
T.S.A of the resulting cuboid
⇒ ( 36) = 450 + 2 ( lb + bh + lh )
2

= 2 ( 48 + 16 + 48)
⇒ 1296 = 450 + 2 ( lb + bh + lh )
(112) 224 cm2.
= 2=
⇒ 1296 – 450 = 2 ( lb + bh + lh ) =
T.S.A of the cuboid
Volume of a cuboid = l × b × h
Volume of the resulting cuboid = 12 × 4 × 4 =192 cm3. ∴ T.S.A of cuboid = 846 cm2.
∴ T.S.A of the cuboid and volume of the cuboid are 224 cm2 12. Ratio of the height and the radius of a cone = 2: 1.
and 192 cm3, respectively. Let the height be ‘h’ and the radius be ‘r’,
9. A coin can be considered as a cylinder where the thickness of h 2
⇒ =
the coin forms the height of the cylinder. r 1
Volume occupied by the stack of 35 coins= 35 × volume ⇒ h = 2r .
occupied by a coin.
= 16 5π sq. cm.
C.S.A of the cone
Volume of a cylinder = πr 2 h
C.S.A = πrl
0.8
Here, r = = 0.4 cm, h = 0.2 cm. l= r 2 + h2 = r 2 + ( 2r ) =
2
5r 2 = r 5
2
22 ∴ πrl = πr × r=5 16 5π
Volume occupied by a coin = × ( 0.4) × 0.2
2

7 ⇒ r 2 = 16 ⇒ r = 4 cm.
⇒ Volume occupied by the stack of 35 coins Hence, the required base radius of the cone is 4 cm.
2
22  4  2
= ×   × × 35 13. Let the radii of the two spheres be r1 and r2 .
7  10  10
4 3
22 16 2 πr1
r13 8
= × × × 35
7 100 10 = 3 = =
Ratio of volumes
4 3 r23 27
3520 πr
= = 3.52 cm3 3 2
1000 3
 r1  8
∴ Volume occupied by the stack = 3.52 cm3. ⇒  r  = 27
2
10. Given, for a cuboid,
T.S.A = 108 cm2, L.S.A = 60 cm2 and l = 6 cm. 1 r 8 2
⇒ = 3 =
T.S.A = L.S.A + 2(base area) r2 27 3
T.S.A = L.S.A + 2lb 2
108
= 60 + 2lb 4πr12 r12  r1 
⇒ =
Ratio of surface areas = =
108 = 60 + 2 × 6 × b 4πr2 2 r2 2  r2 
12
= b 108 − 60 2
 2 4
48 = =
b = 4 cm.
=  3  9
12
∴ The ratio of the surface areas of the spheres = 4: 9.
L.S.A= 2h ( l + b )= 60
1 th
⇒ 2h ( 6 + 4 ) =
60 14. Hemispherical bowl ‘A’ can hold
4
of the liquid more than

60 that of B.
⇒ 2=
h = 6.
10 Let ‘B’ hold ‘x’ litres of the liquid
6 x  5x
⇒ h= = 3cm.  litres i.e,
Then, ‘A’ can hold  x + litres of the liquid.
2 4  4
∴ The breadth and the height of the cuboid is 4 cm and 3 cm 5x
respectively. 4 5
⇒ The ratio of volumes of A and =B = .
11. Given, x 4
l+b+h =36 cm Let r1 and r2 be the radii of the hemispherical bowls.
The longest diagonal of a cuboid of dimensions 2 3
πr
l × b × h= 2
l +b +h 2 2
⇒ 3 1 =5
2 3 4
2 2
15 2
l +b +h = 2 πr
3 2
Squaring on both the sides, 3
r13 5 r 5
(15 2 ) =
2
2
l +b +h = 2 2
225 × 2= 450 . ⇒ =⇒  1  =
r23 4 r
 2 4
Answer and Explanations 11

r1 5 Height of the cylinder ( h1 ) = 21 cm.


∴ = 3
r2 4 1 2
Volume of a cone = πr2 h2 .
Hence, the required ratio is 3
5 :34. 3
6
= 14 cm.
15. Diameter of the hemispherical bowl Radius of the identical cones ( r2 ) = = 3 cm.
2
14
= = 7 cm.
⇒ Radius of the hemispherical bowl Height of the identical cones ( h2 ) = 7 cm.
2
1
 he copper plating is done on the curved surface area of
T ⇒ πr12 h1 = n × πr2 2 h2
the hemispherical bowl. 3
C.S.A of a hemispherical bowl = 2πr 2 1
⇒ r12 h1 = n × r2 2 h2
22 3
= 2× ×7×7 =308 cm2
7 1
⇒ ( 7 ) × 21 = n × × ( 3) × 7
2 2

308 3
= m2
10000 7 2 × 21 × 3
⇒n= 2
= 0.0308 m2. 3 ×7
The cost of copper plating the outer surface is ` 500 per ⇒n= 49.
m2.
∴ The number of cones that can be filled with ice cream = 49.
∴ Cost of copper plating the hemispherical bowl of area
0.0308 m2 = 0.0308 × 500 = ` 15.4
18. Volume of a cube of edge ‘a’ units = a3 .
16. When a right-angled triangle is revolved about one of its
perpendicular sides the solid shape so obtained is a cone.
=
Volume of each cubical box ( 4=
) 64 cm
3
3

Volume of the cubical container = ( 36) .


3

A
∴ Maximum number of cubical boxes that can be placed in
Volume of cubical container
the cubical container =
Volume of cubical box
10 cm 36 × 36 × 36
8 cm
= = 9 × 9 × 9 = 729 .
4×4×4
∴  he number of cubical boxes that can be placed in the
T
cubical container = 729.
B 19. Let the radius and the height of the identical cylinders be ‘r’
O 6 cm and ‘h’ respectively.
So, radius of the identical cones is ‘r’.
Volume of 6 identical cylinders = Volume of 18 identical cones
Here, AOB is the right-angled triangle which is revolved about Let the height of the identical cones be ‘x’ units.
the side 8 cm. So, the side 8 cm acts as the height of the cone. 1
Height of cone (h) = 8 cm. ⇒ 6 × πr 2 h= 18 × × πr 2 x
3
Radius (r) = 6 cm. 2 2
Slant height (l) = 10 cm
⇒ 6πr h =π 6 r x
The C.S.A of a cone = πrl ⇒ h= x.
22 ⇒ Height of the identical cones = h.
⇒ C.S.A =πrl = × 6 × 10 =188.57 cm2 ∴ Height of the identical cones = Height of the cylinder.
7
1 20. From the question,
The volume of a cone = πr 2 h volume of the cubical container = volume of the cuboidal
3
1 22
container. 13
= × × 6 × 6 × 10 = 377.14 cm3 cm3. Let the side of the cubical container be ‘x’
3 7 ∴ x3 = 9 × 8 × 3
 hus, C.S.A of the solid shape obtained = 188.57 cm2
T x3 = 216
and the volume = 377.14 cm3. x = 3 216 = 6
17. The volume of the cylindrical container is equal to the volumes ∴ side of the cubical container = 6 cm.
of all cones in which ice cream is filled. 21. Given, side of a cube = 4 cm
Let ‘n’ be the number of cones in which ice cream is filled. Side of cube = Diameter of sphere
So, volume of cylinder = n × Volume of an ice cream cone. Diameter of sphere = 4cm
Volume of a cylindrical container = πr12 h1 4
∴ Radius of sphere = = 2 cm
Given radius of the cylinder ( r1 ) = 7 cm. 2
12 Fundamentals of Mathematics

Volume of the gap = volume of cube – volume of sphere = 2 3


4 Volume of a hemispherical tank = πr
(4) – π ( 2)
3
3
3
3
2 22
× ( 7)
3
= 64 – 28.07 = 35.93 cm3 Volume of a hemispherical tan k =
 ×
3 7
22. Internal diameter of hemispherical tank = 14 m 2156
14 = = 718.66 m3
Internal radius of hemispherical tank = m = 7m 3
2 Volume of water pumped into the tank = Volume of
1kilo liter = 1 m3
Tank contains 50kl = 50 m3 of water hemispherical tank – 50 m3
= 718.66 – 50 = 668.66 m3
Statistics
Answer and Explanations 14
Ascending order:
STEPWISE MARKING SOLUTIONS
1 2 3 1 1 7 7 8
3 ,3 ,3 ,3 ,3 ,3 ,3 ,3
Objective Type Questions 4 5 7 2 2 11 8 9
Since the number of observations (n) is even, the median is the
1. (d) Range = Highest value – Least value th
⇒ 37 = Highest value – 64  n th  n  
average of   and   + 1 observations.
⇒ Highest value = 101 2  2  
2. (b) The required upper limit is 75. th th
4 observation + 5 observation
Median = 
24 + 36 2
3. (c) The required class mark = = 30
2 1 1
3 +3
2 + 4 + 6 + ........ + 20 10 (10 + 1) 2 2 = 31
4. The required mean = = = 11
10 10 2 2
 (1 Mark)
5. False
5. Given scores are 3, 4, 3, 5, 4, 6, 6 and x.
6. True
Here, frequency of 3 is 2; frequency of 4 is 2; frequency of 6 is
7. Median 2; frequency of 5 is 1. (1 Mark)
But mode is 6.
 n + 1 th
8.   ∴ x must be 6. (1 Mark)
 2 
x + 2 + 2 x + 3 + 3x + 4 + 4 x + 5
6. Given, = x + 2 (1 Mark)
4
Subjective Type Question
⇒ 10x + 14 = 4x + 8 (1 Mark)
Exercise 1 ⇒ 6x = –6
⇒ x = –1. (1 Mark)
1. The marks scored by Pavan are: 98, 97, 99, 100 and 98
The sum of the observations 7. 6
Mean =  (1 Mark)
The number of observations Exercise 2

=
(98 + 97 + 99 + 100 + 98) = 492 = 98.4  (1 Mark) 1. The least score = 2
5 5 The highest score = 100
2. First 30 natural numbers are
1, 2, 3, 4, 5, …..30. C.I Tally Marks Frequency
The sum of the observations 1 – 10 ||| 3
A.M. =  (1 Mark)
The number of observations
11 – 20 || 2
1 + 2 + 3 + 4 + 5 + ..... + 30
A.M. = 21 – 30 ||| 3
30
465 31 31 – 40 || 2
= = = 15.5  (1 Mark)
30 2 41 – 50 ||| 3
3. Given observations are 51 – 60 ||| 3
12, 24, 8, 15, 19, 10, 10, 7, 11
Ascending order : 7, 8, 10, 10, 11, 12, 15, 19, 24 61 – 70 |||| 5
Since the number of observations (n) is odd,
 n + 1 th
The median is 
 2 
 observation (1 Mark) 71 – 80 |||| ||| 8

 9 + 1 th 81 – 90 | 1
= 
 2 
 observation = 5th observation = 11. (1 Mark) 14
91 – 100 |||| | 6
4. Given observation are
3 1 1 1 7 2 7 8 Total 36
3 ,3 ,3 ,3 ,3 ,3 ,3 ,3
7 2 2 4 9 5 11 9  (3 Marks)
2 Fundamentals of Mathematics

2. Range = 80 – 55 = 25. Class Average height Length of the bar


Size of the class interval = 4 (in cm) (in cm)
Range
Number of classes = VIII 120 12
length of the class interval
IX 134 13.4
25 1
= = 6 ≈7 X 156 15.6
4 4
 (2 Marks)
Class interval Tally marks Frequency
Weight (in kg) Scale: on Y-axis 1cm = 10cm
Y
55 – 58 5 20 15.6
||||
13.4
59 – 62 |||| | 6 12
11
10.5
63 – 66 7 10 10
|||| ||

67 – 70 |||| | 6

71 – 74 ||| 3
0
75 – 78 || 2 V VI VII VIII IX X

79 – 82 | 1 X
 (2 Marks)
Total 30
2. Here, the class intervals are continuous.
3. The following histogram is drawn according to the method
described above. (1 Mark)
Height (in cm) x Number of children (f) fx
10
150 7 1050 Y
9
Scale: on X-axis 1 cm = 10 units
151 5 755 8 on Y-axis 1 cm = 1 unit
7
152 4 608
6
Frequency

153 4 612 5
154 3 462 4
155 2 310 3
2
Total ∑f = 25 ∑fx = 3797
1
(21/2 Marks) 0
10 20 30 40 50 60 70 80 90 X
⇒ N =∑f = 25
∑fx = 3797 Classes
Histogram
Σfx 3797  (3 Marks)
Mean = = = 151.88cm .(11/2 Marks)
N 25 3.

Exercise 3 Class Interval Mid value Frequency (f)


0 – 10 5 12
1. Scale : on Y – axis 1 cm = 10 cm
10 – 20 15 13
Class Average height Length of the bar
20 – 30 25 25
(in cm) (in cm)
30 – 40 35 20
V 100 10
40 – 50 45 10
VI 105 10.5
 (2 Marks)
VII 110 11
Answer and Explanations 3

30

25

Frequencies 20

15

10

X
–5 0 5 10 15 20 25 30 35 40 44 50 55
Midvalues
 (2 Marks)

2. Given scores are 23, 24, 27 31, 37, 46 and 78


PROBLEMS FOR ADDITIONAL PRACTICE
23 + 24 + 27 + 31 + 37 + 46 + 78 266
= = 38 .
7 7
Objective Type Question
3. Given scores in ascending order are 9, 12, 13, 14,
1. (b) The ascending order is 8, 12, 17, 17, 19, 25, 38, 51
18, 24, 26
17 + 19
Median = = 18 Number of scores = 7
2 (7 + 1) th
2. (c) The mode is 13, if x = 13 for all real values of x. Median = observation = 4th observation = 14
2
3. (b) Median. 4. Given scores in ascending order are 23, 36, 41, 48, 72, 79, 81,
4. (a) 30. 93
Number of scores = 8
5. (d) Frequency polygon. ∴ Median = Mean of 4th score and 5th score
6. False 48 + 72
= = 60
7. True
2
5. Given scores are 2, 4, 2, 3, 3, 4, 6, 8, 6, 2, 2 and 3
8. True
Frequency of 2 is 4
9. True Frequency of 3 is 3
10. Mode
Frequency of 4 is 2
Frequency of 6 is 2
11. 7 Frequency of 8 is 1
12. Middle ∴ Mode = 2

n ( n + 1) 6. Given scores are 11, 12, 11, 12, 13, 11, 12, 13, and 14
13. Frequency of 11 is 3
2 Frequency of 12 is 3
Frequency of 13 is 2
Frequency of 14 is 1
Subjective Type Question
∴ Mode is 11 and 12
14
Exercise 1 7. Given that the ages (in years) of ten students are 12, 13, 14, 15,
16, 17, 18, 19, 20, and 21.
1. Given scores are 4, 6, 7, 13, 25 and 35
Sum of the scores Mean age =
(12 + 13 + 14 + 15 + 16 + 17 + 18 + 19 + 20 + 21)
Mean =
Number of scores 10
4 + 6 + 7 + 13 + 25 + 35 90 165
= = = 15 . = = 16.5 years.
6 6 10
4 Fundamentals of Mathematics

8. Given observation are


Class interval Tally marks Frequency
12, 27, 13, 15, 21, 32, 10, 14
12 + 27 + 13 + 15 + 21 + 32 + 10 + 14 75 – 79
|||| |||| |
11
A.M. =
8 80 – 84 ||| 3
144
= = 18 85 – 89 |||| 4
8
90 – 94 ||| 3
9. First six multiples of 6 are
95 – 99 || 2
6, 12, 18, 24, 30, 36
6 + 12 + 18 + 24 + 30 + 36 126 Total 40
A.M. = = = 21 .
6 6
2. Range = Highest score – Least score = 77 – 20 = 57
10. All the integers from – 8 to 2 are: – 8, – 7, – 6, – 5, – 4, – 3, – Size of the class interval = 10
2, – 1, 0, 1, 2. 57
There are 11 (= n) observations. Number of classes = = 5.7 ≈ 6
10
 n + 1 th 11 + 1 th
Median is   observation =   observation = 6th Class interval Tally marks Frequency
 2   2 
observation= – 3 20 – 29 8
|||| |||
11. The number of Audi Cars (in thousands) in ten cities are 7, 9, 30 – 39 |||| 4
13, 14, 15, 18, 8, 4, 6 and 16
Ascending order of the number of cars (in thousands) 40 – 49 12
|||| |||| ||
4, 6, 7, 8, 9, 13, 14, 15, 16, 18
50 – 59 12
Median =
( 4 + 6 + 7 + 8 + 9 + 13 + 14 + 15 + 16 + 18)1000 =
|||| |||| ||
10 60 – 69 9
|||| ||||
110000 70 – 79 ||| 3
= 11000
10 Total 48
12. Given scores are 130, 140, 130, 150, 140, 160, 160 and a.
3.
Here, frequency of 130 is 2; the frequency of 140 is 2; the
frequency of 160 is 2; the frequency of 150 is 1. x f fx
Given that the mode is 140.
∴ a must be 140. 1 2 2
2 2 4
13. The number of Mobile Phones (in thousands) manufactured by
a company in the last eight years are 8, 9, 10, 12, 15, 16, 18 and 3 4 12
22. 4 3 12
The number of years is 8, which is even.
5 4 20
 n th  n th
The median is the mean of   observation and  + 1 6 5 30
2 2 
Total 20 80
observation, that is the mean of 4th and 5th observations.

Median =
(12 + 15)1000 =
27000
= 13500 .
∑f = 20, ∑fx = 80
2 2
∴ Mean x =
∑ fx = 80
=4
14. The total weight of 25 students = 625 kg

The mean weight =


625
= 25 kg
∑f 20
25 4. To count, we can use tally marks. We record tally marks
Exercise 2 in bunches of five, the fifth one crossing the other four
diagonally i.e. |||| .
1. Range = Highest score – Least score = 96 – 60 = 36
Size of the class interval = 5 Thus, we may prepare a frequency table as below.
36 1 Number of Tally marks Number of families
Number of classes = = 7 ≈ 8
5 5 computers (frequency)
Class interval Tally marks Frequency 1 5
||||
60 – 64 7
|||| || 2 |||| 4
65 – 69 5
|||| 3 || 2
70 – 74 5
|||| 4 | 1
Answer and Explanations 5

5. Taking class intervals 1–10, 11–20, 21–30, 31–40 and 2.

Number of employees
41–50, we construct a frequency distribution table for
the above data.
30
First, we write the marks in the ascending order as:
25 D(17.5)
1 2 7 9 15 16 17 18 19 20 E(22.5)
20
22 24 25 25 25 27 28 29 32 33 C(12.5)
15
35 35 37 42 43 44 45 48 50 50
10 F(27.5)

Now, we can prepare the frequency distribution table as 5 B(7.5)


below. G(32.5)
0 A(2.5) 5 10 15 20 25 30 35
Class interval Tally marks Frequency Annual Salary (₹ in lakhs)

1 – 10 |||| 4
3.
11 – 20 6
|||| | Class Interval Mid Values Frequency
21 – 30 8
|||| ||| 0 – 10 5 2
31 – 40 5
|||| 10 – 20 15 3
41 – 50 |||| || 7
20 – 30 25 4

30 – 40 35 5
6.

x f fx 40 – 50 45 6

1 3 3 50 – 60 55 5
2 5 10 60 – 70 65 4
3 4 12
70 – 80 75 3
4 8 32
Total ∑f = 20 ∑fx = 57 80 – 90 85 3

90 – 100 95 1
∑ fx 57
Mean x = ( ) ∑f =
20
= 2.85 4. Scale: on X-axis 10% = 1 cm

Year Pass percentage Length of the bar

Exercise 3 2000 50 5 cm

1. 2001 45 4.5 cm
Number of students

8
2002 70 7 cm
7
2003 65 6.5 cm
6
2004 76 7.6 cm
5
Scale : on X-axis 10% = 1 cm
4 8 7.6
7
7 6.5
3
6
Pass percentage

5
5
4.5 14
2
4
1 3
2
0 40 50 60 70 80 90 100 1
Marks 0
2000 2001 2002 2003 2004
Year
6 Fundamentals of Mathematics

5. Number of students in the class = 10 – 2 + 3


= 13 – 2 = 11.
C.I. 0–5 5 – 10 10 – 15 15 – 20 20 – 25
485
F 1 3 5 3 4 ∴ New mean weight = = 44.09 kgs.
11
Σf i xi
y 2. Given, Mean of the given frequency distribution = =7
Σf i
6
Σf i xi 1 × 3 + 2 × 2 + 5 × x + 7 × 5 + 8 × 4 +11 × 9
=
5 Σf i 3+ 2 + x +5+ 4 +9
4 3 + 4 + 5 x + 35 + 32 + 99
⇒ =7
Frequencies

3 3+ 2 + x +5+ 4 +9
173 + 5 x
2 ⇒ =7
23 + x
1 ⇒ 173 + 5x = 161 + 7x
0 x ⇒ 173 – 161 = 7x – 5x
5 5 10 15 20 25 ⇒ 2x = 12
12
6. x
= = 6.
2
C.I. Mid Values Frequencies ∴ The value of ‘x’ is 6.
20 – 50 35 4 3.

50 – 80 65 6 x 5 10 20 25 30 35
80 – 110 95 8 f 6 7 x y 4 5
110 – 140 125 5 Given, Mean = 20.125 Σfi = 40
140 – 170 155 3 Σf i xi
⇒ = 20.125
170 – 200 185 1 Σf i
FREQUENCY POLYGON
30 + 70 + 20 x +25 y +120 +175
⇒ = 20.125
40
10 395 + 20 x + 25 y
⇒ = 20.125
40
9
⇒ 395 + 20x + 25y = 805
8 D(95) ⇒ 20x + 25y = 805–395 = 410 ... (1)
Σfi = 22 + x + y = 40
7 ⇒ x + y = 40 – 22 = 18 ... (2)
The equations obtained are,
6 C(65) 20x + 25y = 410 ... (1)
x + y = 18 ... (2)
5 E(125)
From (2), y = 18 – x
4 Substitute y = 18 – x in (1).
B(35)
20x + 25y = 410
3 F(155) 20x + 25(18 – x) = 410
20x + 450 – 25x = 410 – 5x = –40
2 x = 8.
y = 18 – x
1 G(185) y = 18 – 10
y = 8.
A(5) H(215)
∴ The values of ‘x’ and ‘y’ are 10 and 8 respectively.
0 20 50 80 110 140 170 200 230
4. It can be observed from the data that, the 1st 12 observations is
20. The 13th observation to the 22nd observation is 25. The 23rd
observation to the 29th observation is 6.
HOTS To find the median, the total number of observations (n)= 46.
If the number of observations (n) is even, then the median is the
1. Given, mean weight of 10 students = 45 kgs.
 n th n th
The sum of the weights of 10 students = 10 × 45 = 450 kgs. average of   observation and  + 1 observation.
Two students weighing 40 kg and 38 kg are shifted to another 2 2 
class and 3 students of weight 35 kg, 42 kg and 36 kg are joined th th
 46   46 
in the class. ⇒ Average of   and  + 1 observation is the
Sum of weights of the students 2 2 
= 450 – (40 + 38) + (35 + 42+ 36) = 450 – 78 + 113 = 485 kgs. median.
Answer and Explanations 7

⇒ Average of the 23rd and 24th observation is the median. 25x – 22.6x = 813.6 – 780
Here, both the 23rd and 24th observations are same and 2.4x = 33.6
equal to 28. 33.6
∴ Median of the given data is 28. =x = 14 .
2.4
5. The frequency distribution table for the given data is as follows: The highest frequency is 14 which is for the observation
25.
Salary (x) ` 20, 000 `50, 000 `80,000 ∴ Mode of the frequency distribution = 25.
Number of 6 3 1 x1 + x2 + x3 +…+ xn
employees (f) 8. =x  ... (1)
n
Σf i xi 20, 000 ×6 + 50, 000 × 3 +80, 000 ×1 Given, sum of the deviations about the mean = 0.
Mean = =
Σf i 10 ( x1 – x ) + ( x2 – x ) + ( x3 – x ) + ...... ( xn – x )
1, 20, 000 +1,50, 000 + 80, 000 3,50, 000 = ( ( x1 + x2 + x3 ..... + xn ) − ( x + x + x .....n times)
= = = 35000.
10 10 From (1) x1 + x2 + x3 ... + xn =
nx
∴ Mean salary = ` 35,000.
= nx − ( x + x + x …..n times)
Median:
The number of observations in the given data is 10. = nx − nx
If the number of observations (n) is even, then the median is the = 0.
 n th n th
average of   observation and  + 1 observation. 9. 7, 9, 11, x, 13, 16, 18
2 2  n = 7 (odd)
10 th 10 th  n + 1 th  7 + 1 th
 verage of   and  + 1 observation is the
⇒ A The median of the data is   observation =  
2 2   2   2 
median. observation= 4th observation.
⇒ Average of the 5th and 6th observation is the median. Since the data is in the increasing order of magnitude, x can
The 5th and 6th observation are the same that is ` 20, 000. take integer values 11, 12 or 13.The value of ‘x’ can only be an
∴ The median salary = ` 20, 000 integer value as, it denotes the number of persons.
Mode: ∴ The possible values of x are 11, 12 and 13.
Mode salary is ` 20, 000 since its frequency is the highest. 10. Given, in a bar graph, length of longest bar – length of shortest
bar = 5cm
6. Given, Mean of the frequency distribution = 12
Shortest barrepresents 50 units and longest bar represents 300
Σf i xi units.
⇒ = 12
Σf i The difference between longest bar and shortest bar represents
4 +18 + 50 +13 x +150 300 – 50 = 250 units.
⇒ = 12 The 250 units are represented by bar of length 5 cm.
19 + x 250
222 + 13x = 228 + 12x So, a bar of length 1cm represents = 50 units.
5
13x – 12x = 228 – 222 = 6
x = 6. ⇒  he number of units represented by a bar of length 2.5 cm
T
= 50 × 2.5 = 125 units.
Σfi = n = 1 + 3 + 5 + x + 10
= 1 + 3 + 5 + 6 + 10 x+ y
11. = 30, ⇒ x + y = 60. ... (1)
= 25. 2
n = 25(odd) y+z
 n + 1 th = 40, ⇒ y + z = 80. ... (2)
∴   observation is the median. 2
 2  x+ z
= 50, ⇒ x + z = 100. ... (3)
 25 + 1 th 2
  observation i.e 13th observation Adding (1), (2) and (3)
 2 
x + y + y + z + x + z = 60 + 80 + 100 = 240
The 13 observation is 13.
th
2(x + y + z) = 240
∴ median of the frequency distribution = 13.
⇒ x + y + z = 120.
7. Given, Mean of the frequency distribution = 22.6. x+ y+ z 14
Σf i xi ∴ average of x, y and z =
= 22.6 3
Σf i 120
= = 40 .
50 + 120 + 200 + 25 x + 270 + 140 3
⇒ = 22.6
36 + x 12. Obtained sum of weights of 30 students = 30 × Obtained mean
780 + 25 x = 30 × 40
= 22.6
36 + x = 1200kgs.
Two weights 30kgs, 35kgs, were misread as 40kgs and 45kgs.
⇒ 780 + 25x = 813.6 + 22.6x
8 Fundamentals of Mathematics

∴ Correct mean weight of the class ⇒ Mean of the data obtained by subtracting 3 from each
Obtained sum of weight of 30 students observation = 5 = 8 – 3.
– (sum of misread observations) It can be observed that, the mean obtained after subtracting
3 from each observation is equal to the difference of the
+ (sum of correct observations) mean of the given data and 3.
=
30 14. When three coins are tossed simultaneously, the sum of the
1200 − ( 40 + 45) + (30 + 35) number of heads that turned up and the number of tails that
= turned up is equal to three.
30
1180 Number of heads Frequency (fi) Number of tails (xi)
= = 39.33 .
30 0 3 3
13. Mean of 4,6, 8, 10 and 12 1 4 2
4 + 6 + 8 +10 +12 40 2 5 1
= = = 8.
5 5 3 3 0
∴ Mean of 4, 6, 8, 10 and 12 = 8. Σf i xi
Mean of the number of times tail turned up =
Mean of the data obtained by subtracting 3 from each Σf i
observation = Mean of (4 – 3), (6 – 3), (8 – 3), (10 – 3),
and (12 – 3) 3 × 3 + 4 × 2 + 5 ×1+ 3 × 0 9 + 8 + 5 + 0 22
= = = = 1.46.
1+ 3 + 5 + 7 + 9 25 15 15 15
= = =5.
5 5
Probability
Answer and Explanations 15
STEPWISE MARKING SOLUTIONS 25 38 63
= + = (1 ½ Marks)
100 100 100
Objective Type Questions 6. The total number of bags = 8
The number of bags which contain more than 15 kg is 5.
1. (b) The probability of an event lies between 0 and 1.
 (1 Mark)
2. (a) {(H,T), (H,H), (T,H), (T,T)} are the four possible 5
outcomes. Hence, required probability =  (1 Mark)
8
Favourable outcomes are {H,T, T,H}
2 1 7. There are 365 days in a non-leap year. It contains 52 complete
∴ Required probability = = weeks and one day.
4 2
That one day can be Mon, Tue, Wed, Thu, Fri, Sat or Sun.
3. (c) 
The probability of getting 4 in a single through of a die Therefore, there are 7 possibilities. For the day left must be
1 Monday. (1 ½ Marks)
is . 1
6 ∴ Hence, the required probability =  (½ MARK)
7
4. (d) The probability of a sure event is 1.
8. The number of classes which have the percentage of attendance
5. (d) 
The probability of non-occurrence of the same event (E) is more than 75, is 3.
6 3
is 1 – 0.4 = 0.6 = = The total number of classes is 6. (1 Mark)
10 5 ∴ Hence the required probability
6. False Number of favourable events 3 1
= = =  (1 Mark)
7. True Total number of events 6 2
8. False 9. The prime numbers among 1, 2, 3, 4, 5 and 6 are 2, 3 and 5.
 (½ Mark)
1
9. The required probability= P(getting 2) + P(getting 3) +
2 P(getting 5)
10. 8 30 120 50 200 1
= + + = =  (1 ½ Marks)
11. 0 600 600 600 600 3
10. The total number of subjects = 6.
Subjective Type Questions In two subjects he scored more than 80% marks. (1 Mark)
1. We know that, in a year there are 2 1
∴ The required probability = =  (1 Mark)
12 months. 6 3
∴ The required probability
Number of favourable cases PROBLEMS FOR ADDITIONAL PRACTICE
=  (½ Mark)
Total number of cases
Objective Type Questions
2 1
= =  (½ Mark) 1. (d) The probability of an impossible event is 0.
12 6
2. If a coin is tossed, then the possible outcomes are H and T. 2. (c) The probability of any event ≤ 1
The favourable outcome is H. (½ Mark) 31
∴  cannot be the probability of any event
The required probability 28
Number of favourable cases 1 3. (a) Favourable outcomes are 2, 4, 6
= .=  (½ Mark)
Total number of cases 2 3 1
∴ Required probability = =
3. The total number of items in the bag 6 2
= 12 + 3 + 7 = 22 4. (b) The sum of the probabilities of all events of an experiment
The number of pencils = 12 (½ Mark) is 1
12 6
∴ The required probability = =  (½ Mark 5. (a) Total number outcomes = 4
22 11 Number of favourable outcomes = 1
4. Total number of pages = 200. 1
The number of numbers less than or equal to 200 that are ∴ Required probability =
4
divisible by 10 is 20. (1 Mark)
Hence, the required probability 6. True

20 1 7. True 15
= =  (1 Mark)
200 10 8. 1
5. Total number of trials = 100 (½ Mark) 9. Elementary
Probability of getting more than one head = P(getting two
heads) + P(getting three heads)
2 Fundamentals of Mathematics

Subjective Type Questions 2. The weights of nine children (in kg) are: 25, 31, 28, 41, 36, 27,
26, 39 and 42
Exercise – 1 Total number of children = 9
The number of children whose weight is lies between 30 kg to
1. Given that, the coin is tossed 1000 times. 40 kg = 3.
∴ The number of possibilities of the occurrence of tail 3 1
= 1000 − 625 = 375 ∴ The required probability = = .
9 3
375 3
∴ Hence, the required probability = = 3. Total number of trials = 200.
1000 8
Head occur 125 times and tail occur 75 times
2. Total number of balls in the box = 20 + 30 + 70 = 120. Probability of getting head
The number of balls which are not black = The total number of Numberof times head occured 125 5
blue and purple balls = 20 + 70 = 90. = = = .
90 3 Total number of trials 200 8
∴ The required probability = = . Probability of getting tail
120 4
Number of times tail occured 75 3
3. Total number of balls played = 30. = = = .
The number of times that Lara hits a six = 6. Total number of trails 200 8
6 1 4. We know that, there are seven days in a week. The number of
∴ The required probability = =
30 5 possibilities that the day selected is Monday = 1.
1
4. The scores of eight students are: 61, 65, 87, 93, 100, 73, 99 and ∴ Required probability =
77 7
Total number of students = 8 5. Two-digit numbers are: 10, 11, 12, … 99.
Number of students who scored less than 77 marks = 3. Total number of two-digit numbers = 90.
3 The multiples of 4 from 10 to 99 are: 12, 16, 20, … 96.
The required probability = .
8 The number of two-digit numbers which are multiplies of
5. Total number of cards = 100. The number of perfect squares 4 is 24 – 2 = 22.
from 101 to 200 are 121, 144, 169, 196 i.e., 4. Hence, the 22 11
∴ The required probability = =
100 − 4 96 24 90 45
required probability = = =
100 100 25 6. The total number of numbers is 1000.
6. There are 8 letters in the word “EQUATION” in which 3 letters The numbers which end with ‘0’ are 10, 20, 30, ….990, 1000
are consonants. i.e., All the multiples 10 end with 0.
3 Therefore, there are 100 such numbers.
Hence, the required probability = .
8 100 1
Hence, the required probability = = .
7. The three-digit numbers are: 100, 101, 102, … 999. 1000 10
The number of three-digit numbers = 999 – 99 = 900 7. Total number of tickets = 50
The number of three-digit odd numbers = 450 From the first 50 numbers, only 7 numbers are perfect squares,
450 1 they are 1, 4, 9, 16, 25, 36, 49.
Hence, the required probability = =
900 2 The number of tickets which are not perfect squares is 43.
8. The scores which are more than 3000 are 5000, 4000 and 3500.
43
The required probability = .
3 50
Hence, the required probability =
7 8. The total number of pages in the book
9. The total number of times the dice is rolled = 800
= 50 + 40 + 10 + 25 + 15 + 80 + 90 + 110 + 120 + 60 = 600
The number of times 6 shows up is 800 − (179 + 121 + 125 + 120 1
∴ The required probability = =
175 + 160) = 40 600 5
40 1 9. Total number of times the coin is tossed = 1000
∴ Hence, the required probability = =
800 20 The number of times 3 heads occur is 1000 − (500 + 100 +
10. Total number of families = 75 150) = 250
The number of families that have only one male child is 38. 250 1
∴ The required probability = =
38 1000 4
Hence, the required probability =
75 10. Total number of trials = 500.
The occurrence of the outcomes 1, 2, 3, 4, 5 and 6 are as
Exercise – 2 follows:
1. Given that, there are 100 cards in the bag. Outcome 1 2 3 4 5 6
The multiples 5 from 1 to 100 are: 5, 10, 15, … 100.
The number of multiples of 5 from 1 to 100 is 20. Frequency 75 80 85 90 95 75
∴ Hence the required probability Favourable cases for getting a prime number is equal to
Number of favourable cases 20 1 (the total number of times) getting the numbers 2 or 3 or 5.
= = = .
Total number of cases 100 5 The number of favourable cases = 80 + 85 + 95 = 260.
Answer and Explanations 3

Hence the required probability 12 1


260 13 ∴ Required probability = = .
= = 60 5
500 25
5. Bowler ‘A’ takes 5 wickets out off 30 balls number of ball of
which he does not take a wicket = 30 – 5 = 25
HOTS 25 5
∴ Required probability = =
1. When three coins are tossed the total numbers of different 30 6
possibilities are 23 = 8
6. Let the number of good apples be ‘x’.
HHT 
∴ Total number of apples = x + 4
H H T  x 7
P(good apple) = =
H T H x+4 9

H T T ⇒ 9x = 7x + 28 ⇒ 2x = 28
 28
T H H =x = 14

T H T 2
 ∴ Number of good apples= 14.
T T H
T T T  7. Given 1st May is a Tuesday. There will be a total of 5 Tuesdays
7 in May.
∴ The required probability = .
8 1stMay, 8th May, 15th May, 22nd May, 29th May
x 6 5
2. P(red ball) = = ∴ Required probability = .
4 + 6 + x 11 31
x 6
⇒ =  x 
10 + x 11 8. A 
= , x < 20 
 xaN 
⇒ 11x = 60 + 6x
⇒ 5x = 60 Odd prime numbers less than 20 are3, 5, 7, 11, 13, 17, 19
⇒ x = 12 7
∴ The number of red balls = 12. ∴ Required probability = .
19
3. The different possibilities when two fair dice are rolled 9. Given, A = {1, 2, 3, 4, ……. 20}
simultaneously x – 3 < 10
(1, 1),(1, 2), (1, 3),(1, 4),(1, 5),(1, 6) ⇒ x< 13.
(2, 1),(2, 2), (2, 3),(2, 4),(2, 5),(2, 6) The elements in set A which satisfy
(3, 1),(3, 2), (3, 3),(3, 4),(3, 5),(3, 6) x < 13 are1, 2, 3, 4, …….. 12
(4, 1),(4, 2), (4, 3),(4, 4),(4, 5),(4, 6) 12 3
(5, 1),(5, 2), (5, 3),(5, 4),(5, 5),(5, 6) ∴ Required probability = = .
20 5
(6, 1),(6, 2), (6, 3),(6, 4),(6, 5),(6, 6)
The combination where the sum of the numbers is greater 10. The first day of the month is a Wednesday
than 8 Wed, Thus Tue, Wed, Thus Tue, Wed, Thus Tue, Wed, Thus Tue, Wed, Thus
1st 8th 15th 22nd 29th
(3, 6), (4, 5), (4, 6), (5, 4), (5, 5), (5, 6), (6, 3), (6, 4), (6, 5), ↓ ↓ ↓ ↓ ↓
(6, 6)
Number of favourable outcomes ∴ Number of days which start with the letter ‘T’ = 9
∴ Required probability =
Total number of outcomes 9
∴ Required probability = .
10 5 31
= =
36 18 11. Multiple of 7 from 1 to 100 are 7, 14, 21, 28, 35, 42, 49, 56, 63,
4. Let a, b and c group of people 70, 77, 84, 91, 98
1 14 7
≥ 50 = × 60 =10 a ≥ 50 ∴ Required probability = =
6 100 50
1 12. Let the number of green balls be ‘x’
≥ 40 × 60 = 20 50 <b≥ 40
3 ∴ Number of blue balls = 30 – x
but less than 50 30 − x 7
3 P(blue ball) = =
≥ 24 × 60 =4018 <c≥ 24 30 10
10
30 − x
but less than 40 ⇒ =7
3
Maximum marks = 60
40 2400 ⇒ 30 – x = 21 ⇒ x = 30 – 21 = 9
Pass% = 40% =
100
× 60=
100
= 24 ∴ Number of green balls = 9. 15
∴ The total number of students who have scored 24 or more 13. Number of times heads shown up = 520
than 24 = 18 + 20 + 10 = 48 Number of times fails shown up = 1000 – 520 = 480
∴ Number of students who have scored less than 24 480 12
i.e., failed = 60 – 48 = 12 ∴ Probability (fails) = = .
1000 25
4 Fundamentals of Mathematics

14. The total number of out comes when two dice are rolled Number of vowels = 5 (a, e, i, o, u)
simultaneously = 6 × 6 = 36 Number of consonants = 26 – 5 = 21
The outcomes where the product of the numbers is a perfect 21
∴ Required probability = .
square is (1, 1), (2, 2),(3, 3),(4, 4),(5, 5),(6, 6) 26
6
∴ Required probability = . 18. Average of 1, 1, 2, 2, 3, 3, 3, 4, 4, 5, 5
36
1+1+ 2 + 2 + 3 + 3 + 3 + 4 + 4 + 5 + 5
15. Given, A = {–3, –2, –1, 0, 1, 2, 3} =
11
x2 – 1 = 0
33
⇒ (x + 1) (x – 1) = 0 ⇒ x = 1, x = –1 = =3
∴ The integers in ‘A’ which satisfy the equation x2 – 1 = 0 11
are 1, –1. 3
2 ∴ Required probability =
11
∴ Required probability = .
7
19. Number of red balls = 2
16. Months which start with a ‘J’ are January, June, July Number of white balls = 3
3 1 2 2
∴ Required probability = = . Probability of picking a red ball = =
12 4 2+3 5
17. A = {letters of English alphabet} 2 3
∴ Probability of picking a ball that is not red = 1 − =.
Number of letters in English alphabet = 26 5 5

You might also like